Вы находитесь на странице: 1из 246

If the tongue deviates to the right side when protruded, the most likely cause is paralysis of

which of the following muscles?


A. Left genioglossus
B. Left hyoglossus
C. Left palatoglossus
D. Right genioglossus
E.

Right hyoglossus

The correct answer is D. The genioglossus muscle is innervated by the hypoglossal nerve. The
function of the genioglossus muscle is to pull the tongue forward (protrude) and toward the
opposite side. When the right genioglossus muscle is paralyzed, the left genioglossus muscle
pulls the tongue forward and to the right.
If the left genioglossus muscle were paralyzed (choice A), the tongue would deviate toward
the left on protrusion because of the unopposed action of the right genioglossus muscle. The
left genioglossus muscle is innervated by the left hypoglossal nerve.
The hyoglossus muscles (choices B and E) are innervated by the hypoglossal nerves. The
function of these muscles is to retract the tongue. These muscles are not active during
protrusion of the tongue.
The palatoglossus muscle (choice C) are innervated by the vagus nerves rather than the
hypoglossal nerves. Their function is to pull the tongue back (retract) and upward toward the
palate.

A patient experiences visual difficulties. When a light is shined in her right eye, there is no
pupillary response in either eye. However, upon shining a light in her left eye, both ipsilateral
and contralateral pupillary responses are apparent. Her extraocular movements are intact.
What is the most likely location of her lesion?
A. Oculomotor nerve, left side
B. Oculomotor nerve, right side
C. Optic nerve, left side
D. Optic nerve, right side
E.

Trochlear nerve, left side

The correct answer is D. Know your cranial nerves! This woman has a "Marcus-Gunn pupil"
with a defect in the afferent pathway of the optic nerve (in this case, on the right side). Recall
that the afferent limb of the pupillary light reflex is the optic nerve (CN II); the efferent limb is
the oculomotor nerve (CN III; parasympathetic fibers). When light is shined into her right eye,
because her right optic nerve is not functioning properly, the light signal is not transmitted to
the central nervous system (CNS), resulting in no pupillary response. As light is shined into
her left eye, the left optic nerve transmits the signal to the CNS, which then sends an
outbound signal through both the right and left oculomotor nerves to cause pupillary
constriction in both eyes. Anatomically, the optic nerve is a special sensory nerve that
originates in the retina of the eye, passes through the optic foramen of the sphenoid bone,
and has a destination in the diencephalon via the optic chiasm.
The oculomotor nerve (choices A and B) innervates all extraocular muscles except the lateral
rectus (innervated by the abducens nerve) and the superior oblique (innervated by the
trochlear nerve choice E). The oculomotor nerve also mediates pupillary constriction
(parasympathetic fibers), eyelid opening (levator palpebrae), and innervates the ciliary
muscle (allowing accommodation).
A patient with a lesion of the left optic nerve (choice C) would have no pupillary responses in
either eye when shining a light in the left eye; pupillary responses would be present in both
eyes when shining a light in the right eye.

A newborn baby is noted to have a left unilateral cleft lip. There are no abnormalities of the
baby's palate. Which of the following developmental defects accounts for this occurrence?
A. Failure of the left lateral palatine process to fuse with the median palatine process
Failure of the left maxillary prominence to unite with the left medial nasal
B. prominence
C. Failure of the primary palate to fuse with the secondary palate
D. Failure of the right and left medial nasal prominences to merge
E.

Failure of the right palatine process to fuse with the left palatine process

The correct answer is B. In the formation of the upper lip, the right and left medial nasal
prominences merge to form the philtrum of the upper lip. The lateral maxillary prominence
then merges with the merged medial nasal prominences. Failure of this merger to occur
results in a unilateral cleft lip.
Failure of a lateral palatine process to fuse with the median palatine process (choice A),
which is synonymous with a failure of the primary palate to fuse with the secondary palate
(choice C), results in a primary cleft palate. Primary clefts of the palate are found anterior to
the incisive foramen of the palate. Primary cleft palates may occur along with cleft lips but
are the result of a separate developmental defect.
Failure of the right and left medial nasal prominences to merge (choice D) results in a median
cleft lip. This is a rare anomaly. Normally, the right and left medial nasal prominences merge
into a single prominence that forms the philtrum of the lip.
Failure of the right and left palatine processes to fuse (choice E) results in a secondary cleft
palate. Secondary cleft palates are found posterior to the incisive foramen. Normally, the
right and left palatine processes fuse together and fuse to the primary palate.

Most fascia of the body that attach to bones attach by which of the following mechanisms?
A. Blending with the covering periosteum
B. Inserting deeply into the cancellous bone
C. Inserting deeply into the cartilage
D. Inserting deeply into the diaphysis
E.

Inserting deeply into the marrow

The correct answer is A. Fascial straps (retinacula) and fascial coverings of muscles or muscle
groups characteristically attach to nearby bones by blending with the covering periosteum.
No deep attachments are usually made by fascia.
Cancellous bone (choice B) is spongy bone, which is usually found in marrow and is not the
site for fascial attachment.
Fascia do not usually attach to cartilage (choice C).
Fascia attaches to bony shafts, or diaphyses (choice D), superficially via the periosteum.
Fascia do not penetrate the bone to reach the marrow (choice E).

Contraction of which of the following muscles contributes most to the backward movement
of the lower jaw during the process of mastication?
A. Digastric
B. Lateral pterygoid
C. Medial pterygoid
D. Mylohyoid
E.

Temporalis

The correct answer is E. Mastication is a complex process involving alternating elevation,


depression, forward movement, and backward movement of the lower jaw. The backwardmovement step is accomplished by the posterior fibers of the temporalis muscle. The
temporalis originates along the temporal lines of the temporal bone and inserts into the
coronoid process of the mandible. It is innervated by the manibular branch of the trigeminal
nerve.
The digastric (choice A) helps to depress (open) the lower jaw during chewing.
The lateral pterygoid (choice B) helps to move the lower jaw forward, laterally, or downward
during chewing.
The medial pterygoid (choice C) helps to elevate (close) the lower jaw during chewing.
The mylohyoid (choice D) helps to depress (open) the lower jaw during chewing.

Several arterial branches penetrate into the distal end of the lesser curvature of the stomach.
Which of the following arteries usually supplies these branches?
A. Left gastric
B. Left gastroepiploic
C. Right gastric
D. Right gastroepiploic
E.

Short gastric

The correct answer is C. The arterial supply of the stomach is complex; it therefore has a
good potential to appear on the NBDE. The right gastric artery supplies the distal lesser
curvature.
The left gastric artery (choice A) supplies the proximal lesser curvature.
The left gastroepiploic artery (choice B) supplies the proximal greater curvature below the
splenic artery.
The right gastroepiploic artery (choice D) supplies the distal greater curvature.
The short gastric artery (choice E) supplies the proximal greater curvature above the splenic
artery.

During a cranial nerve test, the patient cannot elevate her right eye from the abducted
position. Which of the following muscles is paralyzed?
A. Right inferior oblique
B. Right inferior rectus
C. Right lateral rectus
D. Right superior oblique
E.

Right superior rectus

The correct answer is E. The superior rectus muscle can elevate and adduct the eye from the
neutral position. From the abducted position, it is the only muscle that can elevate the eye.
The inferior oblique muscle (choice A) can elevate and abduct the eye from the neutral
position. From the adducted position, it is the only muscle that can elevate the eye.
The inferior rectus muscle (choice B) can depress and adduct the eye from the neutral
position. From the abducted position, it is the only muscle that can depress the eye.
The lateral rectus muscle (choice C) can abduct the eye.
The superior oblique muscle (choice D) can depress and abduct the eye from the neutral
position. From the adducted position, it is the only muscle that can depress the eye.

The two most important hormones controlling Calcium metabolism are secreted from the:
A. Parathyroid and thyroid glands
B. Thyroid and thymus glands
C. Adenohypophysis and parathyroid glands
D. Adrenal cortex and thyroid glands
E.

Parathyroid and adrenal glands

The correct answer is A.PTH, or parathyroid hormone, is considered to be the most


important hormone controlling calcium metabolism. It increases serum calcium by increasing
bone mineral resorption by osteoclasts, increasing digestive absorption of calcium, and
decreasing calcium excretion by the kidney. PTH is secreted by principal cells of the
parathyroid. The antagonist hormone, calcitonin, is secreted by parafollicular cells of the
thyroid (follicular cells secrete thyroxin). It reduces serum calcium by promoting calcium
depositon in bones, decreasing digestive absorption of calcium, and promoting calcium
excretion. Note that the term adenohypophysis in answer choice C refers to the anterior
pituitary. This term and neurohypophysis (posterior pituitary) are often used on NBDE.

Compression of a cranial nerve by a large aneurysm of the right superior cerebellar artery
immediately distal to its origin from the basilar artery would cause which of the following
clinical findings?
A. Loss of abduction of the right eye
B. Loss of adduction of the right eye
C. Loss of depression of the right eye from the adducted position
D. Loss of sensation on the right side of the face
E.

Loss of visual field of the right eye

The correct answer is B. The oculomotor nerve emerges from the interpeduncular fossa of
the midbrain and then passes between the superior cerebellar artery and the posterior
cerebral artery immediately lateral to the basilar artery. Aneurysm of any of these three
arteries may compress the nerve. The oculomotor nerve innervates a number of extraocular
muscles in the orbit, including the medial rectus muscle, which is responsible for adduction of
the eye.
Abduction of the eye (choice A) is accomplished by the lateral rectus muscle. This muscle is
innervated by the abducens nerve, which arises more caudally from the brainstem and is not
in contact with this artery.
Depression of the eye from the adducted position (choice C) is accomplished by the superior
oblique muscle. This muscle is innervated by the trochlear nerve, which emerges from the
dorsal surface of the midbrain and passes lateral to the cerebral peduncle before entering
the cavernous sinus. A superior cerebellar artery aneurysm would not compress this nerve.
Sensation on the face (choice D) is mediated by the trigeminal nerve. This nerve arises from
the anterolateral surface of the pons and is not in contact with this artery.
Loss of visual field (choice E) results from lesions of the optic nerve or other elements of the
visual pathway. These are not in contact with this artery.

A patient with paresthesia of the medial side of the hand and forearm undergoes surgery to
correct it. If the anterior scalene muscle is cut during this surgery, what nearby nerve must be
avoided?
A. Phrenic nerve
B. Recurrent laryngeal nerve
C. Superior laryngeal nerve
D. Suprascapular nerve
E.

Vagus nerve

The correct answer is A. The patient is suffering from scalene triangle syndrome (thoracic
outlet syndrome). The lower trunk of the brachial plexus and the subclavian artery are being
compressed between the anterior scalene muscle and the middle scalene muscle. Incision of
the anterior scalene muscle will relieve this compression. The phrenic nerve lies on the
anterior surface of the anterior scalene muscle deep to the prevertebral fascia. If this nerve is
cut, the diaphragm on that side of the body will be paralyzed. Anatomically, the phrenic
nerve is one of the nerves of the cervical plexus. This nerve distributes to the diaphragm
other nerves of the cervical plexusm including the ansa cervicalis, lesser occiptital, transverse
cervical, supraclavical, and greater auricular nerves, as well as the cervical nerves.
The recurrent laryngeal nerve (choice B) is a branch of the vagus nerve. On the left side it
recurs around the aortic arch, and on the right side it recurs around the right subclavian
artery. It ascends to the larynx in the tracheoesophageal groove. The nerve innervates
several laryngeal muscles and the laryngeal mucosa inferior to the vocal folds. It is not in
contact with the anterior scalene muscle.
The superior laryngeal nerve (choice C) is a branch of the vagus nerve that arises just after
the vagus nerve passes through the jugular foramen. It innervates the cricothyroid muscle of
the larynx and the laryngeal mucosa above the vocal folds. The superior laryngeal nerve is
not in contact with the anterior scalene muscle.
The suprascapular nerve (choice D) is a branch of the upper trunk of the brachial plexus. It
arises from the upper trunk after the upper trunk has passed between the anterior and
middle scalene muscles. The suprascapular nerve innervates the supraspinatus and
infraspinatus muscles.
The vagus nerve (choice E) lies within the carotid sheath within the neck. The carotid sheath
is anterior to the prevertebral fascia. The vagus nerve is not in contact with the anterior
scalene muscle.

A patient's left hypoglossal nerve (CN XII) is injured during a carotid endarterectomy. Which
of the following would most likely result from this injury?
A. Decreased gag reflex on the left
B. Decreased salivation from the left submandibular and sublingual salivary glands
C. Deviation of the tongue to the left on protrusion
D. Inability to elevate the pharynx on the left during swallowing
E.

Inability to perceive sweet and salt taste sensation on the anterior part of the left
side of the tongue

The correct answer is C. The hypoglossal nerve is a pure motor nerve (general somatic
efferent) to the intrinsic and most extrinsic muscles of the tongue. If the nerve is damaged,
denervation atrophy of the affected side will permit the intact musculature of the opposite
side to operate unopposed, thereby protruding the tongue to the side of the injury.
The gag reflex (choice A) is mediated by the glossopharyngeal nerve (CN IX; afferent limb)
and the vagus nerve (CN X; efferent limb).
Choice B is incorrect because the preganglionic parasympathetic fibers that regulate these
two salivary glands are carried by the chorda tympani (which joins with the lingual nerve) to
the submandibular ganglion. Postganglionic fibers are then distributed to these glands.
The muscles responsible for elevation of the pharynx (choice D) are innervated primarily by
the vagus nerve (CN X).
Choice E is incorrect because the taste fibers for the anterior two thirds of the tongue are
carried via the chorda tympani to the facial nerve (CN VII) and hence to the brainstem.

Which of the following structures is lined with epithelium derived from mesoderm of the
ureteric bud?
A. Bowman's capsule
B. Distal convoluted tubule
C. Loop of Henle
D. Proximal convoluted tubule
E.

Ureter

The correct answer is E. The transitional epithelium that lines the ureter, the renal pelvis,
and the major and minor calyces is derived from mesoderm of the ureteric bud, as is the
cuboidal epithelium of the collecting tubules.
The simple squamous epithelium lining Bowman's capsule (choice A) is derived from
mesoderm of the metanephric vesicle.
The simple cuboidal epithelium lining the distal convoluted tubule (choice B) is derived from
mesoderm of the metanephric vesicle.
The simple squamous epithelium lining the loop of Henle (choice C) is derived from
mesoderm of the metanephric vesicle.
The proximal convoluted tubule (choice D) is lined with simple columnar epithelium derived
from mesoderm of the metanephric vesicle.

An infant is born with an abnormally developed falciform ligament. The hepatogastric and
hepatoduodenal ligaments are also malformed. These developmental anomalies are most
likely due to abnormal development of the
A. dorsal mesoduodenum
B. dorsal mesogastrium
C. pericardioperitoneal canal
D. pleuropericardial membranes
E.

ventral mesentery

The correct answer is E. The ventral mesentery forms the falciform ligament, ligamentum
teres, and lesser omentum, which can be divided into the hepatogastric and hepatoduodenal
ligament.
The dorsal mesoduodenum (choice A) is the mesentery of the developing duodenum, which
later disappears so that the duodenum and pancreas lie retroperitoneally.
Both omental bursa and the greater omentum are derived from the dorsal mesogastrium
(choice B), which is the mesentery of the stomach region.
The pericardioperitoneal canal (choice C) embryologically connects the thoracic and
peritoneal canals.
The pleuropericardial membranes (choice D) become the pericardium and contribute to the
diaphragm.

A patient has a severe headache, and is unable to move his right leg. There is no higher
cortical function loss, but deep deep tendon reflexes and Babinski's sign are absent. The
medial aspect of the cerebral hemispheres is most affected. Which artery is most likely
involved?
A. left anterior cerebral artery
B. left middle cerebral artery
C. left posterior cerebral artery
D. right anterior cerebral artery
E.

right posterior cerebral artery

The correct answer is A. The medial aspect of the cerebral hemispheres is supplied by the
anterior cerebral arteries. The small portion of primary motor cortex located here sends
fibers to spinal-cord segments innervating the lower limbs. Because the corticospinal tracts
cross over to the opposite side in the medulla oblongata, the left cerebral hemisphere
controls the right side of the body. Hence, paralysis of the right leg results from loss of blood
flow to the portion of the left hemisphere supplied by the left anterior cerebral artery.
The middle cerebral artery (choice B) supplies the lateral convexity of the brain. Primary
motor cortices on the lateral aspects of the hemispheres send fibers to the brainstem
(innervating the face) and to spinal-cord segments innervating the upper limbs, the trunk,
and the proximal part of the lower extremities. As with the anterior cerebral artery
territories, the right hemisphere innervates the left body, and the left hemisphere innervates
the right body.
The posterior cerebral artery (choices C and E) supplies the occipital cortex; interruption of
blood flow through this artery would probably produce visual deficits rather than paralysis.
The right anterior cerebral artery (choice D) supplies blood to the medial aspect of the right
cerebral hemisphere, which controls the left leg. This patient has paralysis of the right leg.

Following the healing of a tibial fracture, a patient forms a small, new piece of viable bone
near the joint (heterotopic ossification), which is interfering with joint mobility. Which of the
following is the source of the osteoblasts that formed this bone?
A. Cancellous bone near the fracture
B. Circulating stem cells
C. Compact bone near the fracture
D. Marrow
E.

Periosteum

The correct answer is E. The source of bone-forming cells following a fracture is the damaged
periosteum. Occasionally, some of these cells become "lost" and can form small pieces of
inappropriately located bone that impair joint movement. Except in joint cavities, where they
are covered by a layer of hyaline cartilage, bone surfaces are covered by the periosteum. The
periosteum is composed of an outer fibrous and inner cellular layer. The periosteum assists in
the attachment of a bone to surrounding tissues and associated tendons and ligaments.
Surprisingly, neither the cancellous (choice A) nor the compact (choice C) bone near the
fracture is the source of the bone-forming cells.
Circulating stem cells (choice B) can repopulate damaged bone marrow (choice D), but
neither the bone marrow nor the circulating stem cells is the source of the bone-forming
cells.

Injury at the lower border of a rib will most likely damage which structure?
A. intercostal artery
B. intercostal nerve
C. intercostal vein
D. internal intercostal muscle

The correct answer is B. The three structures in the intercostal space are, from superior to
inferior, the intercostal vein, artery, and nerve. The proper site for insertion of an intercostal
drain is superior to a rib, not directly at the level of the superior border but slightly higher to
avoid the collateral branches of the nerve, artery, and vein. The nerve is the most inferior
structure and thus most likely to be damaged by the drain (producing an anesthetic
dermatome). In addition, the nerve is the least protected by the costal groove.
The intercostal artery (choice A) is the middle structure and thus is not the most likely to be
damaged.
The intercostal vein (choice C) is the most superior structure and is thus the least likely to be
damaged. In addition, the vein is the structure best protected by the costal groove (the
further superior the structure, the more protected by the costal groove).
The internal intercostal fibers (choice D) will tend to be separated by the drain. In addition,
the internal intercostal muscle is membranous (internal intercostal membrane) posteriorly,
from the neck of the rib to its angle; therefore, many drains, when inserted, will not even
pass through a layer of internal intercostal muscle.

Hirschsprung's disease indicates a developmental abnormality in which of the following


embryonic tissues?
A. Ectoderm
B. Endoderm
C. Neural crest
D. Neural ectoderm
E.

Splanchnic mesoderm

The correct answer is C. The baby has Hirschsprung's disease, which is due to an absence of
ganglion cells in the wall of the colon. Neural crest cells contribute to the formation of many
adult structures. Among these are all of the postganglionic neurons of the autonomic
nervous system and the sensory neurons of the peripheral nervous system.
Ectoderm (choice A) forms the epidermis of the skin and the parenchymal cells of glands
associated with the skin such as the sweat glands, sebaceous glands, and mammary glands.
Endoderm (choice B) forms the epithelial lining of the gut tube and the parenchymal cells of
glands associated with the gut tube, such as the liver and pancreas.
Neural ectoderm (choice D) forms the central nervous system, the somatic motor neurons of
the peripheral nervous system, and the preganglionic neurons of the autonomic nervous
system.
Splanchnic mesoderm (choice E) forms the visceral peritoneum, visceral pleura, visceral
pericardium, and the stroma and muscle of the wall of the gut, among other structures.

An abrasion results in the total loss of epidermis over a large area of an arm, but one month
later, the abrasion has healed, with regrowth of the epidermis. Which of the following
mechanisms accounts for the restoration of the epidermis over the abraded area?
A. Growth of epidermis from hair follicles and sweat glands in the dermis
B. Migration of endothelial cells from newly grown capillaries
C. Transformation of dermal fibroblasts into epidermal cells
D. Transformation of macrophages into epidermal cells
E.

Transformation of melanocytes into epidermal cells

The correct answer is A. The dermis contains skin appendages (e.g., hair follicles), which
contain epithelial stem cells. In the process of healing a large area where the epidermis has
been lost but the dermis is intact, re-epithelialization occurs by growth of epidermal cells
from the underlying skin appendages, as well as from the intact epidermis along the wound
edges. Physiologically, the dermis lies beneath the epidermis. It has two major components, a
superficial papillary layer and a deeper reticular layer. The papillary layer contains the
capillaries and the sensory neurons, which supply the surface of the skin. The reticular layer
consists of an interwoven meshwork of dense irregular connective tissue.
None of the other cell types are known to directly contribute to the regeneration of
epidermis over abraded skin.

Most of the oocytes in the ovary of a prepubescent girl are in which meiotic stage?
A. Anaphase of the second meiotic division
B. Metaphase of the first meiotic division
C. Metaphase of the second meiotic division
D. Prophase of the first meiotic division
E.

Telophase of the first meiotic division

The correct answer is D. The first meiotic division is the "reduction" meiotic division, in which
the diploid complement of DNA is reduced to a haploid complement. The bulk of oocytes in
premenopausal women, girls, and babies are arrested at prophase of the first meiotic
division. Postmenopausal women have very few viable oocytes. It is important to note that
ovulation occurs before the oocyte is completely mature. The secondary oocyte leaving the
follicle is in metaphase of the second meiotic division (choice C). The cell's metabolic
operations have been discontinued and the oocyte drifts in a state of "suspended
animation," awaiting the necessary stimulus for further development. If fertilization does not
occur, the oocyte disintegrates without completing meiosis.

At which of the following ages does fetal movement first occur?


A. 1 month
B. 2 months
C. 4 months
D. 6 months
E.

7 months

The correct answer is B. Neuromuscular development is sufficient to allow fetal movement


in the eighth week of life. Other features of Week 8 include the first appearance of a thin
skin, a head as large as the rest of the body, forward-looking eyes, appearance of digits on
the hands and feet, appearance of testes and ovaries (but not distinguishable external
genitalia), and a crown-rump length of approximately 30 mm. By the end of the eighth week,
nearly all adult structures have at least begun to develop, and the fetus "looks like a baby."

Which of the following veins empties into the left renal vein?
A. Hepatic
B. Left suprarenal
C. Right gonadal
D. Right renal

The correct answer is B. The left suprarenal vein empties into the left renal vein, which
crosses the vertebral column to reach the inferior vena cava. The left renal vein also
receives the left gonadal vein.
Hepatic veins (choice A) convey blood from the liver to the inferior vena cava as it
approaches the caval orifice of the diaphragm.
The right gonadal vein (testicular in male, ovarian in female; choice C) drains directly to the
inferior vena cava.
The left and right (choice D) renal veins enter the inferior vena cava at vertebral level L2.

Which of the following is the embryologic precursor of the fibrous remnant that runs in a
fissure on the visceral surface of the liver?
A. Ductus venosus
B. Hepatic portal vein
C. Lateral splanchnic artery
D. Ligamentum venosum
E.

Vitelline vein

The correct answer is A. The structure described is the ligamentum venosum (choice D),
which is derived from the ductus venosus.
The hepatic portal vein (choice B) is derived from the vitelline vein (choice E).
The lateral splanchnic arteries (choice C) arise from each side of the dorsal aorta. They
supply intermediate mesoderm and derivatives and give rise to renal, suprarenal, phrenic,
and testicular or ovarian arteries.

A neonate is observed to have a cleft lip. Which of the following is the most likely etiology
of this malformation?
A. Abnormal development of the third and fourth pharyngeal pouches
B. Bony defects of the malar bone and mandible
C. Failure of the maxillary processes and medial nasal swellings to fuse
D. Incomplete joining of the palatine shelves
E.

Insufficient migration of neural crest cells

The correct answer is C. Cleft lip is caused by the failure of the maxillary processes to fuse
with the medial nasal swellings.
Abnormal development of the third and fourth pharyngeal pouches (choice A) can give rise
to DiGeorge syndrome, which is characterized by the underdevelopment or absence of
several structures, including the thymus and parathyroids.
Bony defects of the malar bone and mandible (choice B) are associated with
mandibulofacial dysostosis, which is mainly due to abnormal development of derivatives of
the first arch. This condition is characterized by downward sloping palpebral fissures;
hypoplasia of the malar and mandibular bones; macrostomia; high or cleft palate;
abnormally shaped, low-set ears; and unusual hair growth patterns. Insufficient migration
of neural crest cells (choice E) is an important factor as well.
When the palatine shelves fail to join together (choice D), cleft palate results. Cleft lip and
cleft palate commonly co-occur.

A patient suffers severe head trauma, including a fracture in the region of the foramen
ovale. Which of the following functional losses would most likely be related to this injury?
A. Loss of abduction of the eye
B. Loss of sensation over the forehead
C. Loss of sensation over the zygoma
D. Loss of taste sensation on the anterior 2/3 of the tongue
E.

Paralysis of muscles of mastication

The correct answer is E. The mandibular nerve (V3) passes through the foramen ovale and
may be injured by this fracture. The mandibular nerve is responsible for the innervation of
all of the muscles of mastication: the masseter, the temporalis, the medial pterygoid, and
the lateral pterygoid muscles, as well as the tensor palatini, tensor tympani, and
mylohyoid.
Abduction of the eye (choice A) is produced by the lateral rectus muscle, which is
innervated by the abducens nerve. The abducens nerve leaves the cranial cavity and enters
the orbit by passing through the superior orbital fissure.
Sensation in the skin over the forehead (choice B) is provided by the ophthalmic division of
the trigeminal nerve. The ophthalmic division leaves the cranial cavity and enters the orbit
by passing through the superior orbital fissure.
Sensation in the skin over the zygoma (choice C) is provided by the maxillary division of the
trigeminal nerve. The maxillary division leaves the cranial cavity by passing through the
foramen rotundum and enters the pterygopalatine fossa.
Taste sensation on the anterior 2/3 of the tongue (choice D) is provided by the chordae
tympani, a branch of the facial nerve. The facial nerve leaves the cranial cavity by passing
into the internal auditory meatus and enters the temporal bone. The chordae tympani
leaves the temporal bone by passing through the petrotympanic fissure and enters the
infratemporal fossae.

Which of the following anatomic landmarks is the site of the anterior fontanelle in an
infant?
A. Bregma
B. Coronal suture
C. Lambda
D. Pterion
E.

Sagittal suture

The correct answer is A. Bregma represents the point where the coronal and sagittal
sutures intersect; it is the site of the anterior fontanelle.
The coronal suture (choice B) lies between the frontal and parietal bones.
Lambda (choice C) represents the point where the sagittal and lambdoid sutures intersect;
it is the site of the posterior fontanelle in infants.
The pterion (choice D) is the point on the lateral aspect of the skull where the greater wing
of the sphenoid, parietal, frontal, and temporal bones converge. Recall that the pterion is
the landmark for the middle meningeal artery and that a blow to the temple (e.g., as could
occur in boxing) can lead to a middle meningeal arterial bleed and an epidural hemorhage.
The sagittal suture (choice E) is located between the two parietal bones.

Nissl bodies correspond to which of the following cytoplasmic organelles?


A. Golgi apparatus
B. Mitochondria
C. Nucleoli
D. Rough endoplasmic reticulum
E.

Smooth endoplasmic reticulum

The correct answer is D. Rough endoplasmic reticulum present in neurons is called Nissl
substance, or Nissl bodies. Nissl bodies stain intensely with basic dyes and are found in the
cell body and proximal dendrites, but not in the axon hillock or axon.

Which of the following neurologic deficits would a large meningioma involving the brain's
parasagittal region and the falx cerebri be expected to produce?
A. Altered taste
B. Leg paralysis
C. Loss of facial sensation
D. Ptosis
E.

Unilateral deafness

The correct answer is B. A meningioma (a tumor of the meninges) of the parasagittal


region and the falx cerebri would be located superiorly, between the two hemispheres. In
this position, it could compress the sensory (postcentral gyrus) or motor cortex (precentral
gyrus) supplying the lower extremities.
Taste (choice A) is supplied by cranial nerves VII, IX, and X. These nerves arise in the
brainstem.
Facial sensation (choice C) is supplied by cranial nerve V, the nuclei of which are in the
brainstem.
Ptosis (choice D) can be caused by a deficit in cranial nerve III, which arises from the
brainstem.
Unilateral deafness (choice E) suggests damage to cranial nerve VIII, which arises from the
brainstem.

A mutation affecting the development of the diencephalon could interfere with the
secretion of which of the following hormones?
A. Adrenocorticotrophic hormone (ACTH)
B. Epinephrine
C. Oxytocin
D. Prolactin
E.

Thyroid-stimulating hormone (TSH)

The correct answer is C. The neurohypophysis (posterior pituitary) is derived from an


evagination of the diencephalic neurectoderm. This structure is responsible for releasing
oxytocin and vasopressin to the general circulation. Both hormones are synthesized in cell
bodies contained within the hypothalamus. Oxytocin is a hormone produced by the
hypothalamic cells and secreted into capillaries at the posterior pituitary. It stimulates
smooth muscle contractions of the uterus or mammary glands in the female, but has no
known function in the male.
ACTH (choice A), prolactin (choice D), and TSH (choice E) are all synthesized and released
by the anterior pituitary, or adenohypophysis, which is derived from an evagination of the
ectoderm of Rathke's pouch, a diverticulum of the primitive mouth. Remnants of this
pouch may give rise to a craniopharyngioma in later life.
Epinephrine (choice B) is synthesized and released into the circulation by the adrenal
medulla, a neural crest derivative.

Sensory innervation of the tongue, in order, from anterior to posterior, shows the effect of
development of which branchial arches?
A. First, second, third
B. Third, second, first
C. Second, third, fourth
D. Fourth, third, second

The correct answer is A. Although many students memorize the innervation of the tongue,
it can also be explained in a developmental way. In particular, the tongue is derived from
the first three pharyngeal (branchial) arches, and the cranial nerves associated with each
arch appear from anterior to posterior in the tongue innervation. Note that some texts also
state that the most posterior sections of the tongue are developed form the fourth arch as
well. The cranial nerves of each arch are as follows: Arch 1 (CN V), Arch 2 (CN VII), Arch 3
(CN IX), and Arch 4 (CN X). Note that general sensation from the anterior two thirds of the
tongue is supplied by CN V, taste by CN VII, taste and general sensation from the posterior
one third are supplied by CN IX, and some innervation of the most posterior tongue by CN
X.

Which of the following cell types is derived from neuroepithelial cells?


A. Astrocytes
B. Enterochromaffin cells
C. Melanocytes
D. Odontoblasts
E.

Schwann cells

The correct answer is A. Astrocytes and oligodendrocytes are both derived from glioblasts,
which, in turn, are derived from neuroepithelial cells. Other neuroepithelial cell derivatives
include neuroblasts and ependymal cells. The astrocytes are the largest and most
numerous glial cells. These cells are responsible for maintaining the blood-brain barrier,
creating a three-dimensional framework for the central nervous system, performing
repairs in damaged neural tissues, and controlling the interstitial environment.
All the other choices are derived from neural crest cells. Other neural crest derivatives
include the neurons of the parasympathetic and sympathetic ganglia (including the adrenal
medulla), the dorsal root ganglia of the peripheral nervous system, the sensory ganglia of
cranial nerves V, VII, IX, and X, and the leptomeninges (pia and arachnoid).

During a mastectomy, the surgeon notes that the breast tumor has spread to involve the
muscle layer immediately deep to the breast. Which muscle is involved?
A. External oblique
B. Pectoralis major
C. Platysma
D. Rectus abdominis
E.

Sternocleidomastoid

The correct answer is B. The muscle deep to the breast is the pectoralis major. Breast
cancer can invade this muscle but usually does not because the relatively thick, deep
fascial layer serves as a barrier. Formerly, the pectoralis major was removed during radical
mastectomy, leaving the patient with a major loss of function of the adjacent arm. The
now more frequently used modified radical mastectomy spares the pectoralis major.
The external obliques (choice A) and rectus abdominis (choice D) muscles are in the
abdomen.
The platysma (choice C) and sternocleidomastoid (choice E) muscles are in the neck.

A woman who recently gave birth has elevated prolactin levels. The gland responsible for
secretion of this hormone is derived from which of the following structures?
A. Cerebral vesicle
B. Infundibulum
C. Neurohypophysis
D. Proctodeum
E.

Rathke's pouch

The correct answer is E. The anterior pituitary produces prolactin. The structure originates
from Rathke's pouch, which is itself a diverticulum of the roof of the stomodeum.
The cerebral vesicle (choice A) lies close to Rathke's pouch.
The infundibulum (choice B) comes in contact with Rathke's pouch at the fifth week of
development.
The neurohypophysis (choice C) gives rise to the posterior pituitary.
The proctodeum (choice D) is also known as the anal pit.

A parent is startled to find that squeezing her infant's shoulders with her hand can bring
them nearly together in front of the body. Congenital absence of which of the following
bones should be suspected?
A. Clavicles
B. First ribs
C. Humeri
D. Scapulae
E.

Sternum

The correct answer is A. Congenital absence of the clavicles is a fairly common and not
particularly troublesome anomaly, although affected children have some trouble with
sports. It can be confirmed radiologically. In rare (particularly, obese) patients, the defect is
not diagnosed until a routine chest x-ray performed for other reasons demonstrates the
anomaly.
Congenital defects in ribs (choice B) can occur but would not allow the shoulders to be
brought together.
Congenital absence of the humeri (choice C) would produce absent or markedly shortened
upper arms.
Congenital anomalies of the scapulae (choice D) are rare and would affect posterior
shoulder movement.
Congenital absence of part of the sternum (choice E) produces a hole in the lower part of
the body of the sternum; complete absence is rare.

A man suffers a direct blow to the right eye, producing eye pain and severe bleeding from
the nose. X-rays reveal a blowout fracture of the floor of the orbit. Involvement of which of
the following structures is suggested by the patient's symptoms?
A. Ethmoid air cells
B. Maxillary sinus
C. Oral cavity
D. Pterygopalatine fossa
E.

Sphenoid sinus

The correct answer is B. The floor of the orbit is also the roof of the maxillary sinus. The
bone separating the orbit from the maxillary sinus is frequently quite thin and may fracture
from increased intraorbital pressure caused by a direct blow to the eye. The subsequent
bleeding into the maxillary sinus will result in blood draining from the sinus into the nasal
cavity. The maxillary sinuses are the largest sinuses in the skull. These sinuses lighten the
portion of the maxillae above the teeth and produce mucous secretions that help to
"flush" the interior surfaces of the nasal cavities.
The ethmoid air cells (choice A) are medial to the orbit. This bone is also frequently quite
thin.
The oral cavity (choice C) is separated from the orbit by the maxillary sinus.
The pterygopalatine fossa (choice D) is posteromedial to the orbit. The medial portion of
the inferior orbital fissure communicates between the orbit and the pterygopalatine fossa.
The sphenoid sinus (choice E) is posteromedial to the orbit. There is no communication
between the sphenoid sinus and the orbit.

Which area of the stomach has numerous, normal cuboidal-to-columnar cells with apical,
membrane-bound secretion granules in the gastric glands?
A. Cardiac region
B. Columns of Morgagni
C. Fundic region
D. Greater omentum
E.

Pyloric region

The correct answer is C. The pathologist saw normal chief cells, which are abundant in the
body and fundus of the stomach. Chief cells secrete pepsinogen, which is stored in apical
membrane-bound granules. The body and fundus of the stomach contain high
concentrations of four other types of cells in the epithelium. The parietal (oxyntic) cells are
large, pyramidal, and acidophilic with central nuclei (like a "fried egg"). They make and
secrete hydrochloric acid. The mucous neck cells secrete mucus and appear clear. The
enteroendocrine cells have affinity for silver stains and exhibit a positive chromaffin
reaction; these cells synthesize amines, polypeptides, or proteins.
The cardiac region (choice A) is a narrow, circular band at the transition between the
esophagus and stomach, consisting of shallow gastric pits and mucous glands. It does not
normally contain an abundance of chief cells.
The columns of Morgagni (choice B) are found in the rectum, not in the stomach. These are
mucous membrane infoldings in the submucosa of the proximal anal canal. They would not
contain chief cells.
The greater omentum (choice D) is a four-layered fold of peritoneum that hangs from the
greater curvature of the stomach and attaches to the transverse colon. It would not
contain chief cells.
The pyloric region (choice E) has deep gastric pits into which tubular glands open. The
predominant secretion is mucus. It does not normally contain an abundance of chief cells.
Note that in this question you could have automatically eliminated choices B and D
because they are not gastric structures. If nothing else, you have improved your guessing
odds to 33%.

A knife wound to the face selectively damages the facial nerve. Salivation from which of
the following would be impaired?
A. Parotid and sublingual glands
B. Parotid and submandibular glands
C. Parotid gland only
D. Sublingual gland only
E.

Submandibular and sublingual glands

The correct answer is E. The facial nerve (CN VII) is intimately related anatomically to the
parotid gland but controls salivation from both the submandibular and sublingual glands.
The sublingual salivary glands are located beneath the mucous membrane on the floor of
the mouth. The submandibular salivary glands are found in the floor of the mouth along
the inner surface of the mandible within the manibular groove.
The parotid gland (choices A, B, and C) is innervated by the glossopharyngeal nerve (CN IX).
The parotid glands are located inferiorly to the zygomatic arch beneath the skin covering
the lateral and posterior surfaces of the mandible. Although the facial nerve most often
runs through the parotid gland, it does not supply any innervation there.

Following thyroid surgery, hoarseness and difficulty speaking can be attributed to damage
to a branch of which cranial nerve?
A. Facial
B. Glossopharyngeal
C. Hypoglossal
D. Trigeminal
E.

Vagus

The correct answer is E. The recurrent laryngeal nerves are branches of the vagus (CN X)
and supply all intrinsic muscles of the larynx, except the cricothyroid. The right recurrent
laryngeal nerve recurs around the right subclavian artery. The left recurrent laryngeal
nerve recurs in the thorax around the arch of the aorta and ligamentum arteriosum. Both
nerves ascend to the larynx by passing between the trachea and esophagus, in close
proximity to the thyroid gland. The recurrent laryngeal nerves are therefore particularly
vulnerable during thyroid surgery, and damage may cause extreme hoarseness.
The facial nerve (choice A) innervates the muscles of facial expression, the stapedius
muscle, and the lacrimal, submandibular, and sublingual glands. It also mediates taste
sensation from the anterior two-thirds of the tongue.
The glossopharyngeal nerve (choice B) innervates the stylopharyngeus muscle and the
parotid gland. Visceral afferents supply the carotid sinus baroreceptors and carotid body
chemoreceptors and mediate taste from the posterior one-third of the tongue.
Somatosensory fibers supply pain, temperature, and touch information from the posterior
one-third of the tongue, upper pharynx, middle ear, and eustachian tube.
The hypoglossal nerve (choice C) innervates the intrinsic muscles of the tongue, the
genioglossus, hypoglossus, and styloglossus muscles.
The trigeminal nerve (choice D) receives sensory information from the face and also
innervates the muscles of mastication.

At which of the following locations might a penetrating wound to the heart damage the AV
node?
A. Apex of the heart
B. Interatrial septum
C. Interventricular septum
D. Wall of the right atrium
E.

Wall of the left atrium

The correct answer is B. This question is essentially asking "where is the AV node located
on the heart?" The atrioventricular (AV) node is in the subendocardium of the interatrial
septum. The AV node can also be described as being within the floor of the right atrium
near the opening of the coronary sinus. From the AV node, the Purkinje fibers of the
atrioventricular bundle enter the interventricular septum to carry impulses to the ventricle.
The function of the AV node is to retard the conduction of the cardiac impulses so that
ventricular systole occurs after atrial systole.
The apex of the heart (choice A) is composed of ventricular wall musculature of the left
ventricle. There is no nodal tissue in this region.
The interventricular septum (choice C) contains the common bundle (AV bundle) and the
right and left bundle branches of the cardiac conduction system. These bundles are
composed of Purkinje fibers, which are specialized cardiac muscle cells.
The sinoatrial (SA) node is located in the right atrial wall (choice D), near the entrance of
the superior vena cava. The SA node contains specialized cardiac muscle cells that
depolarize more rapidly than do typical cardiac muscle cells, thereby serving as the
pacemaker of the heart.
The wall of the left atrium (choice E) does not contain any nodal cells.

Inability to extend the hand at the wrist is often accompanied by loss of sensation to which
area?
A. Lower part of the back of the little finger
B. Lower part of the back of the thumb
C. Palmar aspect of the forefinger
D. Palmar aspect of the little finger
E.

Thumbnail bed

The correct answer is B. Associate "wrist drop" with a radial nerve lesion. The radial nerve
also provides sensation to the dorsal aspect of the radial side of the hand, including the
lower part of the dorsal aspect of the thumb and all the fingers, except for the little finger
and half of the ring finger.
The lower part of the back of the little finger (choice A) is supplied by the ulnar nerve.
The palmar aspects of all of the fingers (choices C and D) are supplied by the median nerve,
as are the nailbeds, including the thumbnail bed(choice E).

A premature infant develops a progressive difficulty with breathing over the first few days
of life. Deficient surfactant synthesis by which of the following cell types may have
contributed to the infant's respiratory problems?
A. Alveolar capillary endothelial cells
B. Bronchial mucous cells
C. Bronchial respiratory epithelium
D. Type I pneumocytes
E.

Type II pneumocytes

The correct answer is E. This child has neonatal respiratory distress syndrome (hyaline
membrane disease), which is caused by the inability of the immature lungs to synthesize
adequate amounts of surfactant. Surfactant, which reduces surface tension, helps keep
alveoli dry, and aids in expansion of the lungs, is synthesized by type II pneumocytes.
Alveolar capillary endothelial cells (choice A) are important in maintaining the capillary
structure and permitting flow of gases into and out of the blood stream.
Bronchial mucous cells (choice B) produce the usually thin (in healthy individuals) coat of
mucus that lines the bronchi.
The ciliated bronchial respiratory epithelium (choice C) is responsible for moving the dustcoated mucus layer out of the bronchi.
Type I pneumocytes (choice D) are the squamous cells that line alveoli and permit easy gas
exchange. These cells tend to be immature (and thick) in premature infants but do not
produce surfactant.

Which part of the hypothalamus controls satiety, and would lead to obesity if destroyed?
A. Lateral nucleus
B. Septal nucleus
C. Suprachiasmatic nucleus
D. Supraoptic nucleus
E.

Ventromedial nucleus

The correct answer is E. The ventromedial nucleus is thought to be the satiety center of
the brain. Bilateral destruction leads to hyperphagia, obesity, and savage behavior.
Stimulation inhibits the urge to eat.
Destruction of the lateral nucleus (choice A) results in starvation, whereas stimulation of
this nucleus induces eating.
Destruction of the septal nucleus (choice B) produces aggressive behavior.
The suprachiasmatic nucleus (choice C) receives direct input from the retina and plays a
role in controlling circadian rhythms.
The supraoptic nucleus (choice D), along with the periventricular nucleus, regulates water
balance and produces antidiuretic hormone (ADH) and oxytocin.

In which of the following regions of the liver is the oxidase P450 system located?
A. Bile ducts
B. Intermediate zone
C. "Ito" cells
D. Pericentral vein zone
E.

Periportal zone

The correct answer is D. The liver can be divided into three zones. Zone 1 is periportal,
zone 2 is intermediate (between 1 and 3), and zone 3 is associated with the central vein.
Zone 3 contains the P450 oxidase enzyme system and is most sensitive to injury. The
CP450 system is very important to be familiar with because it is the major site of drug
interaction. For example, the macrolides, which are commonly used in dentistry in
penicillin-allergic patients are potent hepatic enzyme inhibitors, as are antifungals like
ketoconazole and fluconazole.
The P450 system is not located in the bile ducts (choice A).
The intermediate zone (choice B), or zone 2, is the second area most sensitive to ischemic
injury.
"Ito" cells (choice C) are the fat-containing mesenchymal cells located in the space of Disse.
This is the site of vitamin A storage.
The periportal zone (choice E), or zone 1, is the area most sensitive to toxic injury. This is
the area that shows infiltration with hepatitis.

Which of the following gastrointestinal hormones is structurally related to secretin?


A. Cholecystokinin
B. Gastrin
C. Glucagon
D. Somatostatin
E.

Substance P

The correct answer is C. Glucagon, secretin, and vasoactive intestinal polypeptide (VIP) are
all structurally related.
Cholecystokinin (choice A) and gastrin (choice B) form another family of related hormones.
Neither somatostatin (choice D) nor substance P (choice E) are structurally related to
secretin. In addition to their role in the gastrointestinal system, both hormones are also
present in the brain.

Which of the following products would mast cells be most likely to secrete?
A. Bradykinin
B. Complement factor 3a
C. Histamine
D. Interleukin-2
E.

Nitric oxide

The correct answer is C. The cells in question are mast cells, which play an important role
in IgE-mediated allergic responses. It is also important to note that histamine is found in
basophils. They are a normal (minor) constituent of dermal skin and are most definitely
identified with stains, such as the Giemsa stain, that highlight the granularity of their
cytoplasm (mast cells are closely related to blood basophils). Mast cells secrete histamine,
serotonin, many leukotrienes, and platelet-aggregating factor (PAF). Their histamine
secretion after IgE stimulation helps to trigger the acute inflammatory part of the allergic
response.
Bradykinin (choice A) is a nonapeptide found in the plasma. It is a potent vasodilator and
stimulates pain receptors.
Complement factor 3a (choice B) is a plasma protein that induces vascular leakage as part
of the acute inflammatory response.
Interleukin-2 (choice D) is secreted by lymphocytes and augments the immune response.
Nitric oxide (choice E) is released by macrophages and endothelium. It causes vasodilation
and cytotoxicity.

If the lateral wall of the left ventricle receives insufficient blood supply, which artery is
most likely involved?
A. Left anterior descending
B. Left circumflex
C. Left main coronary
D. Right coronary

The correct answer is B. In some patients with coronary artery disease, thallium stress
tests may be performed instead of cardiac catheterization to determine the vessels
involved and the extent of occlusion. The left circumflex (LCx) branch supplies the lateral
wall of the left ventricle; in 10% of the population, it also supplies the posterior wall and
AV node.
The left anterior descending (LAD) branch of the left coronary artery (choice A) supplies
the anterior wall of the left ventricle and the anterior portion of the interventricular
septum.
The left main coronary artery (choice C) gives rise to both LCx and the LAD.
The right coronary artery (RCA; choice D) supplies the right ventricle; in 90% of the
population, it supplies the AV node and posterior and inferior walls of the left ventricle.

While performing a tonsillectomy, the surgeon accidentally damages the glossopharyngeal


nerve. Which of the following functional losses is likely to result from this injury?
A. Loss of sensation on the posterior 1/3 of the tongue
B. Loss of taste on the anterior 2/3 of the tongue
C. Paralysis of the constrictor muscles of the palate
D. Paralysis of the muscles of the soft palate
E.

Paralysis of the muscles of the tongue

The correct answer is A.The glossopharyngeal nerve, or cranial nerve IX, lies in the tonsillar
fossa. This nerve provides general sensory innervation to the mucosa of the pharynx, and
general sensory and taste sensation to the mucosa of the posterior 1/3 of the tongue. The
nerve also innervates the stylopharyngeus muscle, but this innervation occurs proximal to
the point at which the nerve crosses the tonsillar fossa. The full anatomical picture of the
glossopharyngeal nerve is as follows: It originates from the posterior 1/3 of the tongue,
part of the pharynx and palate, as well as the carotid arteries of the neck (sensory). It
passes through the jugular foramen of the temporal bone to foramen lacerum between
the occipital and temporal bones. The destination is to the sensory nuclei of the medulla
oblongata, pharyngeal muscles involved in swallowing, as well as the parotid.
Taste sensation to the anterior 2/3 of the tongue (choice B) is provided by the facial nerve.
The chordae tympani branch of the facial nerve travels with the lingual nerve to reach the
tongue.
The constrictor muscles of the pharynx (choice C) are innervated by the vagus nerve. The
vagus nerve fibers contribute to the pharyngeal plexus.
The muscles of the soft palate (choice D) are mostly innervated by the vagus nerve. The
tensor palati is innervated by the trigeminal nerve (V 3).
The muscles of the tongue (choice E) are innervated by the hypoglossal nerve.

Within which of the following peritoneal structures is the cystic artery located?
A. Falciform ligament
B. Gastrocolic ligament
C. Gastrohepatic ligament
D. Hepatoduodenal ligament
E.

Splenorenal ligament

The correct answer is D. The hepatoduodenal ligament is the portion of the lesser
omentum that connects the liver to the first part of the duodenum. Within the
hepatoduodenal ligament are found the proper hepatic artery and its branches, the
common bile duct and its branches, and the portal vein. The cystic artery is usually a
branch of the right hepatic artery, which is a branch of the proper hepatic artery.
The falciform ligament (choice A) is a mesentery that connects the liver to the anterior
abdominal wall. Within the free edge of the falciform ligament is found the round ligament
of the liver (ligamentum teres hepatis), the adult remnant of the umbilical vein.
The gastrocolic ligament (choice B) is the portion of the greater omentum between the
greater curvature of the stomach and the transverse colon. The gastroepiploic arteries lie
within the gastrocolic ligament along the greater curvature of the stomach.
The gastrohepatic ligament (choice C) is the portion of the lesser omentum between the
liver and the lesser curvature of the stomach. The right and left gastric arteries are within
the gastrohepatic ligament along the lesser curvature of the stomach.
The splenorenal ligament (choice E) is the mesentery that connects the spleen to the
posterior abdominal wall. The splenic artery and splenic vein are within the splenorenal
ligament.

In a histological section of a normal ovary, an oocyte is surrounded by several layers of


follicular cells. A small antrum is present. Which of the following is the correct term for the
entire structure, composed of the oocyte, follicular cells, and antrum?
A. Corpus luteum
B. Graafian follicle
C. Primary follicle
D. Primordial follicle
E.

Secondary follicle

The correct answer is E. Follicles in different stages of maturation have different


appearances. The most primitive follicles, primordial follicles (choice D), are inactive
reserve follicles that contain primary oocytes (arrested in prophase of the first meiotic
division) surrounded by a single layer of flattened follicular cells. Primary follicles (choice
C), the next stage, are slightly larger and contain a central oocyte surrounded by one or
several cuboidal follicular cells. When several small spaces in the follicular mass fuse to
form the antrum (follicular cavity), the follicle is termed a secondary follicle (choice E). The
secondary follicles continue to enlarge and develop a more complex structure that includes
cumulus oophorus, corona radiata, theca interna, theca externa, and zona pellucida. The
Graafian follicle (choice B) is the mature form of the follicle, which extends through the
entire cortex and bulges out at the ovarian surface. After it ruptures and releases the
ovum, the corpus luteum (choice A)develops and secretes progesterone as the cells of the
follicle and the theca interna cells enlarge, become epithelioid, and secrete estrogen. If
pregnancy occurs, it is maintained throughout the pregnancy; if pregnancy does not occur,
the corpus luteum eventually degenerates.

Which of the following veins may be anastomosed to accomplish a porto-caval shunt?


A. Left renal vein and left testicular vein
B. Right renal vein and right suprarenal vein
C. Splenic vein and left renal vein
D. Superior mesenteric vein and inferior mesenteric vein
E.

Superior mesenteric vein and splenic vein

The correct answer is C. The splenic vein drains directly into the portal vein. The left renal
vein drains directly into the inferior vena cava. Anastomosis of these veins would allow
blood from the portal vein to drain retrograde though the splenic vein into the renal vein
and then into the inferior vena cava.
The left renal vein (choice A) drains directly into the inferior vena cava. The left testicular
vein drains directly into the left renal vein. Thus, these veins are already in communication,
and neither vein is part of the portal venous system.
The right renal vein (choice B) drains directly into the inferior vena cava. The right
suprarenal vein also drains directly into the inferior vena cava. Thus, neither vein is part of
the portal venous system.
The superior mesenteric vein (choice D) drains directly into the portal vein. The inferior
mesenteric vein drains into the splenic vein, which then drains into the portal vein. Thus,
neither vein is part of the caval venous system.
The superior mesenteric vein (choice E) drains directly into the portal vein. The splenic vein
also drains directly into the portal vein. Thus, neither vein is part of the caval venous
system.

The patient closes both of his eyelids in response to the left eye being touched with a thin
wisp of cotton as he looks to the right. Which of the following cranial nerves is responsible
for the motor limb of this reflex?
A. Abducens
B. Facial
C. Optic
D. Trigeminal
E.

Trochlear

The correct answer is B. The corneal reflex is tested by touching the cornea of one eye
with a cotton wisp; this causes both eyes to close. The afferent, or sensory, component of
the corneal reflex is mediated by the ophthalmic division of the ipsilateral trigeminal nerve
(V-1). The efferent, or motor, component is mediated by the facial nerve (CN VII),
bilaterally.
The abducens nerve (CN VI; choice A) innervates the lateral rectus muscles, which abduct
the eyes.
The optic nerve (CN II; choice C) is responsible for vision, providing the afferent limb of the
pupillary light reflex. The Edinger Westphal nucleus mediates part of this reflex.
The trigeminal nerve (CN V; choice D) is responsible for the afferent limb of the corneal
reflex. It also innervates the muscles of mastication and provides sensory innervation to
the face.
The trochlear nerve (CN IV; choice E) innervates the superior oblique muscles, which
depress, intort, and abduct the eyes.

An individual has an eye that is persistently directed toward his nose. A lesion of which of
the following nerves could produce this finding?
A. CN III
B. CN IV
C. CN V
D. CN VI
E.

CN VII

The correct answer is D. Cranial nerve VI is the abducens nerve, which supplies the
abductor of the eye, the lateral rectus. A paralysis of the lateral rectus leads to unopposed
adduction, causing the eye to point toward the nose.
Cranial nerve III (choice A) is the oculomotor nerve, which supplies all of the muscles of the
eye, except the superior oblique and lateral rectus. Paralysis of III would impair adduction,
not abduction, of the eye. The eye would tend to rotate downward and outward.
Cranial nerve IV (choice B) is the trochlear nerve, which supplies the superior oblique
muscle. This muscle serves to depress and abduct (down and out) the eyeball. Paralysis of
IV tends to produce double vision but does not cause an obvious deficit in conjugate gaze
without careful testing.
Cranial nerve V (choice C) is the trigeminal nerve, which is a mixed sensory and motor
nerve that supplies the face. It provides sensory innervation to the face and innervates the
muscles of mastication. It does not innervate the eye muscles.
Cranial nerve VII (choice E) is the facial nerve, which innervates the muscles of facial
expression but not the muscles of the orbit. CN VII is additionally involved in salivation,
lacrimation, taste, and general sensation from the external ear.

To anesthetize an area under the nailbed of the index finger, which nerve should be
injected?
A. Axillary
B. Median
C. Musculocutaneous
D. Radial
E.

Ulnar

The correct answer is B. The tumor in question is probably a benign glomus tumor, which
is notorious for producing pain far out of proportion to its small size. The question is a little
tricky because the most distal aspect of the dorsal skin of the fingers, including the nail
beds, is innervated by the palmar digital nerves rather than the dorsal digital nerves.
Specifically, the median nerve, through its palmar digital nerves, supplies the nail beds of
the thumb, index finger, middle finger, and half the ring finger.
The axillary nerve (choice A), musculocutaneous nerve (choice C), and radial nerve (choice
D) do not supply the nail beds. The radial nerve does supply the more proximal skin of the
back of the index finger.
The ulnar nerve (choice E) supplies the nail beds of the small finger and half of the ring
finger.

Examination of a karyotype taken from a metaphase preparation reveals the presence of


an extra chromosome. Which of the following is the most common mechanism of
producing this phenomenon?
A. Balanced translocation
B. Chromosomal breakage
C. Fertilization by two sperm
D. Nondisjunction
E.

Unbalanced translocation

The correct answer is D. Nondisjunction can occur in both meiosis and mitosis and refers
to a failure of paired chromosomes to separate and go to different daughter cells. When
this happens, one daughter cell gets an extra chromosome, and the other daughter cell is
"short" one chromosome.
A balanced translocation (choice A) is an exchange of genetic material between
nonhomologous chromosomes that preserves all critical genetic material.
Chromosomal breakage (choice B) produces fragmented chromosomes and can contribute
to tumorigenesis.
Fertilization by two sperm (choice C) produces triploidy and is seen in many spontaneously
aborted fetuses.
An unbalanced translocation (choice E) occurs when nonhomologous chromosomes
exchange genetic material with a net loss or gain of critical genetic material.

Damage from extension of a tumor in the body of the right lateral ventricle would most
likely affect which other brain structures?
A. Caudate nucleus
B. Cerebellum
C. Hippocampus
D. Hypothalamus
E.

Pons

The correct answer is A. Tumors of the ventricular system of the brain can affect the brain
tissue either directly, via pressure on or invasion into a physically close structure, or
indirectly, by obstructing cerebral spinal fluid (CSF) flow and causing hydrocephalus. The
caudate nucleus is a C-shaped structure that composes part of the wall of the lateral
ventricle throughout its extent. The only structure listed that is adjacent to the body of the
lateral ventricle and would therefore be directly affected by the large tumor described in
the question is the caudate nucleus. Anatomically, the caudate nucleus has a massive head
and a slender curving tail that follows the curve of the lateral ventricle. The head of the
caudate nucleus lies superior to the lentiform nucleus.
The cerebellum (choice B) overlies the fourth ventricle.
The hippocampus (choice C) is adjacent to the inferior (temporal) horn of the lateral
ventricle.
The hypothalamus (choice D) abuts the third ventricle.
The pons (choice E) forms part of the floor of the fourth ventricle.

A man is injured when a bony fragment penetrates the lateral portion of the dorsal
columns in his neck. Which of the following functions would most likely be affected by a
lesion at this site?
A. Fine motor control of the ipsilateral fingers
B. Motor control of the contralateral foot
C. Sweating of the ipsilateral face
D. Proprioception from the ipsilateral leg
E.

Vibratory sense from the ipsilateral arm

The correct answer is E. At this level, the lateral portion of the dorsal columns (funiculus) is
composed of the fasciculus cuneatus. Axons carrying tactile, proprioceptive, and vibratory
information from the ipsilateral arm enter the spinal cord via the dorsal root, ascend the
cord in the fasciculus cuneatus, and synapse in the nucleus cuneatus of the caudal medulla.
Secondary neurons from this nucleus give rise to internal arcuate fibers, which decussate
and ascend to the thalamus (ventral posterolateral nucleus, VPL) as the medial lemniscus.
Tertiary neurons from the VPL project to the ipsilateral somatosensory cortex. Because the
fibers that carry this information do not cross until they reach the medulla, damage to the
fasciculus cuneatus would result in a deficit in tactile, proprioceptive, and vibratory sense
in the ipsilateral arm.
Fine motor control of the fingers (choice A) would be carried principally by the ipsilateral
lateral corticospinal tract in the lateral funiculus of the cord.
Motor control of the contralateral foot (choice B) is carried by the ipsilateral corticospinal
tract in the lateral funiculus of the cord.
Hemianhidrosis (lack of sweating) over half of the face (choice C) could be produced by
interruption of sympathetic innervation to the face, which projects from the hypothalamus
to the intermediolateral cell column at levels T1 and T2. It descends in the lateral funiculus
of the cord. Interruption of this tract results in Horner's syndrome (miosis, ptosis, and
hemianhidrosis).
Proprioception from the ipsilateral leg (choice D) is carried by the fasciculus gracilis in the
medial part of the dorsal columns.

Which of the following areas of the heart have most likely suffered ischemic necrosis
following a myocardial infarct due to blockage of the left circumflex artery in a patient that
has left dominant coronary circulation?
A. Apex of left ventricle and anterior portion of septum
B. Lateral left ventricular wall and posterior portion of the septum
C. Lateral wall of the left ventricle only
D. Posterior portion of the septum only
E.

Right ventricular wall

The correct answer is B. A right dominant coronary circulation is present when the
posterior descending branch originates from the right coronary artery (80% of individuals).
On the contrary, the posterior descending artery originates from the left circumflex artery
in a left dominant circulation (20% of individuals). The posterior descending branch gives
blood to the posterior half of the interventricular septum. Occlusion of the left circumflex
artery in a left dominant circulation will therefore lead to ischemic necrosis in the left
ventricular wall and the posterior interventricular septum.
The apex of the left ventricle (choice A) is dependent on the anterior descending branch;
thus, occlusion of the left circumflex does not affect this portion of the left ventricle.
Infarction of the lateral (free) wall alone (choice C) will result from occlusion of the
circumflex in a right dominant circulation.
An isolated infarct of the posterior interventricular septum (choice D) arises from occlusion
of the posterior descending branch.
Isolated infarcts of the right ventricular wall (choice E) are very rare and would be caused
by occlusion of branches of the right coronary artery.

After falling on his laterally outstretched arm, a patient suffered a dislocation of the
glenohumeral joint. Which of the following nerves is most likely to have been injured from
this dislocation?
A. Axillary nerve
B. Dorsal scapular nerve
C. Lateral pectoral nerve
D. Medial pectoral nerve
E.

Suprascapular nerve

The correct answer is A. When the head of the humerus dislocates from the glenohumeral
joint, it exits inferiorly, where the joint capsule is the weakest. Immediately inferior to the
glenohumeral joint, the axillary nerve exits from the axilla by passing through the
quadrangular space. At this location, the downward movement of the head of the humerus
can stretch the axillary nerve. The axillary nerve innervates the deltoid muscle after leaving
the axilla.
The dorsal scapular nerve (choice B) passes along the medial border of the scapula to
innervate the rhomboid muscles. The nerve does not pass in the region of the
glenohumeral joint.
The lateral and medial pectoral nerves (choices C and D) branch from the lateral and
medial cords of the brachial plexus, respectively, and exit through the anterior wall of the
axilla to innervate the pectoralis major and minor. These nerves do not pass in the region
of the glenohumeral joint.
The suprascapular nerve (choice E) is a branch of the upper trunk of the brachial plexus
and passes over the superior border of the scapula to innervate the supraspinatus and
infraspinatus muscles. This nerve does not pass in the region of the glenohumeral joint.

Injury to which nerve would result in wrist drop, and inability to make a tight fist, even
though all finders can be flexed?
A. Axillary nerve
B. Long thoracic nerve
C. Median nerve
D. Musculocutaneous nerve
E.

Radial nerve

The correct answer is E. Physiologically, the distribution of the radial nerve is as follows:
extensor muscles on the arm and forearm (triceps bracii, brachioradialis, extensor carpi
radials, and extersor carpi ulnaris), ditigal extensors and abductor pollicis, and skin over the
posterolateral surface of the arm. The radial nerve lies in the musculospiral groove of the
humerus and is subject to injury in association with a fracture of the midshaft of the
humerus. The radial nerve innervates the extensor muscles of the forearm, including the
muscles that extend the wrist; paralysis of these muscles results in a wrist drop. Although
the muscles responsible for flexing the digits are not innervated by the radial nerve, the
making of a tight fist requires that the wrist be stabilized with the wrist extensors.
The axillary nerve (choice A) leaves the axilla through its posterior wall and is not located in
the region of the midshaft of the humerus. The axillary nerve innervates the deltoid and
the teres minor muscles, neither of which have any function in the hand.
The long thoracic nerve (choice B) lies against the chest wall and is not subject to injury as
a result of a humeral fracture. The long thoracic nerve innervates the serratus anterior
muscle.
The median nerve (choice C) passes though the arm, but is not in close contact with the
humerus in the midshaft region. The nerve is separated from the bone by the brachialis
muscle. The median nerve innervates many muscles of the anterior compartment of the
forearm responsible for flexion of the wrist and digits.
The musculocutaneous nerve (choice D) is not in contact with the humerus. It innervates
the muscles in the anterior compartment of the arm. These muscles have no function in
the hand.

The postganglionic signals carrying the impulses to constrict arterioles are transmitted
along which of the following fiber types?
A. A- fibers
B. B fibers
C. C fibers
D. Ia fibers
E.

Ib fibers

The correct answer is C. There are two systems currently used for classifying nerve fibers.
The first system groups both sensory and motor fibers together, describing A-, A-, A-,
A-, B, and C fibers. Another system relates only to sensory fibers, describing Ia, Ib, II, III,
and IV categories. Both classification schemes begin with large, myelinated fibers,
progressing to finer, unmyelinated fibers.
The C fiber (or IV fibers) is the only type of fiber that is unmyelinated. Remember that
preganglionic neurons are myelinated, but postganglionic neurons are unmyelinated.
Neurons that carry slow pain and temperature information are also classified as C fibers.
See the table below for more information.
A- or Ia (choice D) Alpha motor neurons, primary afferents of muscle spindles
A- or Ib (choice E) Golgi tendon organ afferents, touch and pressure
A- or II Secondary afferents of muscle spindles, touch and pressure
A- Gamma motor neurons
A- (choice A) Touch, pressure, pain and temperature (fast)
B (choice B) Preganglionic autonomic, visceral afferents
C (choice C) IV Postganglionic autonomic, pain and temperature (slow)

After an automobile accident, a patient has difficulty opening her mouth but has no
difficulty closing it. Which of the following muscles was most likely injured?
A. Lateral pterygoid
B. Masseter
C. Medial pterygoid
D. Orbicularis oris
E.

Temporalis

The correct answer is A. The lateral pterygoid muscles consist of two heads, an upper head
arising from the infratemporal surface of the greater wing of the sphenoid bone, and a
lower head arising from the lateral pterygoid plate. The lateral pterygoids act to pull the
mandible forward in the process of opening the jaw. Other actions include protracting the
mandible and moving it from side to side (as in chewing). The lateral pterygoid is
innervated by the mandibular branch of the trigerminal nerve.
The masseter (choice B) and medial pterygoid (choice C) muscles help to close the jaw,
while the temporalis (choice E) closes and retrudes the mandible. All of these muscles are
innervated by the mandibular branch of the trigeminal nerve.
The orbicularis oris (choice D) muscle purses the lips. It is innervated by the facial nerve.

The primordium of the respiratory system that forms the internal epithelial lining of the
treachea, bronchi, and lungs is derived enterily from which of the following embryologic
entities?
A. Foregut endoderm
B. Midgut endoderm
C. Splanchnic mesoderm
D. Stomodeum
E.

Both endoderm and mesoderm

The correct answer is A. The internal epithelial lining of the trachea, bronchi, and lungs
form from a diverticulum of the ventral foregut wall (endoderm) called the respiratory
diverticulum.
The midgut endoderm (choice B) gives rise to the internal epithelial lining of much of the
gastrointestinal tract, duodenum, small intestine, ascending colon, and right two thirds of
transverse colon.
The splanchnic mesoderm (choices C and E) contributes only to the smooth muscle coat
and cartilaginous skeleton of the trachea and bronchi.
The stomodeum (choice D) is of ectodermal origin and gives rise to the mouth. The
stomodeum eventually becomes continuous with the endodermal foregut of the larynx
(and trachea) and the esophagus.

A fracture of the pterygoid hamulus affects the function of which muscle?


A. Tensor veli palatini
B. Levator veli palatini
C. Medial pterygoid
D. Lateral pterygoid

The correct answer is A. This may seem like an obscure fact, but it is a perennial NBDE
favorite. The tensor veli palatini wraps around a small hook-like extension of the medial
pterygoid plate of the sphenoid bone, known as the pterygoid hamulus. The tensor (veli)
palatini originates from the greater wing of the sphenoid and from cartilage of the auditory
tube. The fibers converge into a tendon, which sharply hooks around the pterygoid
hamulus. The fibers then join with their partners from the other side to form the palatal
aponeurosis to which most of the muscles of the soft palate are attached. The muscle
tenses the soft palate, especially during swallowing.

Through which of the following structures does the greater saphenous vein pass to
eventually join the femoral vein?
A. Anatomic snuff box
B. Antecubital fossa
C. Fossa ovalis
D. Inguinal canal
E.

Popliteal fossa

The correct answer is C. The greater (long) saphenous vein starts on the dorsal surface of
the foot, wraps proximally to follow the medial aspect of the leg below and past the knee,
and then dives into the deep fascia through the fossa ovalis (also called the saphenous
opening) to reach the femoral vein, which becomes the external iliac vein shortly after it is
joined by the greater saphenous vein.
The anatomic snuff box (choice A) is on the wrist. Some authors also refer to an "anatomic
snuff box of the foot," through which the saphenous vein does pass, but this is early in its
course, and not just before it joins the femoral vein.
The antecubital fossa (choice B) is at the elbow.
The inguinal canal (choice D) contains structures going to and from the testes.
The saphenous vein passes near, but not through, the popliteal fossa (choice E) to cross
the knee.

Parathormone will have its greatest effect on which cell type?


A. Osteoclasts
B. Osteoblasts
C. Osteocytes
D. Osteosarcomas

The correct answer is A. Parathormone (PTH) is secreted by principal cells of the


parathyroid glands, small gland pairs embedded in thyroid tissue. Parathyroid hormone will
increase the level of serum calcium by increasing bone calcium resorption, increasing
digestive absorption of calcium, and decreasing kidney excretion of calcium. The bone
resorbing cells are osteoclasts, which are large, multinucleated cells. Osteoblasts are boneformers and secrete bone matrix, primarily of collagen. Osteocytes are mature bone cells
living in lacunae in compact bone. Osteosarcomas are malignant tumors of bone.

A lesion in which of the following structures could be responsible for unilateral hearing
loss?
A. Inferior colliculus
B. Lateral lemniscus
C. Medial geniculate body
D. Medial lemniscus
E.

Organ of Corti

The correct answer is E. The sequence of the auditory pathway is as follows: Organ of Corti
and ventral) superior olivary nuclei lateral lemniscus inferior colliculus medial
geniculate nucleus of the thalamus (MGN) primary auditory cortex (Heschls gyrus).
Each ear projects to both sides of the brainstem and cortex by way of multiple
commissures, including the trapezoid body (which contains fibers crossing contralateral to
the superior olivary nucleus), the commissure of the inferior colliculus (connecting the right
and left inferior colliculi), and another commissure that connects the right and the left
nuclei of the lateral lemniscus. A lesion of any structure up until the superior olivary nuclei
therefore will produce an ipsilateral deafness. The only structure listed that is proximal to
the superior olivary nuclei is the organ of Corti (choice E).
The inferior colliculus (choice A), the lateral lemniscus (choice B) and the medial geniculate
body (choice C) all receive information from both ears, and unilateral hearing loss could
not result from a lesion of any of these structures. The medial lemniscus (choice D) is not a
part of the auditory system. It is part of the somatosensory system, which conveys
proprioception, discriminative touch, and vibration information. More specifically, neurons
of the gracile and cuneate nuclei send projections that decussate as the internal arcuate
fibers and ascend as the medial lemniscus to synapse in the ventroposterolateral nucleus
(VPL) of the thalamus.

Overgrowth of the frontal bone, hands and feet has occurred, as well as tingling in the first,
second, and third digits. There is also loss of strength of the thumb. Which nerve is most
likely to be involved?
A. Anterior interosseous nerve
B. Median nerve
C. Musculocutaneous nerve
D. Radial nerve
E.

Ulnar nerve

The correct answer is B. This question is essentially asking, "Which of the following nerves
innervates the anatomical structures listed?" This patient has acromegaly, which is
characterized by overgrowth of the face, jaws, hands, and feet, enlargement of internal
organs, hyperglycemia, hypertension, and osteoporosis. It is caused by hypersecretion of
growth hormone, often attributed to an adenohypophyseal tumor. Complications include
degenerative joint disease, muscular weakness, neuropathies, and diabetes mellitus. In this
question, although the patient's sensory symptoms may be caused by a neuropathy, it is
very likely that overgrowth in the wrist area has compressed the carpal tunnel, thereby
impinging on the median nerve. Note that the median nerve (root C5-T1) provides motor
innervation to the forearm flexors, thenar muscles, and radial lumbricals. It provides
sensory innervation to the radial 2/3 of the palm, volar surfaces of the thumb, second and
third digits, and radial 1/2 of the fourth digit, as well as the skin over the lateral surface of
the hand.
Damage to the anterior interosseus nerve (choice A), also known as the deep branch of the
median nerve, results in the inability to form a round "O" with the thumb and forefinger.
This is due to impaired function of the flexor pollicis longus. Damage to the anterior
interosseus nerve could explain the patient's thumb dysfunction, but it would not account
for the patient's paresthesias in the first three digits of the hand.
The musculocutaneous nerve (choice C) innervates the arm flexors and provides sensory
information to the anterolateral forearm. It is composed of contributions from C5-C7.
The radial nerve (choice D) innervates the extensors of the arm and forearm and skin of
the posterior arm, forearm, and radial half of the dorsum of the hand (not including the
fingertips). It is composed of contributions from C6-C8.
The ulnar nerve (choice E) provides motor innervation to the ulnar flexors, adductor
pollicis, hypothenar muscles, interosseus muscles, and lumbricals 4 and 5. It provides
sensory innervation to the ulnar half of the wrist, palm, and fourth and fifth digits. It is

composed of contributions from C8-T1.

Karyotypic analysis of a spontaneously aborted fetus demonstrates trisomy of one of the


chromosomes. Which chromosome is most likely to be affected?
A. 8
B. 13
C. 16
D. 18
E.

21

The correct answer is C. Approximately half of all spontaneous abortions are of fetuses
with major chromosomal defects, most commonly trisomy 16, triploidy (due to fertilization
of an egg by two sperm), and 45 X,0 (Turner's syndrome). Trisomy 16 and triploidy do not
produce viable offspring, unlike 45 X,0.
Trisomy 8 (choice A) is one of the very rare causes of live birth trisomies.
Trisomy 13 (choice B) is one of the more common live birth trisomy syndromes (Patau
syndrome).
Trisomy 18 (choice D) is one of the more common live birth trisomy syndromes (Edwards'
syndrome)
Trisomy 21 (choice E) causes Down's syndrome.

Which of the following pharyngeal pouches develops into the palatine tonsil?
A. First
B. Second
C. Third
D. Fourth
E.

Fifth

The correct answer is B. The epithelial lining of the second pharyngeal pouch buds into the
mesenchyme to form the palatine tonsil. Part of the pouch remains in the adult as the
tonsillar fossa.
It is important to review the other choices because pharyngeal pouch derivatives are
typically tested on the NBDE part 1.
The first pharyngeal pouch (choice A) develops into the middle ear cavity and eustachian
tube.
The third pharyngeal pouch (choice C) develops into the thymus and the inferior
parathyroid glands.
The fourth pharyngeal pouch (choice D) gives rise to the superior parathyroid glands.
Recall that abnormal development of the third and fourth pouches leads to DiGeorge
syndrome and results in hypocalcemia, as well as abnormal cellular immunity and
consequent susceptibility to viral and fungal illnesses.
The fifth pharyngeal pouch (choice E) gives rise to the C cells of the thyroid gland. These
cells secrete calcitonin, a hormone that lowers serum calcium.

A nerve is located close to the wrist portion of the tendons of the flexor carpi radialis and
palmaris longus. This nerve arises from which part(s) of the brachial plexus?
A. Lateral and medial cords
B. Lateral cord
C. Middle and lower trunks
D. Posterior cord
E.

Upper and middle trunks

The correct answer is A. The nerve in question is the median nerve, which lies between the
palmaris longus and flexor carpi radialis tendons on the anterior aspect of the forearm. The
median nerve is formed from both the lateral and medial cords of the brachial plexus. The
brachial plexus innervates the shoulder girdle and upper limb with contributions from the
ventral rami of spinal nerves C5 through T1. The nerves that form this plexus originate from
trunks and cords named accordingly.

Most afferents to the cerebral cortex derive from the


A. basal ganglia
B. cerebral cortex
C. hippocampus
D. spinal cord
E.

thalamus

The correct answer is B. Most afferents to the cerebral cortex derive from the cerebral
cortex. This type of extensive interconnection of cortical areas facilitates communication
and integration between cortical areas on the same side or on different sides of the brain.
The basal ganglia (choice A) receive their most prominent input from the cortex but output
to the globus pallidus and the substantia nigra pars reticulata, rather than directly to the
cerebral cortex.
The hippocampus (choice C) sends most of its projections to the mammillary bodies and
the thalamus via the fornix, rather than to the cerebral cortex.
The ascending tracts of the spinal cord (choice D) carry sensory information to brainstem
nuclei, thalamus, and cerebellum, rather than directly to the cerebral cortex.
The thalamus (choice E) projects to the cerebral cortex, but the cortex receives
quantitatively more fibers from other areas of cortex than from the thalamus.

An angiographic study of the distal part of the circle of Willis requires access to the
vertebral artery. Access to this artery can be obtained via the
A. anterior triangle of the neck
B. muscular triangle of the neck
C. posterior triangle of the neck
D. submental triangle
E.

suboccipital triangle

The correct answer is E. The suboccipital triangle lies in the neck at the base of the skull. It
is bounded by the inferior oblique, rectus major, and superior oblique muscles. The
vertebral artery can be found within the suboccipital triangle, lying on the posterior arch of
the atlas, lateral to the midline, typically at about the level of the lower portion of the ear
lobe. Care should be taken in accessing the vessel because the first cervical nerve lies
between the vertebral artery and the posterior arch of the atlas. The circle of Willis
contains the following arteries: anterior communicating, anterior cerebral, posterior
communicating, and posterior cerebral.
The anterior triangle of the neck (choice A) is in the anterior neck and is bounded by the
neck midline, the mandible, and the sternocleidomastoid muscle. The anterior triangle of
the neck is subdivided into the muscular, carotid, submandibular, and submental triangles.
The small muscular triangle (choice B) lies between the superior belly of the omohyoid and
the sternohyoid muscle.
The posterior triangle of the neck (choice C) is on the side of the neck and is bounded by
the sternocleidomastoid muscle, the trapezius, and the middle third of the clavicle.
The submental triangle (choice D), also called the suprahyoid triangle, is found below the
lower jaw and is bounded by the hyoid bone and anterior bellies of the digastric muscles.

Tissue served by branches of which of the following arteries would be most likely affected
by pressure on the splenic artery?
A. Left gastric
B. Left gastroepiploic
C. Right gastric
D. Right gastroepiploic
E.

Short gastric

The correct answer is E. The splenic artery passes behind the stomach and gives off the
short gastric artery and the left gastroepiploic artery immediately after passing the greater
curvature. The left gastroepiploic artery has a strong anastomotic connection to another
arterial supply, while the short gastric does not, so the area supplied by branches of the
short gastric arteries is more vulnerable to ischemia in this setting. If the block had
occurred proximal to, instead of at, the branch point, the short gastric vessels could be
supplied by backflow from the left gastroepiploic artery.
The left gastric artery (choice A) is not supplied by the splenic artery.
The left gastroepiploic artery (choice B) can be alternatively supplied by its anastomotic
connection to the right gastroepiploic artery.
The right gastric artery (choice C) is not supplied by the splenic artery.
The right gastroepiploic artery (choice D) is normally supplied by the gastroduodenal
artery.

Before removing a spleen, the splenic artery and splenic vein are ligated. Within which of
the following peritoneal structures are the splenic artery and vein found?
A. Gastrocolic ligament
B. Gastrosplenic ligament
C. Lesser omentum
D. Splenorenal ligament

The correct answer is D. The splenorenal ligament is the portion of the dorsal mesentery
between the posterior abdominal wall and the spleen. This mesentery transmits the
splenic artery and vein from their retroperitoneal position in the proximal portion of their
course to the peritoneal spleen.
The gastrocolic ligament (choice A) is the portion of the greater omentum between the
greater curvature of the stomach and the transverse colon. This portion of mesentery is
not related to the spleen.
The gastrosplenic ligament (choice B) is the portion of the dorsal mesogastrium between
the greater curvature of the stomach and the spleen. There are no splenic vessels in this
mesentery.
The lesser omentum (choice C) is derived from the ventral mesentery. It is the mesentery
between the lesser curvature of the stomach and the liver and between the first portion of
the duodenum and the liver. It is not related to the spleen.

A person lifts one foot prior to taking a step. Which of the following nerves innervates the
muscle group that allows the person to maintain balance by holding the weight of his body
over the foot remaining on the ground?
A. Femoral nerve
B. First and second sacral nerves
C. Obturator nerve
D. Superior gluteal nerve
E.

Tibial nerve

The correct answer is D. The muscles in question are the gluteus medius and gluteus
minimus, which are the innervated by the superior gluteal nerve. Textbooks often describe
these muscles as abductors of the hip, but in real life, they usually function as described in
the question.
The femoral nerve (choice A) innervates muscles of the anterior thigh, allowing extension
of the leg at the knee.
The first and second sacral nerves (choice B) innervate the piriformis muscle, which is a
lateral rotator of the thigh at the hip.
The obturator nerve (choice C) innervates muscles of the medial thigh.
The tibial nerve (choice E) innervates muscles of the posterior lower leg.

Which pair of muscles is innervated by Cranial Nerve V?


A. Masseter, buccinator
B. Mylohyoid, geniohyoid
C. Lateral pterygoid, medial pterygoid
D. Anterior digastric, posterior digastric
E.

Levator palatini, tensor palatini

The correct answer is choice CIn brief, Cranial Nerve V, the trigeminal, innervates the
muscles of mastication, the two tensors, the anterior digastric and the mylohyoid. The
muscles of mastication are the medial and lateral pterygoid, the temporalis, and the
masseter. The two tensors are the tensor tympani and the tensor (veli) palatini. Note that
among the incorrect answers, buccinator, as a muscle of facial expression, is innervated by
CN VII, the facial nerve, geniohyoid is innervated by CN XII, the hypoglossal, the posterior
digastric is innervated by CN VII, the facial, and levator palatini is innervated by the cervical
plexus.

The endothelial lining of the aorta is composed of which of the following tissue types?
A. Pseudostratified epithelium
B. Simple columnar epithelium
C. Simple cuboidal epithelium
D. Simple squamous epithelium
E.

Stratified columnar epithelium

The correct answer is D. Endothelium lines the cardiovascular and lymphatic vessels and is
composed of simple squamous epithelium. The mesothelium that lines the pleural,
pericardial, and peritoneal cavities is also composed of a single-layer of simple squamous
epithelium.
Pseudostratified epithelium (choice A) is found in the epithelial lining of the respiratory
tract, as well as in the transitional epithelium of the urinary system.
In simple columnar epithelium (choice B), the cells are taller than they are wide. This
epithelium can be found in the intestinal absorptive surface.
Simple cuboidal epithelium (choice C) can be found in the ducts of many glands.
Stratified columnar epithelium (choice E) can be found in portions of the male urethra.

A 5-year-old child who has not had routine pediatric care develops a febrile disease with
cough and a blotchy rash, and is brought to the emergency department. On physical
examination, there is cervical and axillary lymphadenopathy. Also noted is an
erythematous, macropapular rash behind the ears and along the hairline, involving the
neck and, to a lesser extent, the trunk. Examination of this patient's oropharynx would
likely reveal which of the following lesions?
A. Adherent thin, whitish patch on gingiva
B. Cold sores on the lips
C. Curdy white material overlying an erythematous base on the oral mucosa
D. Large shallow ulcers on the oral mucosa
E.

Multiple small white spots on the buccal mucosa

The correct answer is E. The disease described is measles (rubeola), which has the typical
presentation described in the question stem. Measles is caused by a Morbillivirus, an RNA
virus belonging to the Paramyxovirus family. Koplik spots, which are pathognomonic for
measles, are small, bluish-white spots on the buccal mucosa in the early stages of measles.
These lesions appear just before the onset of the characteristic rash (which can also involve
extremities) and fade as the rash develops.
Leukoplakia is a premalignant condition characterized by adherent whitish patches on the
gingiva (choice A) and other sites in the oral cavity.
Cold sores on the lips (choice B) are caused by infection with Herpes virus.
Candida infection (thrush) produces curdy white material loosely attached to an
erythematous base (choice C).
Aphthous ulcers are large shallow ulcers of the oral mucosa (choice D), commonly known
as canker sores.

When removing an impacted mandibular third molar, the oral surgeon must warn the
patient of possible lasting numbness of the tip of the tongue. This loss of general sensation
is due to damage to the
A. auriculotemporal nerve
B. chorda tympani
C. lingual nerve
D. mental nerve
E.

mylohyoid nerve

The correct answer is C. The lingual nerve is a branch of the mandibular division of the
trigeminal nerve that conveys general sensation from the anterior two thirds of the
tongue. It enters the oral cavity by passing just under the mandibular third molar between
the medial pterygoid muscle and the mandibular ramus.
The auriculotemporal nerve (choice A) is a branch of V3 that passes from the infratemporal
fossa to the parotid region. It contains sensory fibers from the region in front of the ear
and the temporomandibular joint and also conveys postganglionic parasympathetic fibers
to the parotid salivary gland.
The chorda tympani (choice B) is a branch of CN VII that travels with the lingual nerve in
the floor of the mouth. It carries taste fibers from the anterior two thirds of the tongue and
preganglionic parasympathetic fibers that synapse in the submandibular ganglion.
The mental nerve (choice D) is a sensory branch of the inferior alveolar nerve that supplies
the skin of the chin and lower lip.
The mylohyoid nerve (choice E) is a motor branch of the inferior alveolar nerve that
supplies the mylohyoid and anterior belly of the digastric muscles.

Stimulation and pressure on the ear canal can sometimees result in fainting. This is
because the posterior half of the external ear canal receives innervation from the
A. auricular branch of the vagus nerve
B. auriculotemporal nerve
C. greater auricular nerve
D. lesser occipital nerve
E.

vestibulocochlear nerve

The correct answer is A. The vagus nerve, through its auricular branch, supplies the
posterior half of the external auditory meatus. Because the vagus also supplies many other
organs, reflex symptoms may occur, including fainting (typically in the elderly), coughing,
and gagging. The vagus nerve is sensory from the pharynx, pinna and external auditory
canal, diaphragm, and visceral organs in the thoracic and abdominopelvic cavities. It is a
motor nerve to the palatal and pharyngeal muscles and visceral organs in thoracic and
abdominal cavities.
The auriculotemporal nerve (choice B) supplies the anterior half of the external auditory
meatus and the facial surface of the upper part of the auricle.
The greater auricular nerve (choice C) supplies both surfaces of the lower part of the
auricle.
The lesser occipital nerve (choice D) supplies the cranial surface of the upper part of the
auricle.
The vestibulocochlear nerve (choice E) supplies hearing and motion sense.

If a clavicle fractures, which muscle might prevent the fractured bone from damaging the
subclavian vessels or large nerves of the arm?
A. Deltoid
B. Pectoralis major
C. Sternocleidomastoid
D. Subclavius
E.

Trapezius

The correct answer is D. The clavicle is the most frequently broken long bone of the body.
However, despite what one might expect, the underlying subclavian vessels and parts of
the brachial plexus are only rarely injured. This is because the subclavius muscle, which
occupies a small groove on the undersurface of the clavicle, apparently protects the
underlying structures. Other functions of this small muscle are not well understood.
The deltoid (choice A) originates from the lateral aspect of the clavicle and is a powerful
abductor of the arm.
The pectoralis major (choice B) originates from the medial aspect of the clavicle and is not
as important as the subclavius with regard to protecting underlying structures.
The sternocleidomastoid (choice C), an important muscle of the anterior neck, attaches to
the medial aspect of the posterosuperior border of the clavicle.
The trapezius (choice E), an important muscle of the posterior neck, attaches to the lateral
aspect of the posterosuperior border of the clavicle.

An esophageal biopsy shows normal smooth muscle and striated muscle in the same
section. Which portion of the esophagus was the source of this biopsy?
A. Lower esophageal sphincter
B. Lower third of the esophagus
C. Middle third of the esophagus
D. Upper esophageal sphincter
E.

Upper third of the esophagus

The correct answer is C. The middle third of the esophagus contains both striated and
smooth muscle.
The lower third (choice B) of the esophagus, including the lower esophageal sphincter
(choice A) contains only smooth muscle.
Some authors identify the cricopharyngeus muscle as an upper esophageal sphincter
(choice D), despite the fact that unlike a true sphincter, this muscle does not completely
encircle the esophagus. The cricopharyngeus is composed exclusively of skeletal muscle;
therefore, the biopsy could not have been from this area of the esophagus.
The muscularis of the upper third of the esophagus (choice E) is composed entirely of
striated muscle.

Which of the following structures constitutes part of the ventricles of the heart?
A. Auricle
B. Crista terminalis
C. Fossa ovalis
D. Sinus venarum
E.

Trabeculae carneae

The correct answer is E. The trabeculae carneae are ridges of myocardium in the
ventricular wall. Remember that the Latin root "carne-" means "flesh" and that the
ventricles are "fleshier" than the atria.
The auricle (choice A) is derived from the fetal atrium; it has rough myocardium known as
musculi pectinati, or pectinate muscle.
The crista terminalis (choice B) is the vertical ridge that separates the smooth portion of
the right atrium from the rough portion; it extends longitudinally from the superior vena
cava to the inferior vena cava.
The fossa ovalis (choice C) is the remnant of the fetal foramen ovale, an opening in the
interatrial septum that allows blood entering the right atrium from the inferior vena cava
to pass directly to the left side of the heart.
The sinus venarum (choice D) is the smooth-walled portion of the atrium that receives
blood from the superior and inferior vena-cava. It is derived from the fetal sinus venosus.

If a patient is unable to touch the tip of his tongue to the roof of his mouth, dysfunction of
which of the following muscles is most likely the cause?
A. Buccinator
B. Geniohyoid
C. Palatoglossus
D. Palatopharyngeus
E.

Tensor palati

The correct answer is C. Elevation of the tongue is carried out by the styloglossus
(innervated by the hypoglossal nerve, CN XII) and the palatoglossus (innervated by the
pharyngeal plexus). The palatoglossus originates on the anterior surface of the soft palate
and inserts in the side of the tongue.
The buccinator (choice A) functions in storing, filling, and emptying the vestibule.
The geniohyoid (choice B) moves the hyoid anteriorly to open the pharynx.
The palatopharyngeus (choice D) produces a "stripping wave" on the posterior pharyngeal
wall.
The tensor palati (choice E) tenses the soft palate.

Another term used to describe compact bone is:


A. Spongy bone
B. Intramembranous bone
C. Bundle bone
D. Woven bone
E.

Lamellar bone

The correct answer is E. There are many overlapping and sometimes confusing terms for
bone types. Compact bone is dense and contains Haversian systems. It is also known as
lamellar (layered) bone. Spongy bone is also known as trabecular bone and does not
contain Haversian systems. Compact bone is normally found on the outside surface of
bones, whereas spongy bone is found on the inside. Bundle bone refers to compact bone
containing collagen attachment fibers, such as those found in the alveolar bone of the
periodontal ligament. Intramembranous bone refers to one type of bone formation, bone
formed from a primitive connective tissue model (not collagen). Woven bone is early
intramembranous bone. The other type of bone is formed from ossification of a
cartilaginous model.

Which of the following muscles allow a patient to continue to flex the elbow after rupture
of the biceps tendon?
A. Brachialis and brachioradialis
B. Flexor carpi ulnaris and flexor carpi radialis
C. Flexor digitorum superficialis and flexor digitorum profundus
D. Pronator teres and supinator
E.

Triceps and coracobrachialis

The correct answer is A. The long head of the biceps tendon is vulnerable to abrasion as it
moves within the bicipital grove between the greater and lesser tuberosities of the
humerus. The tendon occasionally snaps, particularly in the elderly. Fortunately, the
brachialis and brachioradialis are sufficiently strong flexors of the elbow in which function
is retained even if no specific repair of the tendon is undertaken.
The flexor carpi ulnaris and the flexor carpi radialis (choice B) are flexors at the wrist, not
elbow.
The flexor digitorum superficialis and the flexor digitorum profundus (choice C) are flexors
of the wrist and fingers.
The pronator teres and the supinator (choice D) pronate and supinate the forearm,
respectively.
The triceps and the coracobrachialis (choice E) extend the elbow and flex the shoulder,
respectively.

Which of the following nuclei is the most important source of noradrenergic innervation to
the cerebral cortex?
A. Basal nucleus of Meynert
B. Caudate nucleus
C. Locus coeruleus
D. Raphe nucleus
E.

Substantia nigra

The correct answer is C. The locus coeruleus is a dense collection of neuromelanincontaining cells in the rostral pons, near the lateral edge of the floor of the fourth ventricle.
The fact that it appears blue-black in unstained brain tissue gave rise to its name, which
means "blue spot" in Latin. These cells, which contain norepinephrine, provide the majority
of noradrenergic innervation to the forebrain, including the cerebral cortex.
The basal nucleus of Meynert (choice A), a part of the substantia innominata, is a major
collection of forebrain cholinergic neurons. These neurons (together with neurons in septal
nuclei) innervate the neocortex, hippocampal formation, and the amygdala. The basal
nucleus is one of the structures that degenerates in Alzheimer's disease.
The caudate nucleus (choice B) is part of the basal ganglia, located immediately lateral to
the lateral ventricles. There are at least two important cell types in the caudate. GABAergic
projection neurons (the majority) innervate the globus pallidus and substantia nigra pars
reticulata. The GABAergic neurons degenerate in Huntington's disease, leading to enlarged
lateral ventricles that are clearly visible on MRI. The caudate also contains cholinergic
interneurons, which provide most of the acetylcholine to the striatum (caudate and
putamen). The balance of striatal acetylcholine and dopamine is important for the
treatment of patients with extrapyramidal symptoms, sucha as Parkinson's disease or
parkinsonism accompanying therapy with antipsychotic medications.
The raphe nuclei (choice D) are located in the midline at most levels of the brainstem. They
contain seratonergic cell bodies that innervate virtually every part of the central nervous
system.
The substantia nigra (choice E) is located in the midbrain and consists of the substantia
nigra pars compacta and the substantia nigra pars reticulata. The substantia nigra pars
compacta contains the nigrostriatal neurons that are the source of striatal dopamine. This
cell degenerates in Parkinson's disease or in response to neurotoxic agents such as MPTP.
The substantia nigra pars reticulata consists predominately of GABAergic neurons that

innervate the thalamus.

Brunner's glands secrete an alkaline product that helps achieve optimal pH for the activity
of pancreatic enzymes. Where are these glands located?
A. At the base of villi throughout the small intestine
B. In the epithelium of the ampulla of Vater
C. In the mucosa and submucosa of the jejunum
D. In the submucosa of the duodenum
E.

In the submucosa of the ileum

The correct answer is D. Brunner's glands are located in the submucosa of the duodenum.
These glands are connected to the intestinal lumen by ducts that open into certain crypts.
They secrete an alkaline product that protects the duodenal mucosa from the acidic chyme
and helps achieve optimal pH for pancreatic enzymes.
Note that if you did not recall the location of Brunner's glands, the question's description
of their function allowed you to deduce it on the basis of your knowledge of the anatomy
of the small intestine. You should have immediately ruled out choices C and E because they
are too far from the pancreas. Choices B and D would therefore remain as the best
possible answers because of their proximity to the pancreas. If you remembered the
structure and function of the ampulla of Vater, you were left with the correct answer.
The small intestinal villi (choice A) are outgrowths of the mucosa into the lumen. Their
epithelium contains columnar absorptive cells and goblet cells (which produce acid
glycoproteins that protect and lubricate the lining of the intestine). Near the base of each
villus are tubular glands called crypts, whose lining is continuous with the simple columnar
epithelium of the villus. The crypts include Paneth cells, which produce acidophilic
cytoplasmic granules containing bactericidal enzymes.
The ampulla of Vater (choice B) receives bile from the common bile duct and the main
pancreatic duct, delivering it to the duodenum through the major duodenal papilla.
The mucosa and submucosa of the jejunem (choice C) are both included in the permanent
folds called the plica circulares.
The submucosa of the ileum (choice E) is the home of Peyer's patches, which are large
aggregates of lymphoid nodules.

Which structure passes near the uterine artery, and might be confused with it during
surgery?
A. Internal iliac artery
B. Internal iliac vein
C. Ovarian artery
D. Ureter
E.

Uterine vein

The correct answer is D. The ureter passes directly inferior to the uterine artery, lateral to
the body of the uterus near its junction with the cervix ("water flows under the bridge").
During a hysterectomy, therefore, the ureter (instead of the uterine artery) may be
inadvertently ligated.
The internal iliac artery (choice A) gives rise to the uterine artery, the primary blood supply
of the uterus.
The internal iliac vein (choice B) receives blood from the uterine vein (choice E), the
primary venous drainage of the uterus.
The ovarian artery (choice C) arises from the abdominal aorta and is the primary blood
supply of the ovaries.

Which of the following renal structures is most medially located?


A. Major calyx
B. Minor calyx
C. Renal cortex
D. Renal pelvis
E.

Renal pyramid

The correct answer is D. This is a relatively simple question that requires you to visualize
the relationship among the key parts of the kidney and to identify the one that lies most
medially. Because the kidneys ultimately drain into the ureter at their medial poles, you
are looking for the structure that is closest to the ureter. The correct answer is the renal
pelvis. The renal pelvis is the dilated upper portion of the ureter that receives the major
calyces.
In terms of the other answer choices, the order from most lateral to most medial is as
follows: renal cortex (choice C), renal pyramid (choice E), minor calyx (choice B), major
calyx (choice A), and then renal pelvis (choice D).

Weakness when a patient attempts internal rotation of the right arm at the shoulder could
be caused by weakness in which of the following muscles?
A. Infraspinatus
B. Pectoralis minor
C. Subscapularis
D. Supraspinatus
E.

Teres minor

The correct answer is C. The subscapularis muscle arises from the anterior surface of the
scapula and inserts onto the lesser tubercle of the humerus. It is one of the rotator cuff
muscles. Its tendon passes on the anterior side of the shoulder joint capsule, where it
reinforces the capsule. Contraction of this muscle causes internal rotation of the arm at the
shoulder.
The infraspinatus muscle (choice A) arises from the posterior surface of the scapula in the
infraspinous fossa and inserts on the greater tubercle of the humerus. It is one of the
rotator cuff muscles. Its tendon passes along the posterior surface of the shoulder joint
capsule, where it reinforces the capsule. Contraction of the infraspinatus causes external
rotation of the arm at the shoulder.
The pectoralis minor muscle (choice B) arises from the chest wall and inserts onto the
coracoid process of the scapula. The pectoralis minor does not attach to the humerus and
therefore does not cause movement of the humerus at the shoulder.
The supraspinatus muscle (choice D) arises from the posterior surface of the scapula in the
supraspinous fossa and inserts onto the greater tubercle. It is one of the rotator cuff
muscles. Its tendon passes along the superior surface of the shoulder joint capsule, where
it reinforces the capsule. Contraction of the supraspinatus causes abduction of the arm at
the shoulder.
The teres minor muscle (choice E) arises from the axillary border of the scapula and inserts
onto the greater tubercle of the humerus. It is one of the rotator cuff muscles. Its tendon
passes along the posterior surface of the shoulder joint capsule, where it reinforces the
capsule. Contraction of the teres minor causes external rotation of the arm at the
shoulder.

Which of the following hormones is secreted by anterior pituitary cells that stain with
acidic dyes?
A. ACTH
B. FSH
C. LH
D. Prolactin
E.

TSH

The correct answer is D. The cells of the anterior pituitary can be classified as chromophils
(stain with dyes) or chromophobes (do not stain with dyes). The chromophils can be
further divided in acidophils (stain with acidic dyes) and basophils (stain with basic dyes).
The acidophils include the somatotrophs, which secrete growth hormone, and the
mammotrophs, which secrete prolactin.
The basophils include the corticotrophs, which secrete ACTH (choice A), the gonadotrophs,
which secrete FSH and LH (choices B and C), and the thyrotrophs, which secrete TSH
(choice E).

Incomplete fusion of the embryonic endocardial cushions can produce which of the
following congenital defects?
A. Atrioventricular septal defect
B. Coarctation of the aorta
C. Pulmonary stenosis
D. Tetralogy of Fallot
E.

Transposition of the great vessels

The correct answer is A. The endocardial cushions form the fibrous tissue "skeleton" of the
heart, which lies between the chambers of the heart. A failure of fusion of the cushions can
cause an atrioventricular defect, which, in extreme cases, can make the heart behave as if
it were a single chamber. Surgical correction of a large defect may be difficult and may not
yield an effective heart because other malformations, such as small ventricular size, may
coexist.
The causes of coarctation of the aorta (choice B) are unclear but are not thought to be
related to incomplete fusion of the endocardial cushions.
Pulmonary stenosis (choice C) can be caused by fusion of the cusps of the semilunar valve
or by unequal division of the truncus arteriosus.
Tetralogy of Fallot (choice D) is caused by failure of the pulmonary trunk and aorta to line
up with the ventricular openings.
Transposition of the great vessels (choice E) results from abnormal migration of neural
crest cells, leading to failure of spiral development of the aorticopulmonary septum.

Injury to the lower division of the facial nerve during parotid surgery will result in
A. inability to furrow the brow (to frown) on the same side
B. numbness over the angle and mental region of the jaw on the same side
C. ptosis of the eye on the same side
D. weakness in closing the eye on the same side
E.

weakness of the lower lip on the same side

The correct answer is E. The motor component (special visceral efferent) of the facial
nerve (CN VII) exits the skull via the stylomastoid foramen, passes lateral to the styloid
process, and then enters the parotid gland. Within the gland, two divisions can usually be
identified (upper and lower), which in turn give off five named branches that innervate the
muscles of the face. The upper division gives rise to the temporal and zygomatic branches,
which collectively innervate the frontalis, corrugator, and orbicularis oculi muscles. The
lower division gives off the buccal, mandibular, and cervical branches. The largest, the
buccal, innervates the muscles attaching to the upper lip, including the orbicularis oris and
the levators, as well as the buccinator and the muscles of the nose. The mandibular
branches innervate the muscles of the lower lip and of the chin, whereas the cervical
branch innervates the platysma muscle. There are usually communicating branches
between the named terminal nerves so that overlapping innervation of the muscles occurs.
If the lower division is injured, there will be weakness (not frank paralysis because of the
innervation overlap) of the muscles that attach to the lower lip.
An inability to furrow the brow (choice A) would be caused by denervation of the
corrugator supercilii and frontalis muscles, which are innervated by the upper division of
the facial nerve.
Choice B is wrong because once the facial nerve emerges from the stylomastoid foramen,
it is a pure motor nerve (special visceral efferent, or branchiomotor nerve). It carries no
sensory nerve fibers.
Ptosis (a drooping of the upper eyelid; choice C) is the result of a paralysis of the levator
palpebrae muscle, which is innervated by the oculomotor (CN III) nerve.
Choice D is not correct because the orbicularis oculi muscle is innervated by branches from
the upper division of the facial nerve.

The notochord forms on approximately what day after conception?


A. Day 2
B. Day 7
C. Day 17
D. Day 28
E.

Day 60

The correct answer is C. Formation of the notochord is a third week event, occurring as a
relatively early part of development of the trilaminar disc. More specifically, notochord
formation begins about day 16, and the neuroenteric canal forms on about day 18.
Two days after conception (choice A), implantation has not yet occurred.
By seven days after conception (choice B), the blastocyst has usually implanted in the
endometrium.
By day 28 (choice D), gastrulation has occurred, and organogenesis is under way.
By day 60 (choice E), organogenesis is complete.

The layers of the abdominal wall are shown below.


1. Internal oblique 2. External oblique 3. Peritoneum 4. Transversus abdominis
Which of the following corresponds to the order of the layers of the abdominal wall from
superficial to deep?
A. 1-3-4-2
B. 2-1-3-4
C. 2-1-4-3
D. 4-1-2-3
E.

4-2-1-3

The correct answer is C. Questions like these are particularly amenable to the use of testtaking strategies because even if you don't know 100% of the answer, you will be able to
eliminate some answer choices on the basis of knowledge you do have. For example, you
probably are aware that the peritoneum does not lie above the transversus abdominis
muscle, enabling you to eliminate choice A and increase your chances of answering
correctly by 20%. Likewise, the external oblique must be external to the internal oblique,
which eliminates choices A and D.

A patient develops a form of lung cancer that spreads to occlude the thoracic duct. Edema
involving which of the following sites might be expected as a potential complication?
A. Entire left side and right leg
B. Entire right side and left leg
C. Left arm only
D. Right arm and right half of head only
E.

Right arm only

The correct answer is A.. The right lymphatic duct drains the right arm, the right side of the
chest, and the right side of the head. The thoracic duct drains the rest of the body. Both
the right lymphatic duct and the thoracic duct dump into the large venous channels at the
base of the neck at the junction of the subclavian vein and internal jugular vein. Occlusion
of this drainage can produce intractable edema in sites feeding these ducts.
The left side and right leg would be affected, rather than the right side and left leg (choice
B).
The entire left side (rather than just the left arm, choice C) and right leg drain into the
thoracic duct.
The right arm and the right half of the head (choices D and E) drain to the right lymphatic
duct.

The first bony structure that can be palpated below the inferior margin of the medial
portion of the clavicle is the
A. acromion
B. atlas
C. first rib
D. manubrium
E.

second rib

The correct answer is E. The palpable space immediately inferior to the clavicle is the first
intercostal space, and the bone below it is the second rib.
The acromion (choice A) is the lateral extension of the scapular spine.
The atlas (choice B) is the first cervical vertebra, articulating with the occipital bone above
and the axis below.
The first rib (choice C) is hidden under the clavicle.
The manubrium (choice D) is the most superior portion of the sternum.

Following thyroidectomy, hoarseness of the voice may occur. This condition is caused by
damage to the
A. internal laryngeal nerve
B. recurrent laryngeal nerve
C. thyroarytenoid muscle
D. vestibular folds
E.

vocal folds

The correct answer is B. The recurrent laryngeal nerves are branches of the vagus (CN X)
and supply all intrinsic muscles of the larynx, except the cricothyroid. The right recurrent
laryngeal nerve recurs around the right subclavian artery. The left recurrent laryngeal
nerve recurs in the thorax around the arch of the aorta and ligamentum arteriosum. Both
nerves ascend to the larynx by passing between the trachea and esophagus, close to the
thyroid gland.
The internal laryngeal (choice A) nerve is a purely sensory branch of the superior laryngeal
nerve.
The thyroarytenoid (choice C) is an intrinsic muscle of the larynx; its inner fibers are
specialized as the vocalis muscle, which is related to the vocal ligament. It is not usually at
risk during thyroid surgery.
The vestibular folds (choice D), or false vocal folds, are located superior to the true vocal
folds inside the larynx. They are not concerned with phonation.
The vocal folds (choice E) form the boundaries of the rima glottidis inside the larynx and
are not vulnerable during thyroidectomy.

The interlobular arteries of the kidney result from subdivision of the


A. efferent arterioles
B. arcuate arteries
C. interlobar arteries
D. renal artery

The correct answer is B. The renal artery (choice D) enters the kidney at the hilum and
immediately divides into the interlobar arteries (choice C). These arteries supply the renal
pelvis and capsule before passing between the medullary pyramids to the corticomedullary
junction. They then bend almost 90 degrees to form the short, arching arcuate arteries
(choice B), which run along the corticomedullary junction.
The arcuate arteries subdivide into the fine interlobular arteries. These interlobular
arteries give off branches that become the afferent arterioles (choice A) of the glomerulus.

In the adult, neurons are an example of which of the following?


A. Continuously dividing cells
B. Labile cells
C. Permanent cells
D. Quiescent cells
E.

Stable cells

The correct answer is C. Permanent (non-dividing) cells have left the cell cycle and can no
longer undergo mitotic division. Examples that are usually cited include neurons and
cardiac myocytes. One of the major disadvantages of these cells is that once they are
damaged or destroyed, they do not regenerate.
Examples of labile (continuously dividing) cells (choices A and B) include the epithelia of
the body surface, gastrointestinal tract, and urinary tract.
Examples of stable (quiescent) cells (choices D and E) include the cells of the liver and the
glandular organs of the body.

If vertical dimension of occlusion is VDO, and vertical dimension of rest is VDR, and
freeway space is FS, then:
A. VDO + VDR = FS
B. VDO + FS = VDR
C. VDR + FS = VDO
D. None of the above

The correct answer is B. The vertical dimension of occlusion (VDO) is a vertical


measurement on the front of the face when the teeth are in full occlusion (centric
occlusion). When the face is at rest, the teeth are slightly apart, and the vertical dimension
of the front of the face is slightly longer. This is the vertical dimension of rest (VDR). The
distance between the teeth at this point is the freeway space (FS). So when we take the
smaller VDO and add the few millimeters of the FS, we get the slightly longer VDR. VDR is
generally 2 to 5 mm more than VDO.

A nerve running around the neck of the humerus is damaged. The nerve damage causes
muscle denervation, and the inability to raise the arm over the head. The denervated
muscle arises from which of the following?
A. Acromion alone
B. Clavicle and acromion
C. Clavicle and coracoid
D. Coracoid alone
E.

Coracoid and acromion

The correct answer is B. The nerve is the axillary branch of the posterior cord (also called
the axillary nerve), which can be damaged by either fracture or dislocations of the humeral
neck. The muscle supplied by this nerve is the deltoid muscle. The deltoid arises from the
lateral portions of the clavicle and the acromion and inserts into the deltoid tuberosity of
the humerus.
The deltoid arises from both the acromion (choice A) and the clavicle.
The deltoid does not arise from the coracoid process (choices C, D, and E).

Poor blood supply to which of the following tissues is most often problematic during the
healing process after surgery?
A. Adipose tissue
B. Skin
C. Loose connective tissue
D. Muscle

The correct answer is A. Adipose tissue is loose connective tissue dominated by


adipocytes. Surgeons worry about their obese patients more than their skinny ones
because a thick layer of relatively poorly vascularized, subcutaneous fatty tissue is both
mechanically unstable (it holds stitches poorly) and heals very slowly. These patients have
a frequent rate of dehiscence (tearing open of the incision site) with subsequent, difficultto-control infection (access by antibiotics, leukocytes, and serum antibodies are all
hampered by the poor blood supply).
Loose connective tissue (choice C) is well vascularized and surgeons do not usually worry
much about it during the healing process.
Muscle (choice D) usually heals well after surgery.
Skin (choice E) usually heals well, unless it becomes infected.

A patient is able to say "Kuh, kuh, kuh," and "Mi-mi-mi." The patient is unable to say "La-lala." A lesion of which of the following cranial nerves should be suspected?
A. VII
B. VIII
C. X
D. XI
E.

XII

The correct answer is E. Saying "la" requires the tongue to be placed against the roof of
the mouth, which cannot be accomplished if the hypoglossal nerve (CN XII) is
nonfunctional. CN XII innervates the intrinsic muscles of the tongue. The hypoglossal nerve
innervates the genioglossus, hypoglossus, palatoglossus, and styloglossus muscles of the
tongue.
The facial nerve, CN VII (choice A), is needed to move the lips while saying "mi." CN VII
innervates the muscles of facial expression.
The vestibulocochlear nerve, CN VIII (choice B), mediates hearing and maintains
equilibrium and balance.
The vagus nerve, CN X (choice C), is needed to raise the palate while saying "kuh."
The spinal accessory nerve, CN XI (choice D), mediates head and shoulder movements.

Mandibulofacial dysostosis is due to abnormal development of which of the following


structures?
A. First pharyngeal arch
B. First pharyngeal cleft
C. Second pharyngeal arch
D. Second pharyngeal cleft
E.

Third pharyngeal cleft

The correct answer is A. The pharyngeal arches are outpouchings of tissue visible on the
external neck of the embryo. They are separated by the pharyngeal clefts (each one caudal
to its arch). The pharyngeal pouches are the outpouchings of the pharynx visible inside the
embryo that correspond to the arches. The defect described is due to abnormal
development of the derivatives of the first pharyngeal arch. It is thought that the initial
defect may be insufficient migration of neural crest cells.

Several arterial branches penetrate into the distal end of the lesser curvature of the stomach.
Which of the following arteries usually supplies these branches?
A. Left gastric
B. Left gastroepiploic
C. Right gastric
D. Right gastroepiploic
E.

Short gastric

The correct answer is C. The arterial supply of the stomach is complex; it therefore has a
good potential to appear on the NBDE. The right gastric artery supplies the distal lesser
curvature.
The left gastric artery (choice A) supplies the proximal lesser curvature.
The left gastroepiploic artery (choice B) supplies the proximal greater curvature below the
splenic artery.
The right gastroepiploic artery (choice D) supplies the distal greater curvature.
The short gastric artery (choice E) supplies the proximal greater curvature above the splenic
artery.

Which of the following is secreted by the neurohypophysis?


A. FSH
B. TSH
C. ADH
D. LH
E.

STH

The correct answer is C.The first part of answering the question is to recognize the
terminology. In particular, the NBDE often uses the term neurohypophysis for posterior
pituitary, and adenohypophysis for anterior pituitary. The only two hormones of the
posterior pituitary (neurohypophysis) are ADH (vasopressin), which increases water
reabsorption by the kidney, and oxytocin, which stimulates uterine contractions during labor
and delivery. All other pituitary hormones are anterior, form the adenohypophysis. They
include FSH (follicle stimulating hormone), which acts on the ovary, TSH (Thyroid stimulating
hormone), which acts on the thyroid, LH (luteinizing hormone), which acts on the ovary, and
STH (GH, growth hormone) which stimulates long bone growth, and especially acts on
epiphyseal plates.

If a lumbar puncture needle passes through the interlaminar space between the fourth and
fifth lumbar vertebrae, which space does it enter?
A. Epidural
B. Intramuscular
C. Subarachnoid
D. Subcutaneous
E.

Subdural

The correct answer is A. The interlaminar space is the space between the laminae of the
lumbar vertebrae. The ligamentum flavum crosses the interlaminar space. Because there is a
gap between the right and left ligamentum flavum in the midline, this ligament is not
penetrated if the needle is in the midline. Immediately within the bony vertebra is the
epidural space, which lies between the bone and the dura. The epidural space contains a
considerable amount of fat and a venous plexus.
Because the muscles of the back are located on either side of the midline, no muscle is
penetrated when doing a lumbar puncture (choice B).
The subarachnoid space (choice C), which is filled with cerebrospinal fluid, is reached after
the needle passes through the epidural space and pierces the dura and arachnoid. The cauda
equina, rather than the spinal cord, is located in the subarachnoid space of the low lumbar
region where a lumbar puncture is performed.
The subcutaneous space (choice D) contains superficial fascia and fat and is found
immediately beneath the skin. After passing through the subcutaneous fascia, the needle
passes through the supraspinous ligament and the interspinous ligament.
The subdural space (choice E) is a potential space between the dura and arachnoid. Normally,
this space does not exist because the arachnoid is closely applied to the dura. If there were
bleeding into this potential space, the two layers would separate and a subdural hematoma
would be produced.

A hypophysectomy is scheduled for removal of a pituitary adenoma. The tumor will be


approached transnasally, through the
A. cavernous sinus
B. ethmoid sinus
C. frontal sinus
D. mastoid sinus
E.

sphenoid sinus

The correct answer is E. The hypophysis, or pituitary gland, is approached transnasally


through the sphenoid sinus. This small gland lies within the sella turcica, a depression in the
sphenoid bone. The pituitary gland hangs beneath the hypothalamus, connected by a slender
stalk, the infundibulum. The base of the infundibulum lies between the optic chiasm and
mamillary bodies.
Inadvertent transgression into the cavernous sinus (choice A) would lead to torrential venous
or arterial hemorrhage from the cavernous sinus itself or the internal carotid artery
contained within.
The ethmoid sinus (choice B) is related to the medial wall of the orbit.
The frontal lobe lies behind the frontal sinus (choice C).
The mastoid sinus (choice D) lies within the temporal bone and communicates with the
middle ear.

A patient has a herniated intervertebral disc impinging on the right C5 nerve roots. Which of
the following movements would most likely be affected?
A. Extension of the fingers
B. Extension of the shoulder
C. Flexion of the elbow
D. Flexion of the wrist
E.

Pronation of the elbow

The correct answer is C. C5 helps mediate flexion, abduction, and lateral rotation of the
shoulder, and flexion of the elbow. Both C5 and C6 mediate extension of the elbow.
Extension of the fingers (choice A) is mediated by C7 and C8.
Extension of the shoulder (choice B) is mediated by C7 and C8.
Flexion of the wrist (choice D) is mediated by C6 and C7.
Pronation of the elbow (choice E) is mediated by C7 and C8.

Following fracture of the humerus, an adult patient has a biopsy of the healing area. Which of
the following types of bone will the biopsy most likely show?
A. Cancellous
B. Compact
C. Spongy
D. Trabecular
E.

Woven

The correct answer is E. Bone is formed by type I collagen fibers, ground substance, and
hydroxyapatite crystals. The collagen is oriented in a layered or lamellar fashion. It can be
parallel (trabecular bone and periosteum) or concentric (Haversian system). When bone is
formed quickly, as in a healing fracture site, metabolic bone disease, or tumor, the collagen is
randomly oriented and is called woven bone.
Compact bone (choice B) is the dense, calcified, external part of the bone. It is lamellar bone.
Cancellous (choice A), spongy (choice C), and trabecular (choice D) bone are all synonymous
terms for the thinner network of bone within the cortex.

A neonate has a prominent defect at the base of his spine through which his meninges and
spinal cord protrude. A failure of which of the following processes is the most common cause
of this type of defect?
A. Development of the body
B. Development of the pedicle
C. Development of primary vertebral ossification centers
D. Development of the superior articular process
E.

Fusion of the vertebral arches

The correct answer is E. The condition described is spina bifida with myelomeningocele. A
failure of the posteriorly located vertebral arches to fuse posteriorly causes spina bifida,
which can vary in severity from a completely asymptomatic, minor abnormality to protrusion
of the spinal cord and roots through an open skin defect, with a very real risk of minor
trauma or infection causing paralysis.
The bodies of the vertebrae (choice A) are the stacking ovoid structures on the anterior
aspect of the spinal canal.
The pedicles (choice B) attach the body ring that surrounds the spinal canal to the body of
the vertebrae.
Failure of development of one of the paired primary ossification centers (choice C) of the
body can produce very severe scoliosis.
The superior articular process (choice D) articulates with the inferior articular facet of the
vertebra above it.

Microscopic examination of a section of a normal young adult ovary reveals large numbers of
unusually large cells surrounded by a single layer of flat epithelial cells. In which phase of the
cell cycle are these cells arrested?
A. Diplotene stage of the first meiotic division
B. Diplotene stage of the second meiotic division
C. Metaphase stage of mitosis
D. Prophase stage of mitosis
E.

Prophase stage of the second meiotic division

The correct answer is A. The cells described are the primordial eggs, which remain stopped in
the diplotene stage of the first meiotic division from before birth until fertilization, a period
which may be 40 or more years.
Choices B and E are incorrect because the oocytes are stopped in the first, not the second,
meiotic division.
Choices C and D are incorrect because the cells described are oocytes and are not in mitosis.

The intraembryonic mesoderm formed during the third week of embryonic life is principally
derived from
A. definitive ectoderm
B. definitive endoderm
C. epiblast
D. primitive node
E.

yolk sac

The correct answer is C. The bilaminar disc forms in the second week of embryonic life by
differentiation of embryoblast cells into epiblast (primary ectoderm adjacent to the amniotic
cavity) and hypoblast (primary endoderm adjacent to the yolk sac; (choice E) layers.
During the third week of life, a linear thickening of some of the epiblast cells produces a
primitive streak, bounded rostrally by the primitive node (choice D).
Epiblast cells migrate from the primitive streak downward, replacing the original hypoblast
with definitive endoderm (choice B).
The migrating epiblast also fills the potential space between the two layers with a third germ
layer, the intraembryonic mesoderm (choice C). The remaining epiblast cells (which don't
migrate) form the definitve ectoderm (choice A).

Pain experienced in the anterior half of the external ear canal is transmitted by which of the
following nerves?
A. Auricular branch of the vagus
B. Auriculotemporal nerve
C. Greater auricular nerve
D. Lesser occipital nerve
E.

Vestibulocochlear nerve

The correct answer is B. The ear has a complex sensory nerve supply, which includes all of
the nerves listed. A consequence of this complexity is that pain actually originating in other
sites (teeth and sinuses are notorious) may be misinterpreted as ear pain; or, less commonly,
pain originating in the ear may be misinterpreted as arising from other sites. The anterior half
of the external ear canal is supplied by the auriculotemporal nerve, which also supplies the
facial surface of the upper part of the auricle.
The auricular branch of the vagus (choice A) supplies the posterior half of the external ear
canal.
The greater auricular nerve (choice C) supplies both surfaces of the lower part of the auricle.
The lesser occipital nerve (choice D) supplies the cranial surface of the upper part of the
auricle.
The vestibulocochlear nerve (choice E) supplies hearing and motion sense.

The suprahyoids, including mylohyoid, digastric, and geniohyoid, act as accessory:


A. elevators
B. depressors
C. retruders
D. protruders
E.

none of the above

The correct answer is B. The major depressor (opener) is the lateral pterygoid, which also
protrudes and causes lateral motion. Mylohyoid, digastric, and geniohyoid can aid this
motion. Mylohyoid can produce opening movement by contracting between the hyoid bone
and the mylohyoid line on both sides of the mandible. The anterior digastric produces an
opening motion as it contracts between the digastric facial sling on the hyoid bone and the
interior of the mandible (digastric fossa). The geniohyoid can contract between the hyoid
bone and the genial tubercle on the mandibular symphysis.

In which of the following organs are fenestrated endothelial cells common?


A. Heart
B. Liver
C. Lungs
D. Pancreas
E.

Stomach

The correct answer is B. Fenestrated ("windowed") endothelial cells, which permit free flow
of serum across the endothelium, are an unusual feature of the hepatic sinusoids. They are
not seen in the other organs listed.
Along the same line, fenestrated capillaries are capillaries containing "windows" or pores that
span the endothelial lining. The pores permit the rapid exchange of water and solutes as
large as small peptides between the plasma and interstitial fluid.

A fall, with hyperextension of the hand, causes anterior dislocation of a carpal bone. Which
nerve is compressed by this injury?
A. Anterior interosseous nerve
B. Median nerve
C. Posterior interosseous nerve
D. Radial nerve
E.

Ulnar nerve

The correct answer is B. The carpal bone that has been dislocated is the lunate, one of the
four proximal carpal bones. This bone has been dislocated into the carpal tunnel, which is
anterior to the carpal bones. The median nerve is the only nerve that traverses the carpal
tunnel.
The anterior interosseous nerve (choice A) is a branch of the median nerve that arises in the
proximal forearm and innervates the deep muscles of the anterior forearm. This nerve does
not traverse the carpal tunnel.
The posterior interosseous nerve (choice C) is a branch of the radial nerve that arises in the
proximal forearm and innervates several muscles of the posterior compartment of the
forearm. This nerve does not traverse the carpal tunnel.
The radial nerve (choice D) innervates posterior muscles of the arm, some posterior muscles
of the forearm, and areas of skin on the posterior surface of the upper limb. It does not pass
through the carpal tunnel.
The ulnar nerve (choice E) innervates some muscles in the anterior compartment of the
forearm and many muscles in the hand. The nerve crosses the wrist superficial to the flexor
retinaculum and is therefore not within the carpal tunnel. It is spared compression by the
dislocation of the lunate.

During development, the formation of the kidney is induced by which of the following
structures?
A. Allantois
B. Mesonephric duct
C. Mesonephros
D. Metanephric duct
E.

Urogenital ridge

The correct answer is D. The metanephric duct (also known as the ureteric bud) is a
diverticulum of the mesonephric duct. It grows to the metanephric mass of the urogenital
ridge. It induces the development of the metanephros, which will give rise to the excretory
units of the definitive kidney. The metanephros, in turn, induces the metanephric duct to
divide into the calyces and the collecting tubules.
The allantois (choice A) is an endodermal diverticulum of the yolk sac. It becomes
incorporated into the urogenital sinus and contributes to the formation of the urinary
bladder. It is not involved with the formation of the kidney.
The mesonephric duct (choice B) is a mesodermal duct into which the mesonephric tubules
drain. The mesonephric duct is the same as the pronephric duct, but at a later stage of
development. The mesonephric duct develops into the ductus deferens in the male. The
metanephric duct is a diverticulum of the mesonephric duct.
The mesonephros (choice C) is the embryonic kidney, which functions in the embryo before
the metanephros (or definitive kidney) develops. The mesonephros develops from the
urogenital ridge.
The urogenital ridge (choice E) is a longitudinal elevation of the intermediate mesoderm. The
embryonic and adult kidneys, as well as the gonads, develop from this mesoderm.

Compression of the obturator nerve will result in weakness of which of the following
muscles?
A. Adductor magnus
B. Biceps femoris
C. Rectus femoris
D. Sartorius
E.

Vastus medialis

The correct answer is A. The obturator nerve innervates the muscles of the medial
compartment of the thigh. The adductors are located inferior to the acetabular surface.
These include the adductor longus, adductor brevis, adductor magnus, and gracilis muscles.
The adductor magnus is also innervated by the tibial nerve.
The biceps femoris (choice B) is in the posterior compartment of the thigh. The long head of
the biceps femoris is innervated by the tibial portion of the sciatic nerve, and the short head
of the biceps femoris is innervated by the common peroneal portion of the sciatic nerve.
The rectus femoris (choice C) and vastus medialis (choice E) are two of the four heads of the
quadriceps femoris muscle. All four heads of the quadriceps femoris muscle are in the
anterior compartment of the thigh and are innervated by the femoral nerve.
The sartorius muscle (choice D) is in the anterior compartment of the thigh and is innervated
by the femoral nerve.

Biopsy demonstrates epithelial metaplasia. Which of the following cell types was most likely
observed in the involved areas?
A. Ciliated columnar epithelium
B. Cuboidal epithelium
C. Keratinizing squamous epithelium
D. Nonciliated columnar epithelium
E.

Nonkeratinizing squamous epithelium

The correct answer is D. The medical condition is Barrett's esophagus, in which the normally
nonkeratinizing squamous epithelium (choice E) of the esophagus undergoes metaplasia to
gastric or intestinal-like epithelium composed of nonciliated columnar epithelial cells.
Barrett's esophagus typically develops in the setting of chronic gastroesophageal reflux and
significantly increases the risk of later development of adenocarcinoma of the distal
esophagus.
Ciliated columnar epithelium (choice A) is found in the respiratory tract.
Cuboidal epithelium (choice B) is found in the kidney, peritoneal lining, and pleural lining.
Keratinizing squamous epithelium (choice C) is found in skin.
Nonkeratinizing squamous epithelium (choice E), in addition to being the normal epithelium
of the esophagus, is found in mouth, nose, and vagina.

Food and oxygen can reach the osteocytes of compact bone through:a) osseous matrixb)
canaliculi c) capillaries d) cartilaginous matrix e) Volkman canals
A. a, b, and c
B. a, c, and e
C. b, c, and d
D. a, c, and d
E.

b, c, and e

The correct answer is E. Bone matrix is generally impermeable to diffusion of nutrients for
any great distance. Osteocytes in need of food and oxygen, and also needing to excrete
waste, therefore need to be near some transport system. In compact bone, Haversian canals
carry arteries, veins, capillaries, and nerves. Horizontal branches of these canals are
sometimes termed Volkman canals. Small canaliculi (small canals) branch out of these canals,
and also radiate from the lacunae that house the osteocytes. All of these structures therefore
can be involved in nutrient transport. Cartilaginous matrix, unlike osseous (bone) matrix, is
permeable, slowly, to nutrient travel. There is no cartilaginous matrix, however, in fully
formed bone tissue.

Which of the following is a derivative of the second pharyngeal arch?


A. Eustachian tube
B. External auditory meatus
C. Palatine tonsil
D. Stylohyoid muscle
E.

Tensor tympani

The correct answer is D. The stylohyoid is derived from the second pharyngeal arch, which
also gives rise to the muscles of facial expression, the stapedius, the posterior belly of the
digastric muscle, Reichert's cartilage, and the facial nerve.
The Eustachian tubes (choice A) are derived from the first pharyngeal pouch, which also gives
rise to the middle ear cavity and the inner epithelial lining of the tympanic membrane.
The external auditory meatus (choice B is derived from the first pharyngeal cleft, which also
gives rise to the outer epithelial lining of the tympanic membrane.
The palatine tonsil (choice C) is derived from the epithelial lining of the second pharyngeal
pouch.
The tensor tympani (choice E) is derived from the first pharyngeal arch, which also gives rise
to the muscles of mastication, the anterior belly of the digastric muscle, the mylohyoideus,
the tensor veli palantini, the maxillary and mandibular bones, and the maxillary and
mandibular divisions of the trigeminal nerve.
Note that knowing the embryology of these structures helps you remember the innervation
of the muscles of the face.
First pharyngeal arch
Second pharyngeal arch muscles of facial expression facial nerve

On microscopic examination of bone, small thread-like canals are seen radiating out from the
area of each osteocyte. These structures are the
A. Volkman canals
B. Canaliculi
C. Osteons
D. Haversian canals
E.

Lacunae

The correct answer is B. In compact bone, the bone is in concentric layers, and small spaces
between layers house the osteocytes, or mature bone cells. These small spaces are called
lacunae. Radiating out from the lacunae are small conducting vessels known as canaliculi.
They help bring nutrients to and waste from the osteocytes. Remember that bone matrix
does not allow diffusion readily. The original source of nutrients to compact bone is
Haversian canals, which contain arteries, veins, and nerves. Smaller perpendicular branches
of these canals are sometimes called Volkman canals. Osteons are also called Haversian
systems, and consist of the Haversian canal, the concentric bone layers around them, the
osteocytes in the concentric rings, the lacunae and canaliculi. Osteons are found in compact
(lamelllar) bone.

A mailman gets a severe bite wound from a pit bull guarding a junkyard. The wound is
cleansed and he receives a booster injection of tetanus toxoid and an injection of penicillin G.
Several days later, the wound is inflamed and purulent. The exudate is cultured on blood agar
and yields gram-negative rods. Antibiotic sensitivity tests are pending. The most likely agent
to be isolated is
A. Bartonella henselae
B. Brucella canis
C. Clostridium tetani
D. Pasteurella multocida

The correct answer is D.Pasteurella multocida is a gram-negative rod that is normal flora of
the oral cavity of dogs and cats. It often causes a local abscess following introduction under
the skin by an animal bite. Most cases occur in children who are injured while playing with a
pet.
Bartonella henselae (choice A) is a very small, gram-negative bacterium that is closely related
to the rickettsia, although it is able to grow on lifeless media. It is the cause of cat-scratch
disease (a local, chronic lymphadenitis most commonly seen in children) and bacillary
angiomatosis (seen particularly in patients with AIDS). In this latter patient population, the
organism causes proliferation of blood and lymphatic vessels, causing a characteristic
"mulberry" lesion in the skin and subcutaneous tissues of the afflicted individual.
Brucella canis(choice B) is a gram-negative rod that is a zoonotic agent. Its normal host is the
dog, but when it gains access to humans, it causes an undulating febrile disease with malaise,
lymphadenopathy, and hepatosplenomegaly. The normal route of exposure is by way of
ingestion of the organism.
Clostridium tetani(choice C) is a gram-positive spore-forming anaerobic rod. It causes tetanus
(a spastic paralysis caused by tetanospasmin). There may be no lesion at the site of
inoculation, and exudation would be extremely rare.

A football player experiences an anterior dislocation of the shoulder. Cutaneous sensation


over the lower half of the deltoid muscle is impaired. These findings suggest damage to
which of the following nerves?
A. Axillary
B. Median
C. Musculocutaneous
D. Radial
E.

Ulnar

The correct answer is A. The axillary nerve can be damaged during anterior dislocation of the
shoulder, causing loss of sensation in the skin overlying the lower half of the deltoid muscle.
The median nerve (choice B) supplies sensation to the anterior arm, palm, and distal aspects
of the lateral three-and-a-half fingers.
The musculocutaneous nerve (choice C) supplies sensation to the lateral surface of the arm
and forearm.
The radial nerve (choice D) supplies sensation to the back of the arm, forearm, and hand.
The ulnar nerve (choice E) supplies sensation to the medial side of the arm, forearm, and
hand.

In a posterolateral herniation of the nucleus pulposus of the intervertebral disk between


vertebrae C4 and C5, what neural structure is most likely to be injured?
A. Anterior ramus C5
B. Posterior ramus C4
C. Spinal cord
D. Spinal nerve C4
E.

Spinal nerve C5

The correct answer is E. A posterolateral herniation of the nucleus pulposus is the most
common type of herniation. This herniation results in the nucleus pulposus occupying space
within the intervertebral foramen. The spinal nerve in the intervertebral foramen between
vertebrae C4 and C5 is the C5 spinal nerve.
The anterior and posterior rami (choices A and B) are found lateral to the intervertebral
foramina and are not compressed by a herniated nucleus pulposus.
The spinal cord (choice C) is within the vertebral canal. A posterior herniation of a nucleus
pulposus (less common) may compress the spinal cord.
Spinal nerve C4 (choice D) is found in the intervertebral foramen between vertebrae C3 and
C4 and would not be compressed by this herniation.

When a large bolus of food is swallowed, the hyoid bone moves anteriorly to open the
pharynx. The muscle responsible for this action is innervated by which of the following
nerves?
A. C1 (ansa cervicalis)
B. CN IX
C. Pharyngeal plexus
D. CN V3
E.

CN VII

The correct answer is A. The muscle that moves the hyoid anteriorly to open the pharynx is
the geniohyoid. This muscle is innervated by the ansa cervicalis branch of C1. The ansa
cervicalis also innervates the thyrohyoid and the intra hyoids.
Cranial nerve IX (choice B) innervates the stylopharyngeus.
The pharyngeal plexus (choice C) innervates the palatoglossus, palatopharyngeus, and
cricopharyngeus.
Cranial nerve V-3 (choice D) innervates most of the muscles of mastication.
Cranial nerve VII (choice E) innervates the buccinator and other muscles of facial expression.

Observation of a hematoxylin and eosin-stained microscope slide reveals that the nuclei are
blue. What is the basis for this observation?
A. Eosin binds to carbohydrates
B. Eosin binds to lipids
C. Eosin binds to nucleic acids
D. Hematoxylin binds to lipids
E.

Hematoxylin binds to nucleic acids

The correct answer is E. Blue hematoxylin binds to polyanions such as RNA and DNA. Nuclei
contain large amounts of DNA and RNA, and they are consequently almost always blue. The
nuclei of dysplastic and cancerous cells are often enlarged and hyperchromatic (e.g., darker
blue) compared with normal cells of similar cell lines because these altered cells often have
extra DNA (are aneuploid) and/or RNA (are metabolically active).
Pink eosin binds relatively nonselectively to cellular components, particularly proteins.
Cytoplasm of different cell lines can be pink, purple, or blue, depending principally on the
number of ribosomes in the cytoplasm. Consequently, blue-tinged cytoplasm tends to
suggest high synthetic activity (i.e., abundant ribosomes).

Damage to the cervical sympathetic trunk in a patient is most likely to be seen in which of the
following findings?
A. Constriction of the right pupil
B. Dilation of the right pupil
C. Inability to abduct the right eye
D. Inability to close the right eye
E.

Paralysis of the platysma muscle on the right side

The correct answer is A. The sympathetic trunk lies posterior to the internal jugular vein.
Preganglionic sympathetic nerve fibers can be damaged in a cervical trunk injury. These nerve
fibers synapse in the superior cervical ganglion on postganglionic sympathetic neurons that
innervate structures in the head. The dilator pupillae muscle (smooth muscle of the iris that
dilates the pupil) is sympathetically innervated; paralysis of this muscle due to interruption of
its innervation results in constriction of the pupil (miosis).
Dilation of the pupil (choice B) would result from paralysis of the sphincter pupillae muscle
(smooth muscle of the iris that constricts the pupil). This muscle is parasympathetically
innervated and would not be affected by this injury.
An inability to abduct the eye (choice C) would result from paralysis of the lateral rectus
muscle, an extraocular muscle that is innervated by the abducens nerve. It would not be
affected by this injury.
An inability to close the eye (choice D) would result from paralysis of the orbicularis oculi
muscle, a skeletal muscle of the face. This muscle is innervated by the facial nerve and would
not be affected by this injury.
The platysma muscle (choice E) is a skeletal muscle in the superficial fascia of the neck. It is
innervated by the facial nerve and would not be affected by this injury.

The greater omentum is derived from which of the following embryonic structures?
A. Dorsal mesoduodenum
B. Dorsal mesogastrium
C. Pericardioperitoneal canal
D. Pleuropericardial membranes
E.

Ventral mesentery

The correct answer is B. Both the omental bursa and the greater omentum are derived from
the dorsal mesogastrium, which is the mesentery of the stomach region. The dorsal
mesentery of the stomach becomes greatly enlarged and forms a large pouch that extends
inferiorly between the body wall and the anterior surface of the small intestine. This is the
greater omentum.
The dorsal mesoduodenum (choice A) is the mesentery of the developing duodenum, which
later disappears so that the duodenum and pancreas come to lie retroperitoneally.
The pericardioperitoneal canal (choice C) embryologically connects the thoracic and
peritoneal canals.
The pleuropericardial membranes (choice D) become the pericardium and contribute to the
diaphragm.
The ventral mesentery (choice E) forms the falciform ligament, ligamentum teres, and lesser
omentum.

The parotid duct pierces the buccinator muscle nearest to the maxillary
A. First molar
B. Second molar
C. Third molar
D. First premolar
E.

Second premolar

The correct answer is B. The parotid gland lies lateral to the buccinator muscle. The facial
nerve and its branches pass through it. It produces a serous-only salivary secretion, which is
second in volume to that produced by the submandibular. The parotid is the largest salivary
gland in size. Its duct, known as Stensen's duct, pierces the buccinator in the vicinity of the
maxillary second molar.

Ligation of which of the following arteries would be most likely to produce the most
significant ischemic damage to the cerebrum?
A. Anterior cerebral artery at its origin from the internal carotid artery
B. Anterior communicating artery
C. Middle cerebral artery at its origin from the internal carotid artery
D. Posterior cerebral artery at its origin from the basilar artery
E.

Posterior communicating artery

The correct answer is C. The circle of Willis is a series of arterial anastomoses that circle
around the pituitary stalk at the ventral surface of the brain. Specifically, the circle of Willis
encircles the infundibulum of the pituitary gland. Hence, the brain can receive blood from
either the carotids or the vertebrals. If any of the vessels of the circle of Willis become
blocked, blood can be routed through the remaining vessels. The object of this question is to
determine which vessel does not belong to the circle of Willis. Of the options listed, only the
middle cerebral artery does not belong to the circle and instead courses laterally to supply
the entire lateral convexity of the brain. Therefore, ligation of this vessel would produce the
most ischemic damage.

Within which of the following anatomic spaces is the meningeal artery normally found?
A. Anterior cranial fossa
B. Inferior orbital fissure
C. Middle cranial fossa
D. Posterior cranial fossa
E.

Superior orbital fissure

The correct answer is C. The middle meningeal artery is a branch of the maxillary artery. It
enters the intracranial cavity through the foramen spinosum, which is located in the floor of
the middle cranial fossa. A tear of the middle meningeal artery results in accumulation of
blood between the inner table of the skull and the dura mater. This is called an epidural
hematoma.
The anterior cranial fossa (choice A) receives the central processes of the bipolar neurons
that form the olfactory nerve. This enters the intracranial cavity via the cribriform plate of the
ethmoid bone.
The inferior orbital fissure (choice B) contains veins that communicate with venous plexuses
in the pterygopalatine and infratemporal fossae.
The posterior cranial fossa (choice D) contains the brainstem and cerebellum. The vertebral
arteries enter the posterior cranial fossa through the foramen magnum, where they unite to
form the basilar artery.
The superior orbital fissure (choice E) contains the ophthalmic vein and all of the cranial
nerves that innervate the eye (CN III, IV, VI), except for the optic nerve (CN II).

A thyroid mass usually moves with swallowing because the thyroid gland is enclosed by
which of the following fascia?
A. Carotid sheath
B. Investing layer of the deep cervical fascia
C. Pretracheal fascia
D. Prevertebral fascia
E.

Superficial fascia

The correct answer is C. The pretracheal layer of the cervical fascia runs from the investing
layers in both sides of the lateral neck and splits to enclose the thyroid gland. Superiorly, it
attaches to the laryngeal cartilages; inferiorly, it fuses with the pericardium. As a result of
these connections, the thyroid gland moves with laryngeal movements. Anatomically, the
thyroid gland curves across the anterior surface of the trachea just below the thyroid
cartilage that forms most of the anterior surface of the larynx. The two lobes of the thyroid
gland are united by a slender connection, the isthmus.
The carotid sheath (choice A) contains the vagus nerve, internal jugular vein, carotid artery,
and lymph nodes.
The investing layer of the deep cervical fascia (choice B) splits to enclose the trapezius and
sternocleidomastoid muscles.
The prevertebral fascia (choice D) covers muscles arising from the vertebrae.
The superficial fascia (choice E) lies immediately deep to the platysma muscle.

A pathologist looks at a histologic section of an aorta stained to demonstrate the many wavy
fibers that provide the aorta with the flexibility to stretch and return to its original shape.
These fibers are also found in substantial numbers in the
A. biceps muscle
B. liver
C. mesentery
D. spleen
E.

vocal cords

The correct answer is E. The fibers are elastic fibers. You should remember three specific
sites where these are found: large arteries (particularly the aorta), vocal cords (or cords), and
ligamenta flava (which connect the vertebrae). Small amounts of elastic tissue are also found
in skin and to a much lesser degree in other tissues. The vocal folds (or cords) protect the
entrance to the glottis. They are located inferior to the ventricular folds. The vocal folds are
highly elastic because they contain bands of elastic tissue called the vocal ligaments. The
vocal folds are involved in the production of sounds.
Elastic tissue is not found in muscle (choice A) or mesentery (choice C).
Elastic tissue is not found in substantial amounts in the liver (choice B) or spleen (choice D).

While testing the gag reflex during a cranial nerve examination, it is noted that when the
right side of the pharyngeal mucosa is touched, the patient's uvula deviates to the right.
When the left side of the pharyngeal mucosa is touched, the patient does not gag. Which of
the following is the most likely location of his lesion?
A. Left glossopharyngeal nerve and left vagus nerve
B. Left glossopharyngeal nerve only
C. Left vagus nerve only
D. Right glossopharyngeal nerve and right vagus nerve
E.

Right glossopharyngeal nerve only

The correct answer is A. The gag reflex requires the glossopharyngeal nerve for the sensory
limb of the reflex (unilateral) and the vagus nerve for the motor limb of the reflex (bilateral).
A lesion of the left glossopharyngeal nerve will denervate the sensory receptors on the left
side of the pharynx, so when the left side is touched, the patient does not feel it and does not
gag. If the left vagus nerve is lesioned, the left side of the soft palate will not elevate during a
gag, and the uvula will deviate to the right. In this case, the patient only feels the touch on
the right side (left glossopharyngeal lesion) and only elevates the right side of the palate (left
vagus lesion).
If the patient had a lesion of the left glossopharyngeal nerve only (choice B), there would
have been no gag reflex when the left side was touched, but there would be a normal gag,
without deviation of the uvula, when the right side was touched.
If the patient had a lesion of the left vagus nerve only (choice C), the patient would have
deviation of the uvula to the right when a gag was elicited, but touching either side of the
pharynx would elicit a gag.
If the patient had a lesion of the right glossopharyngeal nerve and the right vagus nerve
(choice D), touching the right side of the pharynx would not elicit a gag, and touching the left
side of the pharynx would elicit a gag with the uvula deviating to the left.
If the patient had a lesion of the right glossopharyngeal nerve only (choice E), there would be
no gag when the right side was touched, but there would be a normal gag, without deviation
of the uvula, when the left side was touched.

Which of the following placental components is derived from the mother rather than the
fetus?
A. Chorion
B. Cytotrophoblast
C. Lacunar network
D. Primary villus
E.

Syncytiotrophoblast

The correct answer is C. The placenta is derived from both maternal and fetal tissues. The
mother produces the capillary bed that forms the lacunar network of the placenta.
The fetus produces both the cytotrophoblast (choice B) and the syncytiotrophoblast (choice
E), which together form the chorion (choice A). The earliest projections of this are called the
primary villi (choice D).

A person skiing for the first time goes too fast and grabs hold of a tree with one arm to stop
himself. In doing so, he is most likely to injure which of the following?
A. Lower part of the brachial plexus
B. Median nerve
C. Radial nerve
D. Ulnar nerve
E.

Upper part of the brachial plexus

The correct answer is A. Grabbing at something to stop oneself, as in the above example, or
when a skidding motorcyclist tries to catch a lamp pole, or delivering the arms in an extended
position during a breech delivery, are the more common mechanisms by which the lower
part of the brachial plexus is damaged. The result of this type of injury is the loss of motor
function of the intrinsic hand muscles and numbness along the inner aspect of the hand
(ulnar side). The brachial plexus innervates the shoulder girdle and upper limb and divides
into the median, radial, and ulner nerves and their branches. There are contributions from
the ventral rami and spinal nerves C5-7 into these structures.
Damage to the median nerve (choice B) can happen in carpal tunnel syndrome or when a
person attempts to cut their wrists.
Damage to the radial nerve (choice C) can occur with humeral fractures or when one (often
under the influence of alcohol) falls asleep with his or her arm over the back of a chair.
Damage to the ulnar nerve (choice D) can occur when the elbow is struck ("funny bone").
Damage to the upper part of the brachial plexus (choice E) can occur when someone falls on
the point of the shoulder.

An aneurysm of the axillary artery within the axilla is most likely to compress which of the
following neural structures?
A. Axillary nerve
B. Long thoracic nerve
C. Lower trunk of the brachial plexus
D. Medial cord of the brachial plexus
E.

Musculocutaneous nerve

The correct answer is D. Within the axilla, the axillary artery is within the axillary sheath and
is surrounded by the three cords of the brachial plexus, which are also within the axillary
sheath. An aneurysm of the axillary artery may compress any of the three cords. The brachial
plexus innervates the shoulder girdle and upper limb with contributions from the ventral
rami of spinal nerves C5-T1. The nerves that form this plexus originate from trunks and cords
named according to their anatomical location.
The axillary nerve (choice A) is a branch of the posterior cord that leaves the axillary sheath,
then exits the axilla through the quadrangular space to innervate the deltoid muscle.
The long thoracic nerve (choice B) is not within the axillary sheath. It arises from the anterior
rami of the fifth, sixth, and seventh cervical nerves in the neck and courses along the chest
wall to innervate the serratus anterior muscle.
The lower trunk of the brachial plexus (choice C) is not in the axilla. It is formed in the neck
from the anterior rami of the eighth cervical and first thoracic spinal nerves.
The musculocutaneous nerve (choice E) is not within the axillary sheath. It arises as a branch
of the lateral cord of the brachial plexus and enters the arm to innervate the muscles of the
anterior compartment of the arm.

A deep venous thrombosis embolizes, and the embolus lodges at the bifurcation of the
pulmonary trunk, almost completely occluding the vessel. Which part of the heart would be
most significantly and immediately affected by this event?
A. Left main coronary artery
B. Left ventricle
C. Right atrium
D. Right main coronary artery
E.

Right ventricle

The correct answer is E. A massive pulmonary embolus can interrupt pulmonary blood flow,
producing acute cor pulmonale with abruptly developing right ventricular dilatation because
the right ventricle pumps blood directly into the area of blockage, the pulmonary artery.
Other parts of the heart are secondarily affected somewhat later. Acute cor pulmonale is a
surgical emergency, requiring immediate correction of the underlying problem, which is
usually a pulmonary embolus lodged early in the pulmonary circulation. Acute cor pulmonale
is less common than chronic cor pulmonale, which is seen as a complication of many chronic
lung diseases.
The left main coronary artery (choice A) and right main coronary artery (choice D) would be
affected secondarily to the reduced blood flow to the left heart and aorta, from which the
coronary arteries arise.
The left ventricle (choice B) would be affected secondarily by reduced blood flow from the
pulmonary veins to the left atrium.
The right atrium (choice C) would be affected after the right ventricle because it is farther
from the circulatory block.

The primitive streak normally gives rise to which of the following structures?
A. Dorsal root ganglia
B. Lining of the gastrointestinal tract
C. Notochord
D. Spinal cord
E.

Thyroid gland

The correct answer is C. The primitive streak is the region of the epiblast through which the
cells that give rise to the notochord and the mesoderm of the embryo pass. The only adult
derivative of the notochord is the nucleus pulposus of the intervertebral disk. The mesoderm
gives rise to many different tissue types, including muscle, connective tissue, and blood.
The dorsal root ganglia (choice A) are derived from neural crest cells. The neural crest
develops at the time of neurulation, which is a process of infolding of the neural ectoderm
that is induced by the notochord.
The lining of the gastrointestinal tract (choice B) is derived from endoderm. The endoderm
also gives rise to evaginations of the gastrointestinal tract, such as the liver and pancreas. The
smooth muscle and connective tissue of the gastrointestinal tract are derived from
mesoderm.
The spinal cord (choice D) is derived from neural ectoderm, which invaginates during
neurulation to form the neural tube. The caudal part of the neural tube forms the spinal cord,
and the rostral part of the neural tube forms the brain.
The thyroid gland (choice E) is derived from endoderm. The endoderm of the floor of the
pharynx evaginates to form the thyroglossal duct, which descends to form the thyroid gland.
The adult site of the evagination of the thyroglossal duct is marked by the foramen cecum on
the tongue.

A middle-aged patient is diagnosed with a condition that causes excruciating pain near her
nose and mouth when touched. The involved nerve is derived from which of the following
branchial arches?
A. First
B. Second
C. Third
D. Fourth

The correct answer is A. The clinical history suggests trigeminal neuralgia, which is
characterized by extreme pain along the distributions of the maxillary and mandibular
subdivisions of the fifth cranial nerve, associated with a "trigger zone." The trigeminal nerve
is derived from the first branchial arch.
The second branchial arch (choice B) gives rise to the muscles of facial expression and is
innervated by the facial nerve, cranial nerve VII.
The third branchial arch (choice C) is innervated by the ninth cranial nerve, the
glossopharyngeal, which innervates the stylopharyngeus muscle.
The fourth branchial arch (choice D) gives rise to most pharyngeal constrictor muscles and is
innervated by the tenth cranial nerve, the vagus nerve.

A muscle arises from the medial and lateral processes of the tuberosity of the calcaneus and
inserts into the lateral side of the base of the proximal phalanx of the fifth toe. How does this
muscle act on the fifth toe?
A. Abduction
B. Adduction
C. Extension
D. Flexion of the middle phalanx on the proximal phalanx
E.

Flexion of the proximal phalanx on the metatarsal

The correct answer is A. Abduction is movement away from the longitudinal axis of the body.
The muscle is the abductor digiti minimi, the function of which is to abduct (separate) the toe
away from the fourth toe. The abductor digiti minimi is supplied by the lateral plantar nerve.
This muscle also helps act as a "spring" on the sole of the foot during walking. The key to this
question is the insertion on the lateral side of the toe. Contraction on this side pulls the toe
away from midline.
Adduction (opposition of the toes, choice B) is supplied by the plantar interossei.
Extension (choice C) is supplied by the extensor digitorum longus. Flexion is movement in the
anterioral posterior plane that reduces the angle between the articulating joints. Extension
occurs in the same plane but increases the angle between articulating elements.
The flexor digitorum brevis flexes both the middle phalanx on the proximal phalanx (choice
D) and the proximal phalanx on the metatarsal (choice E).

In which regions of the thorax is the thoracic duct found?


A. Anterior and middle mediastinum
B. Anterior and superior mediastinum
C. Middle and posterior mediastinum
D. Middle and superior mediastinum
E.

Posterior and superior mediastinum

The correct answer is E. The mediastinum is divided into four regions. The region above the
manubriosternal junction (level of fourth thoracic vertebra) is the superior mediastinum. The
region below the manubriosternal junction is divided into the anterior mediastinum (anterior
to the pericardium), the middle mediastinum (within the pericardium), and the posterior
mediastinum (posterior to the pericardium). The thoracic duct enters the thorax through the
aortic hiatus of the diaphragm. At this point it lies in the posterior mediastinum, the region
posterior to the pericardium. As it ascends through the thorax and passes the level of the
fourth thoracic vertebra, it enters the superior mediastinum.
The anterior mediastinum (choices A and B) contains the thymus and fatty connective tissue.
The middle mediastinum (choices A, C, and D) contains the heart and the roots of the great
vessels.
In addition to the thoracic duct, the superior mediastinum (choices B, D, and E) contains the
ascending aorta, aortic arch, branches of the aortic arch, descending aorta, superior vena
cava, brachiocephalic veins, thymus, trachea, and esophagus.
In addition to the thoracic duct, the posterior mediastinum (choices C and E) contains the
descending aorta, azygos vein, hemiazygos vein, and the esophagus.

Atherosclerotic occlusion of which of the following arteries would result in insufficient


perfusion of the urinary bladder?
A. External iliac
B. Inferior epigastric
C. Internal iliac
D. Internal pudendal
E.

Lateral sacral

The correct answer is C. The bladder is supplied by the vesicular branches of the internal iliac
arteries. The internal iliacs arise from the common iliac artery.
The external iliac (choice A) also arises from the common iliac artery. It makes no
contribution to the blood supply of the bladder.
The inferior epigastric (choice B) is a branch of the external iliac artery. It serves as a
landmark in the inguinal region.
The internal pudendal (choice D) is a branch of the anterior division of the internal iliac
artery. It supplies the rectal area and the organs.
The lateral sacral (choice E) is a branch of the posterior division of the internal iliac artery.It
supplies sacral structures.

Which cranial nerve carries pain sensation from the posterior third of the tongue?
A. Trigeminal
B. Facial
C. Hypoglossal
D. Glossopharyngeal
E.

Vagus

The correct answer is D. The sensory innervation of the tongue is as follows: the anterior two
thirds of the tongue is innervated by CN V (trigeminal) for touch sensation, and CN VII (facial)
through the chorda tympani for taste sensation. CN IX (glossopharyngeal) innervates the
posterior one third for touch and taste. Some texts state that the most posterior areas of the
tongue may have innervation from CN X (vagus) also. The dividing line for the two thirds and
one third is the row of circumvallate papillae. These papillae themselves are innervated by
CN IX (glossopharyngeal).

A man cannot abduct and adduct the digits of his hand, or oppose his thumb. This occurred
after trying to break a fall. Which neural structure has been injured?
A. Lower trunk of the brachial plexus
B. Median nerve
C. Musculocutaneous nerve
D. Upper trunk of the brachial plexus
E.

Ulnar nerve

The correct answer is A. The lower trunk of the brachial plexus contains nerve fibers from
the eighth cervical and first thoracic spinal nerves. These nerve fibers innervate the intrinsic
muscles of the hand, including the interosseous muscles, responsible for abduction and
adduction of the digits, and the opponens muscle, responsible for opposition of the thumb.
The lower trunk ascends from the lower neck and upper thorax to reach the axilla. Upward
traction on the upper limb may stretch the lower trunk and injure these nerve fibers.
The median nerve (choice B) innervates many muscles of the anterior compartment of the
forearm and some muscles in the hand, including the opponens muscle. The median nerve,
however, does not innervate the interosseous muscles, responsible for abduction and
adduction of the digits.
The musculocutaneous nerve (choice C) innervates the muscle of the anterior compartment
of the arm. It does not innervate any muscles in the hand.
The upper trunk of the brachial plexus (choice D) contains nerve fibers from the fifth and
sixth cervical spinal nerves. The nerve fibers innervate muscles in the proximal part of the
upper limb, including muscles around the shoulder and axilla. No muscles in the hand are
innervated by these nerve fibers.
The ulnar nerve (choice E) innervates many muscles in the hand, including the interosseous
muscles, which are responsible for abduction and adduction of the digits. However, it does
not innervate the opponens muscle, which is responsible for opposition of the thumb.

Which of the following cell types is derived from neural crest cells?
A. Astrocytes
B. Ependymal cells
C. Microglia
D. Oligodendroglia
E.

Pseudounipolar cells

The correct answer is E. Pseudounipolar cells of spinal and cranial nerve ganglia derive from
the neural crest. Other cell types that derive from the neural crest include Schwann cells,
multipolar ganglion cells of autonomic ganglia, chromaffin cells of the adrenal medulla,
odontoblasts, melanocytes, and cells that make up the leptomeninges (pia and arachnoid).
All of the other cell types listed derive from the neural tube. Physiologically, in a unipolar or
pseudounipolar neuron, the dendritic and axonal processes are continuous. The cell body lies
off to one side. Sensory neurons of the peripheral nervous system are usually unipolar, and
their axons may be myelinated.

Which of the following structures lie in the free edge of the lesser omentum?
A. Common bile duct, cystic duct, and hepatic artery
B. Cystic duct, hepatic artery, and hepatic vein
C. Hepatic vein and cystic duct
D. Portal vein, common bile duct, and hepatic artery
E.

Portal vein, hepatic artery, and hepatic vein

The correct answer is D. The free edge of the lesser omentum contains three important
structures: the common bile duct, the hepatic artery, and the portal vein.
Neither the cystic duct (choices A, B, and C) nor the hepatic vein (choices B, C, and E) lies in
the free edge of the lesser omentum.

Which of the following best describes the direction of blood flow through a patent ductus
arteriosus in an infant?
A. From aorta to left pulmonary artery
B. From aorta to left pulmonary vein
C. From aorta to right pulmonary artery
D. From left pulmonary artery to aorta
E.

From right pulmonary artery to aorta

The correct answer is A. The ductus arteriosus connects the left pulmonary artery to the
aortic arch. It is derived from the left sixth aortic arch. During prenatal life, the pressure
gradient causes blood to flow from the left pulmonary artery to the aorta. This allows fetal
blood to bypass the pulmonary system. However, after birth, the pressure gradient reverses,
and if the ductus arteriosus remains patent, the flow is from the aorta to the left pulmonary
artery.
The ductus arteriosus does not connect to the pulmonary veins or the right pulmonary artery
(choices B, C, and E).
The flow through the ductus arteriosus is from the left pulmonary artery to the aorta (choice
D) prior to birth, it but reverses after birth.

An infant is noted to have an adducted and internally rotated shoulder with an extended,
pronated elbow. Loss of flexion and supination by which of the following muscles accounts
for the elbow position?
A. Biceps
B. Brachialis
C. Coracobrachialis
D. Deltoid
E.

Pronator teres

The correct answer is A. Loss of abductors and external rotators can cause an adducted and
internally rotated shoulder. The biceps accomplish both of these motions. Loss of flexion and
supination by the biceps muscle causes the extended, pronated elbow ("porter's tip" or
"waiter's tip" position).
The brachialis muscle (choice B) is a pure flexor of the elbow.
The coracobrachialis muscle (choice C) functions at the elbow by helping to hold an object
under the arm.
The deltoid muscle (choice D) is a powerful abductor of the shoulder.
The pronator teres muscle (choice E) is a powerful pronator of the elbow.

A patient who has been exhibiting various endocrine abnormalities has an MRI scan of the
head. This scan reveals a small tumor of the pituitary gland. If this tumor expands laterally,
which of the following nerves will most likely be affected first?
A. Abducens nerve
B. Oculomotor nerve
C. Optic nerve
D. Trigeminal nerve
E.

Trochlear nerve

The correct answer is A. The pituitary gland is located in the pituitary fossa within the skull.
The floor of this fossa is formed by the sella turcica. The lateral walls of the fossa are formed
by the cavernous sinuses. The abducens nerve passes through the cavernous sinus along with
the internal carotid artery. As the tumor expands laterally, the first nerve that will be
encountered is the abducens nerve, producing a lateral rectus palsy. The abducens nerve or
cranial nerve VI originates from the pons and enters the lateral rectus muscle. It is
responsible for aiding in eye movements.
The oculomotor nerve (choice B) lies in the lateral wall of the cavernous sinus. It is farther
from the pituitary gland than is the abducens nerve.
The optic nerve (choice C) is anterosuperior to the pituitary gland. Upward expansion of the
tumor may compress the optic chiasm.
The trigeminal nerve (choice D) is found posterior to the cavernous sinus. Two of its three
divisions (ophthalmic and maxillary divisions) pass through the lateral wall of the cavernous
sinus and are farther from the pituitary gland than is the abducens nerve.
The trochlear nerve (choice E) is also in the lateral wall of the cavernous sinus and would be
affected later if the tumor continued to expand.

The large cephalic vein can best be found between which of the following muscles?
A. Biceps and triceps
B. Brachialis and medial head of the triceps
C. Coracobrachialis and brachialis
D. Deltoid and pectoralis major
E.

Sternocleidomastoid and deltoid

The correct answer is D. The cephalic vein arises from the elbow along the anterior lateral
surface of the arm. Specifically, the superficial arch empties into the cephalic vein, which
ascends along the radial side of the forearm. It can be reliably found in the deltopectoral
groove between the deltoid and pectoralis major. This groove lies diagonally (arising
medially) below the clavicle. The cephalic vein is good for cannulation because its location is
particularly constant.
The cephalic vein is unrelated to the triceps, although it does pass in the arm over the biceps
(choice A).
The cephalic vein is not related to either the triceps or the brachialis (choice B).
The cephalic vein is not related to either the coracobrachialis or the brachialis (choice C).
The cephalic vein is not related to the sternocleidomastoid (choice E).

An inflammatory process in the temporal bone has resulted in a swelling of the facial nerve
within the facial canal. Which muscle may be paralyzed as a result of this compression?
A. Anterior belly of the digastric
B. Geniohyoid
C. Stapedius
D. Stylopharyngeus
E.

Masseter

The correct answer is C. The stapedius muscle is innervated by the facial nerve (cN VII). This
muscle is located in the middle ear and attaches to the neck of the stapes. Contraction of the
stapedius reduces the amplitude of oscillation of the stapes and thus reduces the perceived
loudness of a sound. Paralysis of this muscle may result in hyperacusis. Note that the seventh
nerve also innervates all muscles of facial expression.
The anterior belly of the digastric muscle (choice A) is a muscle in the floor of the mouth that
is innervated by the mandibular division of the trigeminal nerve.
The geniohyoid muscle (choice B) is one of the suprahyoid muscles in the neck. This muscle is
innervated by C1 spinal nerve fibers that travel for a short distance with the hypoglossal
nerve.
The stylopharyngeus muscle (choice D) is one of the longitudinal muscles of the pharynx that
acts to elevate the pharynx.
The masseter muscle (choice E) is one of the muscles of mastication. All of the muscles of
mastication are innervated by the mandibular division of the trigeminal nerve.

Intramuscular injections should be given in the upper, outer quadrant of the buttocks to
prevent damage to which of the following nerves?
A. Common peroneal
B. Lateral femoral cutaneous
C. Obturator
D. Sciatic
E.

Superior gluteal

The correct answer is D. Injections are given in the upper, outer quadrant of the buttocks to
prevent damage to the sciatic nerve, which is present in the lower quadrant. The other
nerves listed are not particularly vulnerable to injections into the buttocks.
The common peroneal nerve (choice A) is a branch of the sciatic nerve that diverges from it
in the popliteal fossa. It then divides into the superficial and deep peroneal nerves.
The lateral femoral cutaneous nerve (choice B derives from the lumbar plexus, emerges
slightly below the anterior superior iliac spine, and supplies the skin of the anterior thigh
down to the knee.
The obturator nerve (choice C) derives from the lumbar plexus, diverges from the femoral
nerve in the psoas muscle, and passes medially along the lateral pelvic wall to run in the
obturator canal, where it divides into anterior and posterior divisions. The anterior division
generally supplies the gracilis, adductor brevis, and adductor longus; the posterior division
generally supplies the obturator externus and the adductor part of the adductor magnus.
The superior gluteal nerve (choice E) is a branch of the sacral plexus. It supplies the gluteus
minimus and medius and the tensor fascia lata. Small branches of this nerve are likely to be
encountered in the upper outer quadrant of the buttock, but injection here is relatively safe.

The heart of an embryo first begins beating at which of the following ages?
A. 2 weeks
B. 3 weeks
C. 4 weeks
D. 6 weeks
E.

8 weeks

The correct answer is C. While the third-week embryo is a primitive trilaminar plate, in the
fourth week, the heart begins to form and begins beating almost immediately.
Hematopoiesis occurs in the yolk sac, and a primitive circulatory system connects the
capillary plexuses of the yolk sac and chorion to the embryo. Partitioning of the atrium also
begins in the fourth week. During the fifth week, cardiac septa form and the atrioventricular
(AV) cushions fuse. By the sixth week, the heart is close to fully formed. This early sequence
for the heart explains why it is so difficult to try to prevent congenital malformations of the
cardiovascular system from occurring because the mother of a 6-week-old fetus is only about
8 weeks from her last menstrual period and may have assumed that she just "missed a
period" (a very common phenomenon) for reasons other than pregnancy.

A patient has suffered a penetrating wound in the left fourth intercostal space immediately
lateral to the sternal border. Which of the following thoracic structures is most likely to have
been injured?
A. Left atrium
B. Left ventricle
C. Right atrium
D. Right ventricle
E.

Upper lobe of the left lung

The correct answer is D. The right ventricle forms most of the anterior wall of the heart and
extends from approximately the right border of the sternum to approximately 2 inches to the
left of the sternum at the level of the fourth intercostal space.
The left atrium (choice A) forms the posterior wall of the heart. The only portion of the left
atrium seen on the anterior surface of the heart is the left auricular appendage, which is at
the level of the second intercostal space on the left.
The left ventricle (choice B) forms most of the left border of the heart and the diaphragmatic
surface of the heart. It forms the anterior wall of the heart in a region from approximately 2
to 3 inches from the left border of the sternum from the third to the fifth intercostal space.
The right atrium (choice C) forms the right border of the heart. Its anterior surface is on the
right side of the sternum from approximately the third to the sixth rib.
The left lung (choice E) is displaced away from the sternum on the left side by the presence
of the heart.

A patient with damage to the humerus and weakness in lateral rotation and abduction of the
arm has probably damaged which nerve?
A. axillary
B. dorsal scapular
C. radial
D. suprascapular
E.

thoracodorsal

The correct answer is A. Because of the proximity of the axillary nerve to the glenohumeral
joint, a fracture of the surgical neck of the humerus or an inferior dislocation of the
humerous could damage the nerve. The axillary nerve innervates the deltoid muscle. The
deltoid abducts, adducts, flexes, extends, and rotates the arm medially. The axillary nerve
also innervates the teres minor, which rotates the arm laterally.
The dorsal scapular nerve (choice B) innervates both the major and minor rhomboid muscles.
These muscles raise the medial border of the scapula upward and laterally.
The radial nerve (choice C) innervates muscles involved in the movement of the forearm and
hand.
The suprascapular nerve (choice D) innervates the supraspinatus and infraspinatus. The
supraspinatus abducts the arm, whereas the infraspinatus rotates the arm laterally. This
nerve travels along the posterior aspect of the scapula and would not easily be subjected to
injury in a dislocation of the shoulder joint.
The thoracodorsal nerve (choice E) innervates the latissimus dorsi, which adducts, extends,
and rotates the arm medially.

If the tendon of the biceps brachii is severed at the elbow, and the cut causing this injury
extends 2 cm medially, which of the following structures will be injured?
A. Brachial artery
B. Musculocutaneous nerve
C. Profunda brachii artery
D. Radial nerve
E.

Ulnar nerve

The correct answer is A. The brachial artery is immediately medial to the tendon of the
biceps brachii at the elbow. As the artery enters the forearm, it is covered by the bicipital
aponeurosis, a broadening of the biceps tendon.
The musculocutaneous nerve (choice B) does not cross the elbow. The musculocutaneous
nerve gives off all of its muscular branches to muscles in the arm. The remainder of the nerve
is then renamed the lateral cutaneous nerve of the forearm, which passes the elbow lateral
to the tendon of the biceps.
The profunda brachii artery (choice C) arises from the brachial artery in the proximal part of
the arm. It accompanies the radial artery in the musculospiral groove and then divides into
the radial collateral artery and middle collateral artery, which cross the elbow lateral to the
tendon of the biceps.
The radial nerve (choice D) lies within the musculospiral groove along the back of the
humerus, then passes between the brachioradialis muscle and the brachialis muscle at the
elbow, lateral to the tendon of the biceps.
The ulnar nerve (choice E) crosses the elbow posterior to the medial epicondyle of the
humerus. It then passes between the two heads of the flexor carpi ulnaris and courses
through the forearm deep to this muscle.

Radiographic studies confirm that an embryo has split at the blastocyst stage. Splitting of the
embryo at the blastocyst stage results in which of the following?
A. Conjoined twins
B. Dizygotic twins
C. Fraternal twins
D. Monozygotic twins
E.

Single gestation

The correct answer is D. Monozygotic twins, or identical twins, develop from a single
fertilized egg that subsequently splits during either the blastomere or blastocyst stage. It is
more common in the blastocyst stage at the end of the first week. This results in two inner
cell masses in the same blastocyst cavity. They usually develop a common placenta and
chorionic cavities but separate amniotic cavities. Splitting in the second week usually results
in shared amniotic cavities as well.
Conjoined twins (choice A), or Siamese twins, result from incomplete splitting of the embryo.
Dizygotic twins (choice B) and fraternal twins (choice C) are the same, and are the most
common type of twins. They share the same genetic relatedness as do siblings of separate
pregnancies. This type of twinning occurs because of simultaneous double ovulation followed
by fertilization by two sperm. They each develop their own placenta and membranes.
A single gestation (choice E), and a single birth, is the result of a single fertilization without
any splitting of the embryo.

Which salivary gland cells have folded cell membranes at their base that are filled with large
numbers of mitochondria?
A. Intercalated duct cells
B. Striated duct cells
C. Serous secretory cells
D. Mucus secretory cells
E.

Myoepithelial (basket) cells

The correct answer is B. Striated duct cells line sections of salivary ducts, which modify the
composition of saliva. In particular, they may add potassium, remove sodium, or add
bicarbonate. This ionic movement, mostly against a concentration gradient, requires energy
in the form of ATP. This ATP is made in mitochondria. Rows of mitochondria in these cells,
located near the base, appear in light microscopy to be stripes or striations. Mucous and
serous secretory cells, because of their protein secreting function, may have highly
developed ribosomes, rough endoplasmic reticulum, and golgi complexes. Intercalated duct
cells have no unique specializations of note. Myoepithelial cells have contractile ability,
somewhat like muscle cells.

A wound to the posterior left axillary line, between the ninth and tenth rib, and extending
approximately 5 cm deep, will most likely damage which organ?
A. Ascending colon
B. Duodenum
C. Left kidney
D. Left lobe of the liver
E.

Spleen

The correct answer is E. The spleen follows the long axes of ribs 9 to 11 and lies mostly
posterior to the stomach, above the colon, and partly anterior to the kidney. It is attached to
the stomach by a broad mesenterial band, the gastrosplemic ligament. Therefore, it is the
most likely organ of the group to be pierced by a sharp object penetrating just above rib 10 at
the posterior axillary line. Note that the pleural cavity, and possibly the lower part of the
inferior lobe of the lung, would be pierced before the spleen.
The ascending colon (choice A) is on the wrong side (the right) to be penetrated by a sharp
instrument piercing the left side.
Most of the duodenum (choice B) is positioned too far to the right to be affected by this
injury. Even the third part of the duodenum, which runs from right to left, would still be out
of harm's way. In addition, the duodenum lies at about levels L1 to L3, placing it too low to be
injured in this case.
The superior pole of the left kidney (choice C) is bordered by the lower part of the spleen.
However, it is crossed by rib 12 and usually does not extend above rib 11. It would probably
be too low and medial to be injured in this case because this penetration is at the posterior
axillary line.
The left lobe of the liver (choice D) is positioned just beneath the diaphragm, just over and
anterior to the stomach. The anterior positioning of this structure makes it an unlikely
candidate for injury in this case. Even with deep penetration at the correct angle, it would
not be penetrated before the spleen.

Mucopurulent exudate from maxillary sinusitis would be most likely to drain through an
ostium in the
A. bulla ethmoidalis
B. hiatus semilunaris
C. inferior nasal meatus
D. sphenoethmoidal recess
E.

superior nasal meatus

The correct answer is B. This patient has two risk factors for sinusitis: chronic rhinitis and
allergy. She probably also has aspirin allergy, which is associated with the triad of nasal
polyps, asthma, and sinusitis. In maxillary sinusitis, exudate may drain into the middle meatus
through an ostium in the hiatus semilunaris, which contains openings to the frontal and
maxillary sinuses and anterior ethmoidal cells.
The bulla ethmoidalis (choice A), also part of the middle meatus, contains an opening to the
middle ethmoidal air cells.
The inferior nasal meatus (choice C) receives fluid from the nasolacrimal duct, which drains
tears from the medial aspect of the orbit to the nasal cavity.
The sphenoethmoidal recess (choice D) is located above the superior concha and contains an
opening for the sphenoid sinus.
The superior nasal meatus (choice E) is located above the superior concha and contains an
opening for the sphenoid sinus.

Which of the tongue papillae are NOT vascular?


A. Fungiform
B. Filiform
C. Vallate
D. Foliate

The correct answer is B. The four major lingual papillae types are listed in the answer.
Fungiform papillae are fairly numerous, vascular, have taste buds, and are found primarily on
the anterior tongue. Filiform are avasacular, without taste buds, are the most numerous, and
are found in rows. Vallate are largest, usually 7-9 in number, contain taste buds, and serous
salivary glands of Von Ebner. Foliate are on the lateral surface of the tongue in ridges but are
usually considered rudimentary and nonfunctional in man.

A patient is involved in an accident which tears the left lateral pterygoid muscle completely.
On attempting to open, the patients mandible will move:
A. Left
B. Right
C. In an elevating direction
D. In a direct protruding direction

The correct answer is A.There are two ways to answer this question. The formulaic method
says that damage to a lateral pterygoid or the innervation to it will make the mandible
deviate toward the side of the damage. A deeper understanding of the question would show
that the lateral pterygoid is the primary protruder of the mandible. Contraction of both
lateral pterygoids produces straight protrusion. If the left lateral pterygoid is torn, the right
muscle will begin to protrude the right side, with the left side remaining stable, and acting as
a stationary pivot point. As a result, the mandible will pivot out and toward the left (the side
of injury). Note that damage to muscles or innervation to one side of a tongue will also
produce deviation toward the injured side.

Which of the following helps to anchor an epithelial cell to the basement membrane?
A. Adherent junction
B. Connexon
C. Gap junction
D. Hemidesmosome
E.

Tight junction

The correct answer is D. Desmosomes are specializations of the lateral surface of the cell
formed from the juxtaposition of two disc-shaped structures in adjacent cells, acting much
like spot welds between the two cells. Hemidesmosomes are basically desmosomes between
a single cell and an extracellular matrix structure, such as a basement membrane.
Adherent junctions (choice A), or zonula adherens are bandlike junctions that help attach
adjacent epithelial cells to each other.
Gap junctions consist of a hexagonal lattice of tubular proteins called connexons (choice B),
which form channels allowing communication between cells.
Tight junctions (choice E), or zonula occludens, are formed by the fusion of the outer leaflets
of apposed cell membranes on the lateral cell surfaces, just beneath the apical poles. They
form a barrier to permeability, or a seal around the cell.

From which of the following arteries does the sphenopalatine artery arise?
A. External carotid
B. Facial
C. Maxillary
D. Ophthalmic
E.

Transverse facial

The correct answer is C. The sphenopalatine artery is the terminal branch of the maxillary
artery. The maxillary artery arises from the external carotid artery and then passes through
the infratemporal fossa, giving off branches to structures in this region. The artery then
passes through the pterygomaxillary fissure to enter the pterygopalatine fossa. The terminal
branch then passes through the sphenopalatine foramen to enter the nasal cavity and supply
much of the nasal mucosa, particularly in the posterior region of the nasal cavity.
The external carotid artery (choice A) arises from the common carotid artery at the carotid
bifurcation. The external carotid artery has eight branches that supply the head and neck
region: the superior thyroid, ascending pharyngeal, occipital, lingual, facial, posterior
auricular, maxillary, and superficial temporal arteries.
The facial artery (choice B) arises from the external carotid artery. It provides blood supply to
much of the facial region. The nasal cavity (particularly the anterior portion) receives some of
its blood supply from branches of the facial artery.
The ophthalmic artery (choice D) arises from the internal carotid artery immediately after the
internal carotid artery emerges from the cavernous sinus. The ophthalmic artery passes
through the optic canal to supply structures in the orbit.
The transverse facial artery (choice E) is a branch of the superficial temporal artery. It
supplies blood to structures in the upper portion of the lateral face.

A physician notes weakness of a patient's thumb in extension, although rotation, flexion,


abduction, adduction, and opposition are normal. Which of the following nerves is most likely
involved?
A. Median and radial
B. Median and ulnar
C. Median only
D. Radial only
E.

Ulnar only

The correct answer is D. All three of the nerves listed innervate muscles that supply the
thumb. Extension is provided by the extensors pollicis longus and brevis, which are
innervated by the radial nerve.
The median nerve (choices A, B, and C) supplies the thenar group, which allows the thumb to
oppose, flex, abduct, and rotate.
The ulnar nerve (choices E and B) supplies the adductor pollicis, which adducts the thumb.

While lying supine in bed eating, a child aspirates a peanut. Which of the following
bronchopulmonary segments would this foreign object most likely enter?
A. Apical segment of the left upper lobe
B. Apical segment of the right upper lobe
C. Medial segment of the right middle lobe
D. Posterior basal segment of the left lower lobe
E.

Superior segment of the right lower lobe

The correct answer is E. Because the right main bronchus is wider and more vertical than the
left, foreign objects are more likely to be aspirated into the right main bronchus. The superior
segmental bronchus of the lower lobar bronchus is the only segmental bronchus that exits
from the posterior wall of the lobar bronchi. Therefore, if a patient is supine at the time of
aspiration, the object is most likely to enter the superior segmental bronchus of the lower
lobe.
None of the segmental bronchi of the left lung (choices A and D) are likely to receive the
object because the object is less likely to enter the left main bronchus.
The apical segment of the right upper lobe (choice B) is not likely to receive the foreign
object because of the sharp angle that the upper lobar bronchus makes with the right main
bronchus and the sharp angle that the apical segmental bronchus makes with the lobar
bronchus.
The medial segmental bronchus of the right middle lobe (choice C) arises from the anterior
wall of the right middle lobar bronchus. Therefore, when the patient is supine, the effect of
gravity will tend to prevent the object from entering this segmental bronchus.

Which muscle has fibers that enter the articular disc and capsule?
A. Medial pterygoid
B. Buccinator
C. Temporalis
D. Lateral pterygoid
E.

Masseter

The correct answer is D. Only the lateral pterygoid enters the temperomandibular joint
(TMJ). Its fibers join the articular disc and capsule. These fibers are from the superior head of
the muscle. Fibers from the inferior head enter the bony head of the mandibular condyle.
The lateral pterygoid protrudes the mandible, depresses (opens) it, and moves it left and
right. None of the other muscles listed enter the TMJ. Medial pterygoid and masseter
primarily elevate (close) the mandible, whereas temporalis elevates (anterior fibers) and
retrudes (posterior fibers). Buccinator forms the substance of the cheek and is also a muscle
of facial expression. It is not a muscle of mastication.

Luminal narrowing of which of the following vessels would compromise blood flow through
the renal arteries?
A. Abdominal aorta
B. Celiac trunk
C. Common iliac artery
D. Inferior mesenteric artery
E.

Superior mesenteric artery

The correct answer is A. The renal arteries emerge from the abdominal aorta at about the
level of the L1/L2 intervertebral disk and travel at nearly right angles to it (on the right,
passing posterior to the inferior vena cava) to enter the hilum of the kidney.
The celiac trunk (choice B) gives off the common hepatic, splenic, and left gastric arteries.
The common iliac artery (choice C) gives off the internal and external iliac arteries. In
addition, an unascended pelvic kidney may be supplied by the common iliac artery.
The inferior mesenteric artery (choice D) gives off the superior rectal, sigmoid, and left colic
arteries.
The superior mesenteric artery (choice E) gives off the inferior pancreaticoduodenal,
intestinal (ileal and jejunal), right colic, middle colic, and ileocolic arteries.

Which of the following is present in males but not in females?


A. Bulbospongiosus muscle
B. Bulbourethral gland
C. Corpus cavernosum
D. Membranous urethra
E.

Perineal body

The correct answer is B. The bulbourethral glands are paired structures located within the
deep perineal pouch, embedded within the sphincter urethrae. Their ducts pass to the
spongy urethra. The homologous female structures are the greater vestibular (Bartholin's)
glands, which are located in the superficial perineal pouch.
The bulbospongiosus muscles (choice A) lie superficial to the bulb of the penis in males and
to the bulbs of the vestibule in females.
The corpora cavernosa (choice C) are paired structures, consisting of cavernous erectile
tissue that form a large portion of the penile shaft in males and of the body of the clitoris in
females.
The membranous urethra (choice D) is the portion of the urethra that passes through the
urogenital diaphragm in both males and females.
The perineal body (choice E) is the centrally located tendinous structure that provides
attachment for perineal musculature in both males and females. It separates the urogenital
area from the anal area and is an important obstetric landmark.

The left adrenal vein drains directly into which of the following veins?
A. Hemiazygos vein
B. Inferior vena cava
C. Left renal vein
D. Splenic vein
E.

Superior mesenteric vein

The correct answer is C. The left adrenal vein and the left gonadal vein (either testicular or
ovarian) drain into the left renal vein. The left renal vein then drains into the inferior vena
cava. In contrast, the right adrenal vein and right gonadal vein drain directly into the inferior
vena cava.
The hemiazygos vein (choice A) receives the venous drainage from the body wall on the left
side of the thorax and abdomen. No visceral organs drain directly to the azygos or
hemiazygos veins.
The inferior vena cava (choice B) receives the direct venous drainage from the right adrenal
vein, but not the left adrenal vein. Remember, the inferior vena cava is on the right side of
the abdomen.
The splenic vein (choice D) receives the venous drainage from the spleen and part of the
pancreas and stomach. The splenic vein is part of the portal venous system.
The superior mesenteric vein (choice E) receives venous drainage from much of the intestinal
tract. It is part of the portal venous system and joins with the splenic vein to form the portal
vein.

If a person has normal musculature, but has difficulty swallowing, which nerves should be
tested for function?
A. Hypoglossal and phrenic
B. Hypoglossal and splanchnic
C. Glossopharyngeal and vagus
D. Phrenic and vagus
E.

Splanchnic and vagus

The correct answer is E. The upper 2/3 of the esophagus contains striated muscle. It is
derived from the pharyngeal arches and innervated by the vagus nerve (CN X). The lower 1/3
contains smooth muscle from splanchnic mesoderm and is innervated by the splanchnic
plexus. The vagus nerve has mixed sensory and motor functions. The sensory fibers innervate
the sensory nuclei and autonomic centers of the medulla oblongata. The motor fibers
innervate muscles of the palate, pharynx, and respiratory and cardiovascular systems.
The hypoglossal nerve (choices A and B), or CN XII, moves the tongue.
The phrenic nerve (choices A and D), derived from C3, C4, and C5, innervates the muscle of
the diaphragm.
The glossopharyngeal nerve (choice C), or CN IX, functions in taste, swallowing, and
salivation, as well as monitoring the activity of the carotid body.

Which of the following cells is the germ cell closest to the basal lamina in the seminiferous
tubule?
A. Primary spermatocyte
B. Secondary spermatocyte
C. Spermatid
D. Spermatogonia
E.

Spermatozoa

The correct answer is D. Maturation of germ cells (spermatogenesis) within the seminiferous
tubules occurs in a concentric pattern, with the less mature spermatogonia near the basal
lamina and the mature forms near the tubule center. Along this route the developing sperm
are nurtured by sertoli cells. Spermatogonia are 2N cells and mature into larger primary
spermatocytes (4N) (choice A). These mature into secondary spermatocytes (2N) (choice B),
and finally into spermatids (1N) (choice C). Spermatids undergo spermiogenesis to become
mature spermatozoa (choice E). Acrosomes form from the Golgi apparatus, and a flagellum
forms from microtubules. Unneeded organelles are shed. The seminiferous tubules of a
reproductive-age male should exhibit all stages of maturation, with mature flagellated sperm
in their centers, no longer associated with sertoli cells.

Which muscle is the chief mover of the mandible TOWARD the left?
A. Left medial pterygoid
B. Left lateral pterygoid
C. Right medial pterygoid
D. Right lateral pterygoid

The correct answer is D. There are two methods for answering this question. The formula
method states that the right lateral pterygoid moves the mandible left, whereas the left
lateral pterygoid moves the mandible right. The understanding method begins with the idea
that the medial pterygoid is a closer (elevator). The lateral pterygoid connects to the condyle
and is a protruder. If the right lateral pterygoid pulls the right condyle out while the left
lateral pterygoid and condyle remain stable, the left side will act as a stationary pivot point
while the right side protrudes outward and also toward the left (medially).

Secretory basket cells are located


A. inside secretory acini
B. surrounding the intercalated duct
C. between secretory cells and the basal lamina
D. surrounding the striated duct

The correct answer is C. Secretory basket cells are found in salivary gland acini and are also
called myoepithelial cells. The name implies that they are epithelial cells with muscle-like
contractile properties. The are found between secretory cells and the basal lamina and exert
pressure on the secretory cells to release their secretions. Secretory acini themselves are
composed of mucous or serous secreting cells. Intercalated duct cells are nonspecialized
cuboidal epithelial cells. Striated duct cells are columnar epithelial cells with striations
(stripes) of rows of mitochondria used for ATP generation for active transport.

If cirrhosis causes obstruction of the portal circulation within the liver, portal blood could still
be conveyed to the caval system via which of the following?
A. Azygos and hemiazygos veins
B. Gonadal veins
C. Internal iliac veins
D. Splenic vein
E.

Vesical venous plexus

The correct answer is A. The esophageal venous plexus, which drains into the azygos and
hemiazygos veins within the thorax, has anastomoses with branches of the left gastric vein.
Thus, following blockage of the portal vein, portal blood may enter the superior vena cava via
the azygos system. Other important portacaval connections include the superior rectal vein
with the middle and inferior rectal veins; paraumbilical veins and epigastric veins
(engorgement of these vessels results in caput medusae); and the colic and splenic veins with
renal veins and veins of the posterior body wall.
The gonadal veins (choice B) exclusively drain the gonads (although in the female, the
ovarian vein communicates with the uterovaginal plexus). These vessels have no
anastomoses with portal veins.
The internal iliac veins (choice C), which drain most of the pelvis and much of the inferior
extremities, have no demonstrated portal anastomoses.
The splenic vein (choice D) is incorrect because it's in itself a component of the portal venous
system.
The vesical venous plexus (choice E), which is situated well within the pelvis and drains the
bladder and the prostate (or uterus and vagina) has no association with portal vessels.

A child who has had abnormal development of the membranous bones has a broad skull with
associated facial and dental anomalies. Which other bones are most likely to also be
affected?
A. Clavicles
B. Femurs
C. Metatarsals
D. Phalanges
E.

Tibias

The correct answer is A. In a syndrome called cleidocranial dysostosis, absence of part of the
clavicles accompanies a broad skull and facial and dental anomalies. This syndrome affects
bones formed by intramembranous ossification.
The femurs (choice B), metatarsals (choice C), phalanges (choice D), and tibias (choice E) are
cartilaginous (formed by endochondral ossification) rather than membranous bones.

Which of the following fibers provide the only output from the cerebellar cortex?
A. Climbing
B. Golgi cell
C. Granule cell
D. Mossy
E.

Purkinje

The correct answer is E. Two basic things about cerebellar circuitry that are well-worth
knowing are that Purkinje cells of the cerebellar cortex project to the deep cerebellar nuclei
and that these nuclei project out of the cerebellum. Purkinje cells are located in the second
(Purkinje) cell layer of the cerebellar cortex and form inhibitory synapses on the deep
cerebellar nuclei.
Climbing fibers (choice A) are afferents to the cerebellum. Specifically, they originate from
the medullary olivary nuclear complex, enter the cerebellum through the inferior cerebellar
peduncle, and project to the deep cerebellar nuclei and the lateral cerebellar hemispheres.
Golgi cell bodies (choice B) reside in the granule cell layer of the cerebellar cortex and project
predominantly to the granule cells, where they form inhibitory synapses. Therefore, they
reside in and project to the cerebellar cortex.
Granule cells (choice C) reside in the granule cell layer of the cerebellar cortex and send their
axons to the moleculary layer, where they bifurcate into parallel fibers. Granule cells,
therefore, both reside in and project to the cerebellar cortex.
Mossy fibers (choice D) consist of all of the afferents (except the climbing fibers) to the
cerebellum. These fibers project to deep cerebellar nuclei and glomeruli in the granular layer
of the cerebellar cortex.

Which membrane is in the way is you try to reach the lesser peritoneal sac and head of the
pancreas after penetrating the greater peritoneal sac?
A. Falciform ligament
B. Gastrohepatic ligament
C. Gastrosplenic ligament
D. Hepatoduodenal ligament
E.

Splenorenal ligament

The correct answer is B. The gastrohepatic ligament is the part of the lesser omentum that
separates the greater peritoneal sac from the right portion of the lesser peritoneal sac. This
portion of the lesser omentum has no significant blood vessels within it and may be incised
for surgical access.
The falciform ligament (choice A) is a mesenteric membrane between the liver and the
anterior abdominal wall. This ligament is within the greater peritoneal sac and does not
separate it from the lesser peritoneal sac.
The gastrosplenic ligament (choice C) passes from the greater curvature of the stomach to
the spleen. It separates the greater peritoneal sac from the left portion of the lesser
peritoneal sac. Incision of this structure would be the appropriate surgical approach to gain
access to the left side of the lesser peritoneal sac.
The hepatoduodenal ligament (choice D) is part of the lesser omentum and separates the
greater peritoneal sac from the right portion of the lesser peritoneal sac. It forms the anterior
border of the epiploic foramen. However, the hepatoduodenal ligament contains the
common bile duct, the proper hepatic artery, and the portal vein, and therefore may not be
incised for surgical access.
The splenorenal ligament (choice E) passes from the spleen to the parietal peritoneum on the
anterior surface of the left kidney. It separates the greater peritoneal sac from the left
portion of the lesser peritoneal sac. This ligament contains the splenic artery, splenic vein,
and the tail of the pancreas, and therefore may not be incised to gain access to the lesser
peritoneal sac.

Following a fracture of the humerus, which of the following is responsible for producing the
majority of the new bone that will reunite the two fragments?
A. Cancellous bone
B. Cartilage
C. Compact bone
D. Marrow
E.

Periosteum

The correct answer is E. When the periosteum is torn during a fracture, it supplies cells that
develop into osteoblasts and are the major producers of the new bone that reunites the two
ends. Heterotopic ossification (bone formed outside the regular bone) can occur as a
complication of fracture if some of the osteoblastic cells are misdirected into adjacent
tissues. The periosteum assists in the attachment of the osteoblasts to surrounding tissues
and to associated tendons and ligaments. This cellular layer functions in bone growth and
participates in repair after an injury.
Pre-existing cancellous bone (choice A) and compact bone (choice C) are not the major
source of osteoblasts that form the new bone.
Cartilage (choice B) and marrow (choice D) do not contribute to new bone formation after
fracture.

Which of the following cell types is derived from neuroepithelial cells?


A. Astrocytes
B. Enterochromaffin cells
C. Melanocytes
D. Odontoblasts
E.

Schwann cells

The correct answer is A. Astrocytes and oligodendrocytes are both derived from glioblasts,
which, in turn, are derived from neuroepithelial cells. Other neuroepithelial cell derivatives
include neuroblasts and ependymal cells. The astrocytes are the largest and most numerous
glial cells. These cells are responsible for maintaining the blood-brain barrier, creating a
three-dimensional framework for the central nervous system, performing repairs in damaged
neural tissues, and controlling the interstitial environment.
All the other choices are derived from neural crest cells. Other neural crest derivatives
include the neurons of the parasympathetic and sympathetic ganglia (including the adrenal
medulla), the dorsal root ganglia of the peripheral nervous system, the sensory ganglia of
cranial nerves V, VII, IX, and X, and the leptomeninges (pia and arachnoid).

A man's chest is compressed during a car accident, causing a posterior displacement of the
clavicle at the sternoclavicular joint. Which of the following structures would be most at risk?
A. Aorta
B. Esophagus
C. Heart
D. Superior vena cava
E.

Trachea

The correct answer is E. The sternoclavicular joint is quite strong and dislocates only with
difficulty. Dislocations, particularly posterior dislocations, are occasionally seen and may be
dangerous because of impingement onto the trachea, causing respiratory difficulties. The
trachea is a tough flexible tube with a diameter of approximately 2.5 cm and a length of
approximately 11 cm. The trachea begins anterior to the vertebra C6 in a ligamentous
attachment to the cricoid cartilage and ends in the mediastinum at the level of vertebra T5.
At this point, it branches to form the right and left primary bronchi.
Both the aorta (choice A) and the superior vena cava (choice D) are more posterior than the
trachea and are therefore less vulnerable.
The esophagus (choice B) is located behind the trachea and is less vulnerable.
The heart (choice C) lies below the clavicle and would not be at particular risk.

Damage to the upper fibers of the trapezius muscle would most likely impair which of the
following movements?
A. Depression of the medial end of the scapula
B. Elevation of the acromion
C. Flexion of the neck
D. Raising of the medial border of the scapula
E.

Turning of the face sideways

The correct answer is B. The trapezius is a large muscle arising in the posterior midline from
the superior nuchal line of the skull, ligamentum nuchae, and spinous processes of all of the
thoracic vertebrae. The upper fibers, which are potentially damaged by wounds to the
posterior and lateral neck, insert on the crest of the spine of the scapula, the medial border
of the acromion, and the lateral third of the clavicle. The upper fibers serve to elevate the
acromion.
The lower fibers of the trapezius depress the medial end of the scapula (choice A).
The sternocleidomastoids are important in extension of the head at the atlanto-occipital joint
and flexion of the cervical vertebral column (choice C).
The rhomboids and levator scapulae are important in raising the medial border of the scapula
(choice D).
Contraction of a single sternocleidomastoid muscle is important in turning of the face
sideways (choice E) in the contralateral direction.

During anatomy lab, a dental student notes a fibrous band that runs on the visceral surface of
the liver. It is attached on one end to the inferior vena cava and on the other end to the left
branch of the portal vein. In the embryo, this structure corresponds to the
A. ductus venosus
B. ligamentum teres
C. ligamentum venosum
D. umbilical arteries
E.

umbilical vein

The correct answer is A. This question could have tricked you if you didn't catch the key
words, "in the embryo." If you read the question too quickly and thought you were going to
be asked to identify the structure described, you probably chose choice C (ligamentum
venosum) because that is indeed the structure in question. In the embryo, however, this
fibrous band is actually the ductus venosus. The ductus venosus is an embryonic vessel that
allows blood to bypass the fetal liver; this prevents the depletion of oxygen and nutrient-rich
blood in the hepatic sinusoids.
The embryonic umbilical arteries (choice D) become the medial umbilical ligaments.
The embryonic umbilical vein (choice E) actually becomes the fibrous ligamentum teres
(choice B). The ligamentum teres is located in the free margin of the falciform ligament.

A person sits with his legs together and then opens his legs against lateral resistance. Which
nerve controls this type of movement?
A. deep peroneal nerve
B. femoral nerve
C. obturator nerve
D. sciatic nerve
E.

superior gluteal nerve

The correct answer is E. The motion described is hip abduction. The superior gluteal nerve
from roots L4-S1 is responsible for hip abduction.
The deep peroneal nerve (choice A) controls dorsiflexion of the foot. If you see a patient on
the NBDE with "foot drop," think of damage to the deep peroneal nerve.
The femoral nerve (choice B) is responsible for knee extension.
The obturator nerve (choice C) is responsible for hip adduction. The hip adduction machine
would provide resistance to the medial aspects of the thighs, and the individual would push
his/her legs together against the machine's resistance.
The sciatic nerve (choice D) controls knee flexion.

A biopsy specimen demonstrates a ciliated columnar epithelium. From which of the following
locations in the female genital tract was the biopsy obtained?
A. Cervix
B. Endometrium
C. Fallopian tube
D. Ovary
E.

Vagina

The correct answer is C.The fallopian tube is the only structure in the female genital tract
with a ciliated columnar epithelium; the beating of the cilia helps move the egg into the
uterus. This fact is also sometimes clinically helpful because dilated and deformed fallopian
tubes can be microscopically distinguished from cystic ovarian tumors by the presence of the
cilia. Each uterine tube is a hollow, muscular tube measuring approximately 13 cm in length.
Each uterine tube is divided into three regions: the infundibulum, the ampulla, and the
isthmus.
The cervix (choice A) and vagina (choice E) are lined by squamous epithelium.
The endometrium (choice B) is lined by columnar epithelium (although a few ciliated cells
may be present).
The covering of the ovary (choice D) is cuboidal epithelium, and cysts within the ovary can be
lined by cuboidal or nonciliated columnar epithelium.

A CT scan of the head demonstrates a mass in the olfactory groove area. The axons likely
compressed by this mass project to which of the following structures?
A. Insula
B. Nucleus ambiguus
C. Postcentral gyrus
D. Precentral gyrus
E.

Pyriform cortex

The correct answer is E. The olfactory nerves can be damaged by head injury, severe
infection, and tumors (such as meningiomas) that may compress the olfactory bulbs.
Unilateral damage is often not noticed by the patient, but bilateral damage (which
surprisingly can also be missed if it develops slowly) may lead to occasional bizarre errors in
odor identification. The olfactory nerves are technically the neurons on the mucosal side of
the cribriform plate that send processes through the plate to synapse in the olfactory bulb,
which in turn sends axons to the pyriform cortex (the primary olfactory cortex).
The insula (choice A) is thought to contain the primary gustatory (taste) cortex.
The nucleus ambiguus (choice B) provides the special visceral efferent fibers carried by the
glossopharyngeal and vagus nerves.
The postcentral gyrus (choice C) contains the primary sensory cortex.
The precentral gyrus (choice D) contains the primary motor cortex.

An atrial septal defect results from failure of the


A. ostium primum to form within the septum primum
B. ostium secundum to form within the septum primum
C. septum primum to fuse with the endocardial cushions
D. septum primum to fuse with the septum secundum
E.

septum secundum to fuse with the endocardial cushions

The correct answer is C. The septum primum (first interatrial septum) develops by growing
from the cranial end of the embryonic atrium toward the endocardial cushions. The gap that
exists between the two atria during this period is the ostium primum. As the septum primum
continues its growth, the ostium primum gets smaller until it is closed when the septum
primum completes its growth and completely fuses with the endocardial cushions. Failure of
the septum primum to fuse completely with the endocardial cushions leaves a persistent
ostium primum, known as a primum-type atrial septal defect.
The ostium primum does not form within the septum primum (choice A). The ostium primum
is the communication between the two atria that exists during the formation of the septum
primum. That is, the ostium primum is the space within the developing atrium not yet
occupied by the septum primum.
The ostium secundum normally forms within the septum primum (choice B) before the
ostium primum closes by fusion of the septum primum with the endocardial cushions. Failure
of the ostium secundum to form would result in embryonic death because there would be no
pathway for blood to pass from the right atrium to the left atrium when the ostium primum
closes, thus depriving the embryo of oxygenated blood.
Most of the septum primum normally disappears. The part that remains forms the valve of
the foramen ovale. This part of the septum primum normally does not fuse with the septum
secundum (choice D) during prenatal life. After birth, the valve of the foramen ovale is
pushed against the septum secundum as a result of the increased pressure in the left atrium.
This achieves functional closure of the foramen ovale. Fusion does not normally occur at this
time; it usually occurs later in life in most people. In some people, however, complete fusion
never occurs (probe patency).
The septum secundum normally does not fuse with the endocardial cushions (choice E).

If the nerve that accompanies the superior laryngeal artery is damaged, which of the
following functional losses will ensue?
A. Loss of sensation in the laryngeal mucosa above the vocal folds
B. Loss of sensation in the laryngeal mucosa below the vocal folds
C. Loss of sensation in the pharyngeal mucosa
D. Paralysis of the cricothyroid muscle
E.

Paralysis of the lateral cricoarytenoid muscle

The correct answer is A. The superior laryngeal artery is a branch of the superior thyroid
artery. It enters the larynx by passing through the cricothyroid membrane. In this region, it is
accompanied by the internal branch of the superior laryngeal nerve. This nerve provides
sensory innervation to the laryngeal mucosa above the vocal folds.
The laryngeal mucosa below the vocal folds (choice B) receives its sensory innervation from
the recurrent laryngeal nerve.
The pharyngeal mucosa (choice C) receives its sensory innervation from the glossopharyngeal
nerve.
The cricothyroid muscle (choice D) receives its motor innervation from the external branch of
the superior laryngeal nerve.
The lateral cricoarytenoid muscle (choice E) receives its motor innervation from the recurrent
laryngeal nerve.

A mass is noted at the back of a young man's tongue. A biopsy's pathology report comes back
with a diagnosis of normal thyroid tissue. This finding is related to the embryonic origin of the
thyroid near which of the following structures?
A. First pharyngeal pouch
B. Foramen cecum
C. Nasolacrimal duct
D. Second pharyngeal arch
E.

Third pharyngeal pouch

The correct answer is B. The thyroid gland originates as a mass of endodermal tissue near
the foramen cecum, which is near the tuberculum impar (which becomes the central part of
the tongue). During development, the thyroid descends in front of the pharynx, maintaining a
connection to the tongue via the thyroglossal duct. Usually, the thyroglossal duct disappears.
Uncommonly, residual ectopic thyroid tissue can be left anywhere along the path, including
at the back of the tongue. (In rare cases, all of the thyroid tissue remains at this site, forming
a mass that should not be excised, for obvious reasons!) Anatomically, the thyroid gland
curves across the anterior surface of the trachea just below the thyroid cartilage that forms
the majority of the anterior surface of the larynx. The two lobes of the thyroid gland are
united by a slender connection, the isthmus.
The first pharyngeal pouch (choice A) develops into the middle ear and eustachian tube.
The nasolacrimal ducts (choice C) connect the eyes to the oropharynx.
The second pharyngeal arch (choice D) develops into many muscles of the face and the
styloid process of the temporal bone.
The third pharyngeal pouch (choice E) develops into the thymus and inferior parathyroid
glands.

Which of the following cranial nerves carries the pain sensation from the tip of tongue?
A. V2
B. V3
C. VII
D. IX
E.

The correct answer is B. This question is essentially asking "which of the following nerves
innervates the tip of the tongue?" The innervation of the tongue is complex. The mandibular
division of the trigeminal nerve (V3) carries general somatic sensation from the anterior twothirds of the tongue. The mandibular branch is also a sensory nerve for the lower gingiva,
teeth, and lips, as well as the palate.
The maxillary division (V2, choice A) carries somatic sensation from the palate, upper gingiva,
and upper lip, as well as the lower eyelid, cheek, nose, and a portion of the pharynx.
The facial nerve (VII, choice C) carries taste from the anterior two-thirds of the tongue, as
well as innervating the lacrimal gland and the submandibular and sublingual salivary glands.
The glossopharyngeal nerve (IX, choice D) carries sensation and taste from the posterior onethird of the tongue.
The vagus nerve (X, choice E) carries sensation from the lower pharynx.

The primary arterial supply to the nasal mucosa is a direct branch of which of the following
arteries?
A. Facial artery
B. Maxillary artery
C. Superficial temporal artery
D. Superior labial artery
E.

Transverse facial artery

The correct answer is B. The major source of blood supply to the nasal mucosa is the
sphenopalatine artery, which is the terminal branch of the maxillary artery. The
sphenopalatine artery enters the nasal cavity from the pterygopalatine fossa by passing
through the sphenopalatine foramen in the lateral wall of the nasal cavity.
The facial artery (choice A) is a direct branch of the external carotid artery. It provides most
of the blood supply to the superficial face.
The superficial temporal artery (choice C) is a terminal branch of the external carotid artery.
The other terminal branch is the maxillary artery. The superficial temporal artery provides
blood supply to the temporal region and the lateral portion of the scalp.
The superior labial artery (choice D) is a branch of the facial artery. The superior labial artery
provides blood supply to the upper lip. It has a septal branch that provides some of the blood
supply to the anterior portion of the septal mucosa of the nasal cavity. However, this is not
the major blood supply to the nasal cavity.
The transverse facial artery (choice E) is a branch of the superficial temporal artery. It
provides blood supply to the parotid gland, the parotid duct, and the skin of the lateral face.

Which of the following tissues normally has the highest percentage of mucus-secreting cells?
A. Esophageal mucosa
B. Oral mucosa
C. Parotid gland
D. Sublingual gland
E.

Submandibular gland

The correct answer is D. The sublingual salivary glands are located beneath the mucous
membrane of the floor of the mouth. Numerous sublingual ducts open along either side of
the lingual frenulum. Salivary glands can contain predominantly serous cells, predominantly
mucous cells, or both in their acini. As you proceed from the midline laterally, acini in the
sublingual gland are almost pure mucous cells, whereas acini in the submandibular gland
(choice E) contain a mixture of serous and mucous cells. Acini in the parotid gland (choice C)
are mostly pure serous cells.
The esophageal mucosa (choice A) and the oral mucosa (choice B) are squamous epithelia
that do not form acini.

In cases of dysphagia where the esophagus is compressed, which structure would be most
likely to cause the compression?
A. Left atrium
B. Left ventricle
C. Pulmonary trunk
D. Right atrium
E.

Right ventricle

The correct answer is A. The left atrium forms most of the posterior wall of the heart. The
esophagus passes immediately posterior to the heart. Enlargement of the left atrium may
compress the esophagus and cause dysphagia. Anatomically, the esophagus begins posterior
to the cricoid cartilage, at the level of vertebrum C6. From this point, it descends toward the
thoracic cavity posterior to the trachea, passes inferiorly along the dorsal wall of the
mediastinum, and enters the abdominopelvic cavity through an opening in the diaphragm,
the diaphragmatic hiatus.
The left ventricle (choice B) forms most of the left border of the heart and most of the
diaphragmatic surface of the heart. The left ventricle is not related to the esophagus.
The pulmonary trunk (choice C) emerges from the right ventricle on the anterior surface of
the heart. The pulmonary trunk is not related to the esophagus.
The right atrium (choice D) forms the right border of the heart. It is not related to the
esophagus.
The right ventricle (choice E) forms most of the anterior wall of the heart and a small portion
of the diaphragmatic surface of the heart. It is not related to the esophagus.

The major structural component of the sperm flagellum is the:


A. microtubule
B. microfilament
C. actin filament
D. mysosin filament
E.

ciliary body

The correct answer is choice A. This is a basic histological question of a type found commonly
on NBDE, which asks about basic cellular organelles and structures. The sperm cell flagellum
is notable for a 9+2 arrangement of fibers formed from microtubules. The 9 single fibers
form a circular ring in cross section around the 2 doubled fibers in the center. One of the
significant facts about the fibers is that they are composed of microtubules, made up of
tubulin. The other significant fact is that all eucaryotic flagella and cilia have this same
structure (for example, cilia form the tracheal lining). Note that procaryotic (bacterial)
flagellea do NOT share this structure.

Which of the following embryonic structures gives rise to the adrenal cortex?
A. Ectoderm
B. Endoderm
C. Mesoderm
D. Mesonephros
E.

Neural crest cells

The correct answer is C. The mesoderm gives rise to the adrenal cortex. In addition, it also
gives rise to connective tissue, cartilage, bone, muscle, blood and lymph vessels, kidneys,
gonads, serous membranes lining body cavities, and the spleen.
The ectoderm (choice A) gives rise to the central nervous system, peripheral nervous system,
epidermis and its appendages, mammary glands, pituitary gland, tooth enamel, and the
neural crest.
The endoderm (choice B) gives rise to the parenchyma of the tonsils, thyroid and parathyroid
glands, thymus, liver, pancreas, the epithelial lining of the gastrointestinal and respiratory
tracts, urinary bladder, urethra, and auditory tube.
The mesonephros (choice D) functions as an interim kidney in the embryo.
The neural crest cells (choice E) give rise to cells of the spinal and cranial nerves, autonomic
ganglia, melanocytes, leptomeninges, connective tissue and bone of branchial arch origin,
and the adrenal medulla.

The x-ray of a child's arm after a fall appears to show a fracture near, but not at, the distal
end of the ulna. Before diagnosing a fracture, you should also consider the possibility that
this is actually which of the following?
A. Articular cartilage
B. Epiphyseal plate
C. Perichondrium
D. Primary ossification center
E.

Secondary ossification center

The correct answer is B. The epiphyseal plate of the bone contains cartilage that is
radiolucent. The plate in a bone that is not yet fully ossified can produce a "line" crossing the
bone near the end. This may be easily mistaken for a fracture by the inexperienced.
Anatomically, the epiphyseal plate separates the epiphysis from the diaphysis.
Articular cartilage (choice A) is radiolucent, but occurs at the very tip of the long bones.
Perichondrium (choice C) is usually difficult to see on x-ray.
Primary (choice D) and secondary (choice E) ossification centers are radiopaque.

A patient received a severe blow to the lateral side of the head, resulting in an epidural
hematoma. Which of the following blood vessels was most likely torn?
A. Anterior cerebral artery
B. Middle cerebral artery
C. Middle meningeal artery
D. Superficial temporal artery
E.

Superior cerebral vein

The correct answer is C. The middle meningeal artery is in the interior of the lateral portion
of the cranial cavity, embedded in the periosteal (outer) layer of the dura. A tear of this
artery results in blood entering the potential space between the outer dural layer and the
skull (epidural space), causing an epidural hematoma.
The anterior cerebral artery and middle cerebral artery (choices A and B) lie on the surface of
the brain. The anterior cerebral arteries supply the medial surface of the cerebral
hemispheres, and the middle cerebral arteries supply the lateral surface of the cerebral
hemispheres. A tear of either of these arteries would result in blood entering the
subarachnoid space (subarachnoid hemorrhage).
The superficial temporal artery (choice D) is a branch of the external carotid artery and is
external to the skull. This artery supplies the skin and other tissue of the temple region.
The superior cerebral veins (choice E) drain the cerebral hemisphere and enter the superior
sagittal sinus. A tear of these veins results in blood entering the potential space between the
dura and arachnoid (subdural space), causing a subdural hematoma.

A patient is unable to close her right eye. Physical examination reveals weakness of the right
orbicularis oculi. Which of the following additional symptoms would likely also be present?
A. Blurred vision
B. Hyperacusis
C. Inability to chew
D. Inability to feel the face
E.

Inability to shrug the shoulder

The correct answer B. The facial nerve innervates the muscles of facial expression. The
visceral motor portion innervates the lacrimal gland and nasal mucous glands via the
sphenopalatine ganglion. The submandibular and sublingual salivary glands are innervated
via the submandiublar ganglion. This patient has a lesion of the facial nerve (VII), which leads
to an inability to close the ipsilateral eye because of damaged motor fibers to the orbicularis
oculi. This patient would also lose her corneal reflex on that side because of an inability to
blink and would have ipsilateral paralysis of the muscles of facial expression distal to the
lesion. If the lesion affected the facial nerve more proximally, additional findings would be
hyperacusis (increased sensitivity to sound because of stapedius muscle paralysis), lack of
taste sensation in the anterior two-thirds of the tongue, and disturbed lacrimation and
salivation.
Blurred vision (choice A) could occur with lesions of the oculomotor (CN III), abducens (CN
VI), or trochlear (CN IV) nerves, which innervate the extraocular muscles. CN III innervates
the medial rectus, inferior rectus, superior rectus, and inferior oblique muscles. CN VI
innervates the lateral rectus and CN IV innervates the superior oblique.
An inability to chew (choice C) would probably be the result of a lesion of the trigeminal
nerve (CN V). Motor fibers of CN V innervate the muscles of mastication (temporalis,
masseter, and medial and lateral pterygoid muscles), and a lesion of these fibers may cause
the jaw to deviate to the side of the weak muscles.
An inability to feel the face (choice D) would also be the result of a CN V lesion. This lesion
could result in the ipsilateral loss of general sensation of the face and also of the mucous
membranes of the oral and nasal cavities.
A lesion of the accessory nerve (CN XI) would cause paralysis of the trapezius muscle, which
results in a sagging of the shoulder and a weakness in attempting to shrug the shoulder
(choice E).

A CT scan reveals a small tumor at the cerebellopontine angle of the brain. Which of the
following nerves is most likely to be affected by this tumor?
A. Facial nerve
B. Glossopharyngeal nerve
C. Optic nerve
D. Trigeminal nerve
E.

Vagus nerve

The correct answer is A. The facial nerve and the vestibulocochlear nerves emerge from the
brain stem at the cerebellopontine angle. These are the two nerves that will be initially
affected by a tumor in this region. The entire anatomy of the facial nerve is as follows: the
facial nerve originates from the taste receptors on the anterior 2/3 of the tongue (sensory)
and from nuclei of the pons (motor). It passes through the internal acoustic canal of the
temporal bone to reach the stylomastoid foramen.
The glossopharyngeal and vagus nerves (choices B and E) emerge from the brain stem at the
postolivary sulcus. This is caudal to the cerebellopontine angle.
The optic nerve (choice C) exits from the optic chiasm on the ventral surface of the
diencephalon. This is rostral to the cerebellopontine angle.
The trigeminal nerve (choice D) emerges from the brain stem at the anterolateral surface of
the pons. This is rostral and ventral to the cerebellopontine angle.

The extraocular muscles are derived from which of the following structures?
A. Branchial arches
B. Optic cup ectoderm
C. Somites
D. Somitomeres
E.

Splanchnic mesoderm

The correct answer is D. The somitomeres are specialized masses of mesoderm found in the
head region that give rise to the muscles of the head. The extraocular muscles are derived
from somitomeres 1, 2, 3, and 5.
The branchial arches (choice A) give rise to muscles of mastication (arch 1), muscles of facial
expression (arch 2), and muscles of the pharynx and larynx (arches 3-6), as well as additional
small muscles.
The optic cup ectoderm (choice B) gives rise to the muscles of the iris (sphincter and dilator
pupillae). These are the only muscles not formed from mesoderm.
Somites (choice C) give rise to the inferior muscles of the neck.
Splanchnic mesoderm (choice E) gives rise to smooth muscle of the viscera and the heart
muscle.

Which of the following respiratory system components is derived from neural crest?
A. Endothelial cells
B. Epithelium of primary bronchi
C. Laryngeal cartilage
D. Tracheal glands
E.

Type I pneumocytes

The correct answer is C. Laryngeal cartilages are derived from neural crest. The larynx is
composed of three cartilages that form the "body of the larynx": the thyroid cartilage, the
cricoid cartilage, and the epiglottis. The larynx also contains three pairs of smaller hyaline
cartilages: the arytenoid, corniculate, and cuneiform cartilages.
The endothelial cells (choice A), in the simple squamous epithelium that lines the pulmonary
capillaries, are derived from visceral mesoderm.
The epithelial lining of primary bronchi (choice B) is derived from endoderm.
Tracheal glands (choice D) and epithelium both derive from endoderm.
Type I pneumocytes (choice E) are derived from endoderm.

An elderly patient has had multiple small strokes lending to an absence of the gag reflex.
These findings suggest involvement of the nucleus of which of the following cranial nerves?
A. Facial (VII)
B. Glossopharyngeal (IX)
C. Hypoglossal (XII)
D. Spinal accessory (XI)
E.

Vestibulocochlear (VIII)

The correct answer is B. Cranial nerve IX is the glossopharyngeal nerve, which has a nucleus
in the medulla and is necessary for the gag reflex. The gag reflex is elicited by touching either
side of the posterior pharynx with a tongue blade, producing bilateral elevation of the palate
and bilateral contraction of the pharyngeal muscles. The afferent of this reflex arc consists of
the ipsilateral glossopharyngeal nerve, while the vagus nerve, bilaterally, supplies the
efferent limb. Although the glossopharyngeal nerve may seem to be one of the less
important cranial nerves, you should remember to test for its function, as a loss of gag reflex
can lead to the patient's death secondary to an aspiration pneumonia. The glossopharyngeal
nerve is a mixed sensory and motor nerve to the head and neck. It originates from the
posterior 1/3 of the tongue, pharynx, palate, and carotid arteries of the neck. Its destination
includes sensory nuclei of the medulla oblongata, as well as the pharyngeal muscles involved
in swallowing.
Cranial nerve VII (choice A) is the facial nerve, which supplies motor function to the face, but
it does not supply the oropharynx.
Cranial nerve XII (choice C) is the hypoglossal nerve, which supplies the intrinsic and most
extrinsic muscles of the tongue. It is not involved in the gag reflex.
Cranial nerve XI (choice D) is the spinal accessory nerve, which supplies the trapezius and
sternocleidomastoid.
Cranial nerve VIII (choice E) is the vestibulocochlear nerve, responsible for hearing and
equilibrium.

An ulcer damages an artery supplying the area of the greater curvature of the stomach.
Which artery is involved?
A. Left gastric
B. Left gastroepiploic
C. Right gastric
D. Right gastroepiploic
E.

Short gastric

The correct answer is D. The right gastroepiploic artery, off the gastroduodenal artery,
supplies the right half of the greater curvature of the stomach and could be directly affected
by ulceration of the greater curvature of the stomach at a site this close (4 cm) to the pyloric
sphincter.
The left gastric artery (choice A), off the celiac trunk, supplies the left half of the lesser
curvature of the stomach.
The left gastroepiploic artery (choice B), off the splenic artery, supplies the left half of the
greater curvature of the stomach. Although it anastomoses with the right gastroepiploic
artery, it is unlikely that this artery would be directly damaged by ulceration of the stomach
near the pyloric sphincter.
The right gastric artery (choice C), off the proper hepatic artery, supplies the right half of the
lesser curvature of the stomach.
The short gastric artery (choice E), actually one of several (4 to 5) short gastric arteries, off
the splenic artery (occasionally the left gastroepiploic), supplies the fundus of the stomach,
which is the most distant from the pylorus.

Which of the following sites contains striated muscle that is not under voluntary control?
A. Bladder
B. Colon
C. Esophagus
D. Gallbladder
E.

Stomach

The correct answer is C. Striated (skeletal) muscle not under voluntary control is an unusual
feature of the upper and middle thirds of the esophagus. The middle third of the esophagus
contains roughly half striated and half smooth muscle; the lower third contains only smooth
muscle. All the other structures listed in the answer choices contain smooth muscle.
As a side note, cardiac muscle cells do not rely on nerve activity to start a contraction.
Instead, specialized pacemaker cells establish a regular rate of contraction. Because these
pacemaker cells regulate the heart and the central nervous system does not, cardiac muscle
is also considered striated, involuntary muscle.

A mass in the anterior midline of the neck, slightly above the larynx is mobile and elevates
upon protrusion of the tongue. This mass is most likely a cyst that developed from which of
the following embryonic structures?
A. First pharyngeal cleft
B. First pharyngeal pouch
C. Second pharyngeal cleft
D. Second pharyngeal pouch
E.

Thyroglossal duct

The correct answer is E. The thyroglossal duct develops as an evagination of the floor of the
pharynx in the region where the tongue develops. The adult foramen cecum of the tongue
marks the site of this evagination. The distal end of this duct normally forms the thyroid
gland; the proximal part of the duct normally degenerates. Failure of a part of the duct to
degenerate may lead to a thyroglossal duct cyst or a median cervical cyst, as seen in this
patient.
The first pharyngeal cleft (choice A) forms the external ear canal. This cleft normally remains
patent.
The first pharyngeal pouch (choice B) forms the middle ear cavity and the auditory tube. This
pouch normally remains patent.
The second pharyngeal cleft (choice C) normally does not remain patent. It is typically
covered over by the overgrowth of the second pharyngeal arch. If part of this pouch does
remain patent, it may form a lateral cervical cyst, which is seen on the lateral side of the neck
along the anterior border of the sternocleidomastoid muscle.
The second pharyngeal pouch (choice D) forms the tonsillar fossa of the pharynx. The
pharyngeal mucosa in this area arises from the endoderm of the pouch. Ingrowth of
mesoderm cells results in the formation of the palatine tonsil.

From which of the following fetal vessels do the umbilical arteries arise?
A. Aorta
B. Carotid arteries
C. Ductus arteriosus
D. Iliac arteries
E.

Pulmonary arteries

The correct answer is D. The paired umbilical arteries arise from the iliac arteries. They
supply unoxygenated fetal blood to the placenta. The single umbilical vein takes the newly
oxygenated fetal blood from the placenta to the liver and then to the inferior vena cava via
the ductus venosus. Near the level of vertebra L4, the terminal segment of the abdominal
aorta divides to form the right and left common iliac arteries. These arteries carry blood to
the pelvis and lower limbs. As these arteries travel along the inner surface, they descend
behind the cecum and sigmoid colon, where each divides to form the internal iliac artery and
external iliac artery.

Which of the following locations in the embryo later forms the dorsal horn of the spinal cord?
A. Alar plate
B. Basal plate
C. Neural crest
D. Rostral end of neural tube
E.

Sulcus limitans

The correct answer is A. The spinal cord arises from the caudal end of the neural tube.
During development, an alar and a basal plate is formed, separated by a longitudinal groove
called the sulcus limitans (choice E). The alar plate forms the dorsal (posterior) part of the
spinal cord and becomes the sensory or afferent portion of the cord. The basal plate
(choiceB) is the ventral (anterior) part of the cord and becomes the motor, or efferent,
portion of the spinal cord and therefore would contain anterior horn cells.
The neural crest (choice C) develops into multipolar ganglion cells of autonomic ganglia,
pseudounipolar cells of spinal and cranial nerve ganglia, leptomeningeal cells, Schwann cells,
melanocytes, chromaffin cells of the adrenal medulla, and odontoblasts.
The brain forms from the rostral end of the neural tube (choice D).

Damage to which of the following structures might produce hair cell loss?
A. Basilar membrane
B. Organ of Corti
C. Reissner's membrane
D. Scala tympani
E.

Scala vestibuli

The correct answer is B. Hearing is the detection of sound, which consists of pressure waves
conducted through air or water. The receptors of the cochlear duct provide us with a sense of
hearing that allows us to detect the quietest whisper and yet remain functional in a crowded,
noisy environment. The organ of Corti contains hair cells from the cochlear branch of the
vestibulocochlear nerve (CN VIII). These cells rest on the basilar membrane (choice A), which
separates the scala tympani (choice D) from the scala media. The hair cells are embedded in
the tectorial membrane, and movement of the basilar membrane below the cells causes the
hairs to bend, which generates action potentials. The tectorial membrane that lies on the hair
cells does not form a boundary between the different scala; the membrane separating the
scala media from the scala vestibuli (choice E) is Reissner's (vestibular) membrane (choice C).

Despite blockage of the celiac trunk, the organs receiving their blood supply from the trunk
continue to operate normally. This is due to anastomoses between which vessels?
A. Left gastroepiploic artery and right gastroepiploic artery
B. Left gastroepiploic artery and right gastroepiploic artery
C. Proper hepatic artery and gastroduodenal artery
D. Right colic artery and middle colic artery
E.

Superior pancreaticoduodenal artery and inferior pancreaticoduodenal artery

The correct answer is E. The superior pancreaticoduodenal artery is a branch of the


gastroduodenal artery, which is a branch of the common hepatic artery, itself a branch of the
celiac trunk. The inferior pancreaticoduodenal artery is a branch of the superior mesenteric
artery. Occlusion of the celiac trunk would allow blood from the superior mesenteric artery
to reach the branches of the celiac trunk via the connections between the superior and
inferior pancreaticoduodenal arteries.
The left gastric and right gastric arteries (choice A) both receive their blood from the celiac
trunk. The left gastric artery is a direct branch of the celiac trunk. The right gastric artery is
usually a branch of the proper hepatic artery, which is a branch of the common hepatic
artery (a branch of the celiac trunk).
The left and right gastroepiploic arteries (choice B) both receive their blood supply from the
celiac trunk. The left gastroepiploic artery is a branch of the splenic artery, which is a branch
of the celiac trunk. The right gastroepiploic artery is a branch of the gastroduodenal artery,
which is a branch of the common hepatic artery (a branch of the celiac trunk).
The proper hepatic and gastroduodenal arteries (choice C) are branches of the common
hepatic artery, which is a branch of the celiac trunk.
The right colic and middle colic arteries (choice D) are both branches of the superior
mesenteric artery.

Biopsy demonstrates epithelial metaplasia. Which of the following cell types was most likely
observed in the involved areas?
A. Ciliated columnar epithelium
B. Cuboidal epithelium
C. Keratinizing squamous epithelium
D. Nonciliated columnar epithelium
E.

Nonkeratinizing squamous epithelium

The correct answer is D. The medical condition is Barrett's esophagus, in which the normally
nonkeratinizing squamous epithelium (choice E) of the esophagus undergoes metaplasia to
gastric or intestinal-like epithelium composed of nonciliated columnar epithelial cells.
Barrett's esophagus typically develops in the setting of chronic gastroesophageal reflux and
significantly increases the risk of later development of adenocarcinoma of the distal
esophagus.
Ciliated columnar epithelium (choice A) is found in the respiratory tract.
Cuboidal epithelium (choice B) is found in the kidney, peritoneal lining, and pleural lining.
Keratinizing squamous epithelium (choice C) is found in skin.
Nonkeratinizing squamous epithelium (choice E), in addition to being the normal epithelium
of the esophagus, is found in mouth, nose, and vagina.

As a result of a viral infection, a patient has swelling of the left facial nerve within the facial
canal. The patient's face appears asymmetrical, and he complains that saliva drips from his
mouth while he is chewing. Paralysis of which of the following muscles accounts for these
symptoms?
A. Buccinator
B. Masseter
C. Palatoglossus
D. Palatopharyngeus
E.

Temporalis

The correct answer is A. Compression of the facial nerve within the facial canal may result in
facial palsy (Bell's palsy). Because the muscles on one side of the face are paralyzed, the face
appears asymmetrical. The buccinator muscle, which is located within the cheek and is
innervated by the facial nerve, functions to hold food against the teeth while it is being
chewed. Paralysis of this muscle can result in food and saliva accumulating between the
teeth and the cheek. The buccinator originates from the alveolar processes of the maxilla and
mandible. It inserts into the fibers of the obicularis oris.
The masseter and temporalis muscles (choices B and E) are innervated by the mandibular
division of the trigeminal nerve. These muscles of mastication function to elevate the
mandible.
The palatoglossus (choice C) and palatopharyngeus (choice D) muscles are innervated by the
vagus nerve. The palatoglossus, with its mucosal covering, forms the palatoglossal fold
(anterior pillar of the fauces), immediately anterior to the palatine tonsil. This muscle
functions to draw the tongue and soft palate closer together, as occurs during swallowing.
The palatopharyngeus, with its mucosal covering, forms the palatopharyngeal fold (posterior
pillar of the fauces), which is immediately posterior to the palatine tonsil. This muscle causes
elevation of the pharynx, as occurs during swallowing.

A surgeon inadvertently sections the recurrent laryngeal nerve during a procedure. Which of
the following muscles would retain its innervation subsequent to this injury?
A. Cricothyroid
B. Lateral cricoarytenoid
C. Posterior cricoarytenoid
D. Thyroarytenoid
E.

Vocalis

The correct answer is A. The recurrent laryngeal nerve is a branch of the vagus nerve, which
innervates all of the intrinsic laryngeal muscles, except for the cricothyroid muscle. The
cricothyroid is attached to the cricoid cartilage and the thyroid cartilage; contraction of this
muscle tends to stretch and adduct the vocal ligament. The cricothyroid is innervated by the
external laryngeal nerve.
The lateral cricoarytenoid muscle (choice B) is innervated by the recurrent laryngeal nerve
and is attached to the cricoid cartilage and the arytenoid cartilage. Its contraction causes
adduction of the vocal ligament.
The posterior cricoarytenoid muscle (choice C) is innervated by the recurrent laryngeal nerve
and is attached to the cricoid cartilage and the arytenoid cartilage. Its contraction causes
abduction of the vocal ligament.
The thyroarytenoid muscle (choice D) is innervated by the recurrent laryngeal nerve and is
attached to the thyroid cartilage and the arytenoid cartilage. Its contraction causes
slackening of the vocal ligament.
The vocalis muscle (choice E) is the most medial part of the thyroarytenoid muscle. It
attaches to either the thyroid cartilage and the vocal ligament, or the arytenoid cartilage and
the vocal ligament. It is innervated by the recurrent laryngeal nerve. Its contraction causes
tension on segments of the vocal ligament.

Zygomycosis, a destructive fungal infection of the sinuses, is likely to reach the brain by which
of the following routes?
A. Cavernous sinus
B. External carotid artery
C. Internal carotid artery
D. Superior sagittal sinus
E.

Superior vena cava

The correct answer is A. This question requires knowledge of pathophysiology with a basic
understanding of anatomy. The cavernous sinuses are located on either side of the body of
the sphenoid bone and become a potential route of infection because they receive blood
both from the face (via the ophthalmic veins and sphenoparietal sinus) and from some of the
cerebral veins. The spread of infection into the cavernous sinus can produce either central
nervous system (CNS) infection or cavernous sinus thrombosis, both of which are potentially
fatal.
The route from the face to the brain is not arterial (choices B and C).
The superior sagittal sinus (choice D) is located in the falx cerebri and drains venous blood
from the brain to other dural sinuses, from which it eventually drains into the jugular vein.
Zygomycosis does not reach the brain by way of the superior sagittal sinus.
The superior vena cava (choice E) drains blood from the upper part of the body into the
heart.

Following a surgical procedure on the right side of the neck, a patient can no longer raise his
right arm above the horizontal position. The patient also cannot shrug his right shoulder.
Which of the following nerves was injured?
A. Axillary nerve
B. Great auricular nerve
C. Greater occipital nerve
D. Spinal accessory nerve
E.

Transverse cervical nerve

The correct answer is D. The spinal accessory nerve crosses the posterior triangle of the neck
immediately deep to the investing fascia of the neck. This nerve innervates the trapezius
muscle, which is responsible for upward rotation and elevation of the scapula. A lesion of this
nerve in the posterior triangle leads to paralysis of the trapezius. Without the ability to
upwardly rotate the scapula, abduction and flexion of the arm above the horizontal plane is
not possible. Also, shrugging of the shoulder is impaired with paralysis of the trapezius
muscle.
The axillary nerve (choice A) does not pass through the neck. It is a branch of the brachial
plexus, and it leaves the axilla to innervate the deltoid and teres minor muscles.
The great auricular nerve (choice B) and transverse cervical nerve (choice E) are branches of
the cervical plexus, which provide cutaneous innervation to the skin of the neck. No muscles
are innervated by these nerves.
The greater occipital nerve (choice C) is the dorsal ramus of the second cervical spinal nerve.
It provides cutaneous innervation to the skin of the back of the head. No muscles are
innervated by this nerve.

Microscopic examination of a PAS-stained histological section of a Graafian follicle


demonstrates a bright reddish-pink, acellular ring around the ovum. Which of the following
terms most accurately describes this ring?
A. Corona radiata
B. Cumulus oophorus
C. Theca externa
D. Theca interna
E.

Zona pellucida

The correct answer is E. The ring described is the zona pellucida, which surrounds the ovum.
The zona pellucida is rich in polysaccharides and glycoproteins and consequently stains
brightly pink or red with PAS stain. Binding of the sperm cell membrane to the zona pellucida
triggers the acrosome reaction, during which acrosomal enzymes are released that digest the
zona pellucida, allowing the spermatozoon to contact and fuse with the ovum cell
membrane. Anatomically, as layers of granulosa cells develop around the primary oocyte,
microvilli from the surrounding granulosa cells intermingle with cells of the primary oocyte.
The microvilli are surrounded by a layer of glycoproteins, and the entire region is called the
zona pellucida.
The follicular cells immediately outside the zona pellucida form the corona radiata (choice A).
The larger cumulus oophorus (choice B) is the hill of follicular cells that surrounds the ovum.
The theca interna (choice D) and externa (choice C) are formed from the connective tissue
surrounding the follicle.

The smooth part of the right atrium derives from which of the following embryonic
structures?
A. Bulbus cordis
B. Primitive atrium
C. Primitive ventricle
D. Sinus venosus
E.

Truncus arteriosus

The correct answer is D. The smooth part of the right atrium (the sinus venarum) is derived
from the sinus venosus. The coronary sinus and the oblique vein of the left atrium also derive
from the sinus venosus.
The bulbus cordis (choice A) gives rise to the smooth part of the right ventricle (conus
arteriosus) and the smooth part of the left ventricle (aortic vestibule).
The primitive atrium (choice B) gives rise to the trabeculated part of the right and left atria.
The primitive ventricle (choice C) gives rise to the trabeculated part of the right and left
ventricles.
The truncus arteriosus (choice E) gives rise to the proximal part of the aorta and the proximal
part of the pulmonary artery.

Which of the following structures does the fetal allantoic duct become in the adult?
A. Cloaca
B. Medial umbilical ligament
C. Urachus
D. Ureter
E.

Urethra

The correct answer is C. The urachus is a fibrous remnant that extends from the umbilicus to
the urinary bladder. It is also known as the median umbilical ligament of the anterior
abdominal wall.
The cloaca (choice A) is the primitive, endoderm-lined region that receives the terminal
portion of the hindgut. It is later subdivided into urogenital and anal areas.
The medial umbilical ligament (choice B) is a paired structure located deep to the peritoneum
of the anterior abdominal wall. It is formed by the obliterated umbilical artery.
The ureter (choice D) is the muscular tube that conveys urine from the kidney to the urinary
bladder.
The urethra (choice E) is the passageway that carries urine from the bladder to the perineum.

Attempts to straighten out a flexed thigh cause great pain in a patient with appendicitis. This
is due to the position of the appendix near which muscle?
A. Adductor magnus
B. Biceps femoris
C. Gluteus maximus
D. Gracilis
E.

Psoas major

The correct answer is E. The path of the psoas major lies in the retroperitoneum and comes
close to the appendix. Acute appendicitis can cause either infection or a sympathetic
inflammation of the psoas. This produces clinically a "positive psoas sign," in which attempts
to straighten the patient's flexed (to relieve pain) hip produce sometimes marked
exacerbation of the pain. None of the other muscles listed pass near the appendix.

An otherwise healthy student taking no medications is concerned because he has noticed


several painless uniform "large bumps" at the back of his tongue. These are most likely
A. aphthous ulcers
B. candidal colonies
C. circumvallate papillae
D. filiform papillae
E.

fungiform papillae

The correct answer is C. The large bumps at the back of his tongue are circumvallate papillae.
These are large circular structures surrounded by moat-like depressions. The lateral surfaces
of these papillae contain taste buds. There are also small serous-only salivary glands in these
papillae.
Aphthous ulcers (choice A) are small, white, or red mouth lesions.
Candidal colonies (choice B) appear in thrush, which occurs more commonly in the
immunocompromised host or in those taking antibacterial drugs. You are told that the
patient is healthy and not taking medications, making this condition unlikely.
Filiform papillae (choice D) are the most numerous papillae of the tongue. They are small,
elongated cones that create the tongue's rough texture. They do not contain taste buds.
Fungiform papillae (choice E) are mushroom-shaped structures scattered among the filiform
papillae. They frequently contain taste buds. They are intermediate in size between filiform
and circumvallate papillae.

You are asked to hold your upper arm against your lateral chest wall, with the palm upward.
You then rotate the hand so that the palm faces downward, without bending the wrist. This
motion is known as:
A. abduction of the forearm
B. adduction of the forearm
C. flexion of the forearm
D. pronation of the forearm
E.

supination of the forearm

The correct answer is D. When the forearm is rotated from anatomic position (palms facing
forward, thumbs out) so that the palm faces posteriorly, the forearm is said to be pronated.
Abduction (choice A) raises the arm to a horizontal position away from the body; adduction
(choice B) is the reverse.
Flexion (choice C) brings the arm or forearm forward, in front of the plane of the body.
Rotation of the forearm so that the palm faces forward (i.e., into anatomic position) is
referred to as supination (choice E).

The tongue will move in which direction when protuded with surgical damage to the right
hypoglossal nerve?
A. Downward
B. Upward
C. Directly forward
D. To the right
E.

To the left

The correct answer is D. There are two ways to answer this question. The formula method is
that a tongue with muscle or nerve injury will protrude toward the side of injury, in this case
the right side. This is similar to the case of the mandible protruding toward the side of injury
when a lateral pterygoid is injured.The logical method is to imagine intrinsic muscles and
extrinsic protruders of the tongue on the right side not receiving stimulation from
innervation. In this case, only the left side protruders will operate. The tongue will protrude
only on the left side, with the immobile right side acting as a stationary pivot while the
tongue moves from left to right (toward the injured side).

A woman suffers a fracture of the left tenth and eleventh ribs. Which of the following organs
is most likely to have been injured by these fractured ribs?
A. Descending colon
B. Jejunum
C. Left adrenal gland
D. Left kidney
E.

Spleen

The correct answer is E. The spleen is a soft, friable organ with a thin capsule and is subject
to injury upon trauma to the left side of the abdomen. It is located in the upper left quadrant
of the abdomen, deep to the left ninth, tenth, and eleventh ribs. It is the most commonly
injured organ in the abdomen. The adult spleen contains the largest number of lymphoid
tissues in the human body.
The descending colon (choice A) lies in a retroperitoneal position on the left side of the
posterior abdominal wall. The descending colon begins at the splenic flexure immediately
inferior to the spleen.
The jejunum (choice B) is a peritoneal structure suspended by a long mesentery. It is located
primarily in the upper left quadrant of the abdomen. Its long mesentery allows the jejunum
to be highly mobile and thus is not likely to be injured by trauma to the body wall.
The left adrenal gland (choice C) is a retroperitoneal structure that lies near the upper pole of
the left kidney. It is embedded within fat and is thus well protected from injury.
The left kidney (choice D) is a retroperitoneal structure that is well protected by fat.

During embryological development, hematopoiesis occurs in different organs at different


times. Which of the following are the correct organs, in the correct sequence, at which
hematopoiesis occurs embryologically?
A. Amnion, yolk sac, placenta, bone marrow
B. Placenta, liver and spleen, yolk sac, bone marrow
C. Placenta, spleen and lymphatic organs, bone marrow
D. Yolk sac, bone marrow, liver and spleen
E.

Yolk sac, liver, spleen and lymphatic organs, bone marrow

The correct answer is E. By the third week of development, hematopoiesis begins in the
blood islands of the yolk sac. Beginning at 1 month of age and continuing until 7 months of
age, blood elements are also formed in the liver. Hematopoiesis occurs in the spleen and
lymphatic organs between 2 and 4 months, and in the bone marrow after 4 months.

The hormone most responsible for regulating sodium balance is secreted from:
A. Zona glomerulosa of the adrenal cortex
B. Zona reticularis of the adrenal medulla
C. Zona fasiculata of the adrenal medulla
D. Zona fasiculata of the adrenal cortex
E.

Zona glomerulosa of the adrenal medulla

The correct answer is choice A.Firstly, the hormone involved is aldosterone, which acts to
increase sodium resorption in the kidney. Note that aldosterone (a mineralcorticoid) and the
glucocorticoids (cortisol, cortisone) are produced by the adrenal cortex, not medulla. The
medulla, with a different developmental origin and cell type, produces catecholamines such
as epinephrine and norepinephrine. The cortex, which is outside the medulla, is in three
regions. On the outside is the Zona Glomerulosa, source of the mineralcorticoids. Inside of
that is the Zona Fasiculata, which together with the innermost layer of the cortex, the Zona
Reticularis, produce glucocortoids. Remember that interior to the Zona Reticularis, you will
find the adrenal medulla. Also note that as a memory aid, the cortex layers from the outside
in are G-F-R, like the GFR of the kidney.

A nursing home patient who aspirates while lying on his back would be most likely to develop
pneumonia involving which of the following sites?
A. Anterior segment of the right upper lobe
B. Apical segment of the right lower lobe
C. Inferior lingular segment of the left upper lobe
D. Lateral segment of the right middle lobe
E.

Superior lingular segment of the left upper lobe

The correct answer is B. Aspiration pneumonia is a common complication observed in


nursing home patients. The most probable site of the pneumonia can be anticipated by
knowing the anatomy of the bronchial tree because the aspirated fluid usually flows
downhill. In a supine or nearly supine patient, the fluid flows into the trachea and then into
either of the (typically the right) main bronchi. The first posteriorly located branch is the one
leading to the apical aspect of (either) lower lobe. The lateral and posterior segments of the
lower lobes are also supplied by posteriorly branching segmental bronchi. In contrast, the
posterior aspects of the upper lobes are somewhat protected by an initial anteriorly directed
bifurcation before their segmental bronchi arise. All other segments of the bronchial tree and
their corresponding portions of lung are more anterior.

The nucleus that lies immediately medial (and deep) to the uncus is the
A. amygdala
B. caudate nucleus
C. claustrum
D. hippocampus
E.

putamen

The correct answer is A. The uncus, which is the medial protrusion of the parahippocampal
gyrus, is an external structure seen on the ventral surface of the temporal lobe. The
amygdala is a collection of nuclei that lies directly beneath the uncus.
The caudate nucleus (choice B) is a deep nuclear structure that lies lateral to the lateral
ventricles.
The claustrum (choice C) is a thin and elongated nucleus that lies just medial to the insular
cortex.
The hippocampus (choice D) is a nuclear structure that lies in the interior of the
parahippocampal gyrus.
The putamen (choice E) is a nuclear structure that resides lateral to the caudate and medial
to the claustrum.

An injection to anesthetize pain from a fracture of the seventh rib should be gien in what
area?
Seventh intercostal space immediately below the seventh rib in the midclavicular
A. line
Seventh intercostal space immediately below the seventh rib just lateral to the angle
B. of the rib
Seventh intercostal space immediately below the seventh rib just medial to the
C. angle of the rib
D. Sixth intercostal space immediately above the seventh rib in the midclavicular line
E.

Sixth intercostal space immediately above the seventh rib just lateral to the angle of
the rib

The correct answer is B. The seventh intercostal nerve (the anterior ramus of the seventh
thoracic spinal nerve) innervates the seventh rib. After passing through the intervertebral
foramen between the seventh and eighth thoracic vertebrae, the nerve lies in the seventh
intercostal space. After passing the angle of the rib, it occupies a position along the lower
border of the rib, in the costal groove. Use of a local anesthetic at this point will anesthetize
the rib.
By the time the intercostal nerve has reached the midclavicular line (choice A), it has already
innervated most of the rib. Use of an anesthetic at this point would not be effective.
While the intercostal nerve is in the intercostal space medial to the angle of the rib (choice
C), it is not along the lower border of the rib.
The intercostal nerve does not lie along the upper border of the rib (choices D and E). Thus,
injection at these sites would not be effective.

If a patient has a drooping right eyelid and a dilated right pupil, which of the following neural
structures is most likely affected?
A. Cervical sympathetic chain
B. Facial nerve
C. Oculomotor nerve
D. Superior cervical ganglion
E.

Trigeminal nerve

The correct answer is C. The oculomotor nerve innervates the levator palpebrae superioris,
which elevates the eyelid. This nerve also innervates the inferior oblique muscles, as well as
the superior, inferior, and medial rectus muscles. The oculomotor nerve also contains
preganglionic parasympathetic fibers that synapse, in the ciliary ganglion, on postganglionic
parasympathetic nerve fibers that innervate the sphincter pupillae muscle, which constricts
the pupil. A lesion of the oculomotor nerve may therefore result in both drooping of the
eyelid (ptosis) and dilation of the pupil (mydriasis).
The cervical sympathetic chain (choice A) contains preganglionic sympathetic nerve fibers,
arising from the upper thoracic spinal cord, which ascend to the cervical sympathetic ganglia.
A lesion of these nerves may result in Horner's syndrome, which includes a ptosis and miosis
(pupillary constriction) and, often, anhidrosis (lack of sweating).
The facial nerve (choice B) innervates the muscles of facial expression, including the
orbicularis oculi muscle. A lesion of this nerve may therefore result in the inability to close
the eye.
The superior cervical ganglion (choice D) contains the cell bodies of postganglionic
sympathetic nerves that innervate structures in the head. A lesion of this structure will cause
Horner's syndrome.
The trigeminal nerve (choice E) provides sensory innervation to much of the head. A lesion of
this nerve may interfere with the corneal blink reflex.

A newborn infant has some of its abdominal viscera protruding through a defect in the
abdominal wall. Which of the following is the likely cause of this defect?
A. Failure of the intestinal loop to retract from the umbilical cord
B. Failure of the yolk stalk to degenerate
C. Failure of peritoneal fusion
D. Incomplete fusion of the lateral body folds
E.

Umbilical herniation

The correct answer is D. During the fourth week of development, the lateral body folds move
ventrally and fuse in the midline to form the anterior body wall. Incomplete fusion results in
a defect that allows abdominal viscera to protrude from the abdominal cavity, a condition
known as gastroschisis.
During development, the midgut normally herniates into the umbilical cord and then
subsequently retracts into the abdominal cavity. Failure of the intestinal loop to retract from
the umbilical cord (choice A) results in omphalocele.
Failure of the yolk stalk to degenerate (choice B) results in an ileal (Meckel's) diverticulum or
a vitelline fistula or cyst. In the early embryo, the gut tube is connected to the yolk sac by a
narrow connection known as the yolk stalk. Normally, this connection degenerates.
During development, certain peritoneal organs fuse with the posterior abdominal wall to
become secondarily retroperitoneal. Failure of this peritoneal fusion (choice C) will result in
certain organs that are normally immobile being mobile (e.g., mobile cecum).
Umbilical herniation (choice E) results from abdominal viscera protruding through a
weakness in the abdominal wall after development. Such protrusions are covered by
subcutaneous fascia and skin, distinguishing them from gastroschisis.

Which glandular area secretes hormones which are products of tyrosine metabolism?
A. Alpha cells of pancreas
B. Beta cells of pancreas
C. Adrenal cortex
D. Adrenal medulla
E.

Testes

The correct answer is D.The products of the adrenal medulla are epinephrine (adrenalin) and
norepinephrine (noradrenalin). The pathway of production of these compunds is a s follows:
tyrosine to DOPA to dopamine to norepinephrine to epinephrine. The hormone secreted by
alpha cells of the pancreas is glucagon, while beta cells secrete insulin. Both are peptide
hormones. The adrenal cortex secretes a variety of hormones including cortisol and
aldosterone, both steroid hormones. The testes secrete testosterone, also a steroid
hormone.

In preparation for a procedure to remove the fingernail on an index finger, the physician
would most likely anesthetize a branch of the
A. anterior interosseus nerve
B. median nerve
C. musculocutaneous nerve
D. radial nerve
E.

ulnar nerve

The correct answer is B. The median nerve supplies the surface of the lateral palm, the
palmar surface of the first three digits, and the distal dorsal surface of the index and middle
fingers (including the nail beds). Therefore, prior to performing surgery in this area, it is
essential to anesthetize a branch of this nerve (possibly a proper digital branch) to eliminate
pain sensation around the nail bed of the index finger. The median nerve of the branchial
plexus distributes to the flexor muscles on the forearm (flexor carpi radialis and palmaris
longus), the pronators (p. quadratus and p. teres), digital flexors, and skin over the lateral
surface of the hand.
Neither the anterior interosseus (choice A) nor the musculocutaneous (choice C) nerves
supplies the hand. The anterior interosseous nerve supplies the flexor pollicis longus, the
lateral half of flexor digitorum profundus, and pronator quadratus. The musculocutaneous
nerve supplies the coracobrachialis, biceps, and most of the brachialis muscle, then becomes
the lateral cutaneous nerve of the forearm.
The radial nerve (choice D) supplies skin on the radial side of the dorsal surface of the hand,
but not the fingertips.
The ulnar nerve (choice E) supplies the palmar and dorsal surfaces of the medial hand,
including the palmar and dorsal surfaces of the fourth and fifth digits.

The superior ophthalmic vein directly communicates with which of the following dural
venous sinuses?
A. Cavernous sinus
B. Occipital sinus
C. Sigmoid sinus
D. Superior petrosal sinus
E.

Straight sinus

The correct answer is A. The anterior continuation of the cavernous sinus, the superior
ophthalmic vein, passes through the superior orbital fissure to enter the orbit. Veins of the
face communicate with the superior ophthalmic vein. Because of the absence of valves in
emissary veins, venous flow may occur in either direction. Cutaneous infections may be
carried into the cavernous sinus and result in a cavernous sinus infection, which may lead to
an infected cavernous sinus thrombosis. The cavernous sinus is lateral to the pituitary gland
and contains portions of cranial nerves III, IV, V1, V2, and VI, and the internal carotid artery.
The occipital sinus (choice B) is at the base of the falx cerebelli in the posterior cranial fossa.
It drains into the confluence of sinuses.
The sigmoid sinus (choice C) is the anterior continuation of the transverse sinus in the middle
cranial fossa. The sigmoid sinus passes through the jugular foramen and drains into the
internal jugular vein.
The superior petrosal sinus (choice D) is at the apex of the petrous portion of the temporal
bone and is a posterior continuation of the cavernous sinus. The superior petrosal sinus
connects the cavernous sinus with the sigmoid sinus.
The straight sinus (choice E) is at the intersection of the falx cerebri and the falx cerebelli in
the posterior cranial fossa. The straight sinus connects the inferior sagittal sinus with the
confluence of sinuses.

During the process of meiosis, a single homologous chromosome pair fails to separate during
the first meiotic division. This failure would be most likely to produce which of the following
conditions if fertilization occurs and an embryo later develops?
A. Balanced translocation
B. Triploidy
C. Trisomy
D. Unbalanced translocation
E.

Uniploidy

The correct answer is C. Meiosis is cell division that produces gametes with half of the
normal somatic chromosome complement. The process described is nondisjunction, which
will cause one daughter cell to have 24 chromosomes, while the other will have 22
chromosomes. When a gamete with the normal 23 chromosomes combines at fertilization
with a gamete with 22 or 24 chromosomes, the embryo will have 47 chromosomes (trisomy)
or 45 chromosomes (monosomy). Nondisjunction can occur in either the first or second
meiotic division.
Balanced translocation (choice A) occurs when non-homologous chromosomes exchange
genetic material in such a way that no critical genetic material is lost.
Triploidy (choice B) is the term used when a cell has 69 chromosomes (3N or 3 sets), and can
occur in tumors or when an egg is fertilized by two sperm.
An unbalanced translocation (choice D) occurs when non-homologous chromosomes
exchange genetic material with a net loss or gain of critical genetic material.
Uniploidy (choice E) is the state of having 23 chromosomes, seen normally in sperm and eggs.

A knife wound to the neck damages the posterior cord of the brachial plexus. Which of the
following muscles would be most likely be paralyzed?
A. Deltoid
B. Flexor carpi ulnaris
C. Flexor digitorum superficialis
D. Flexor pollicis brevis
E.

Palmaris longus

The correct answer is A. The posterior cord supplies the axillary and radial nerves. Of the
muscles listed, only the deltoid is supplied by one of these two nerves, specifically the axillary
nerve. The deltoid originates from the clavicle and scapula. It inserts into the deltoid
tuberosity of the humerus. This muscle is responsible for abduction of the arm.
The flexor carpi ulnaris (choice B) is supplied by the ulnar nerve.
The flexor digitorum superficialis (choice C), the flexor pollicis brevis (choice D), and the
palmaris longus (choice E) are supplied by the median nerve.

A sharp instrument passing through the superior orbital fissure would most likely sever which
of the following structures?
A. Abducens nerve
B. Facial nerve
C. Mandibular nerve
D. Maxillary nerve
E.

Middle meningeal artery

The correct answer is A. A good way to remember what passes through the superior oribital
fissure is that everything that innervates the eye, other than the optic nerve, passes through
this fissure. This incudes the oculomotor nerve(CN III), the trochlear nerve (CN IV), the
ophthalmic nerve (V1), and the abducens nerve (CN VI).
The facial nerve (CN VII; choice B) passes through the internal auditory meatus.
The mandibular nerve (V3; choice C) passes through the foramen ovale.
The maxillary nerve (V2; choice D) passes through the foramen rotundum.
The middle meningeal artery (choice E) passes through the foramen spinosum.

In which of the following structures are the opacifications located with cataract formation?
A. Aqueous humor
B. Cornea
C. Lens
D. Optic nerve
E.

Retina

The correct answer is C. Cataracts are lens opacifications. It is not known whether senile
cataracts represent disease or normal opacification with age. Cataracts may occur as a
consequence of diabetes mellitus, long-term steroids, or congenital infections. They are
successfully treated at present with lens extraction and implantation of prosthetic lenses. The
diagnostic characteristics for cataracts are as follows: 1) blurred vision that is progressive
over months to years, 2) no pain or redness is seen, and 3) lens opacities may be invisible or
grossly visible.
Aqueous humor (choice A) is continually replaced due to active secretion by the ciliary body.
As such, it does not undergo opacification; it is in constant flux.
Corneal opacification (choice B) is generally a consequence of squamous metaplasia, in which
the transparent, nonkeratinized, squamous cells are replaced by opaque, keratinized,
squamous cells. Squamous metaplasia is a reparative process, usually due to friction injury to
the cornea or a vitamin A deficiency.
The optic nerve (choice D) is not transparent, and it does not undergo opacification injury. It
may, however, atrophy due to ischemic, traumatic, infective, or metabolic insults.
The retina (choice E) consists of multiple layers of neural cells. The retina is transparent but is
not the site of cataract formation.

Damage to the parasagittal region and falx cerebri will most likely result in which of the
following neurologic deficits?
A. Altered taste
B. Leg paralysis
C. Loss of facial sensation
D. Ptosis
E.

Unilateral deafness

The correct answer is B. A meningioma of the parasagittal region and the falx cerebri would
be located at the top of the brain, near the midline. In this position, it could compress the
sensory or motor cortex supplying the lower extremities. The falx cerebri is a fold of dura
mater that projects between the cerebral hemispheres in the longitudinal tissues. Its interior
portions attach anteriorly to the crista galli and posteriorly to the internal occipital crest.
Taste (choice A) is supplied by cranial nerves VII, IX, and X. These nerves arise from the
brainstem.
Facial sensation (choice C) is supplied by cranial nerve V, the nuclei of which are in the
brainstem. Furthermore, the area of the sensory cortex that subserves the face is on the
lateral aspect of the cortex and would not be affected by a tumor in the parasagittal region.
Ptosis (choice D) can be caused by a deficit in cranial nerve III, which arises from the
brainstem.
Unilateral deafness (choice E) suggests damage to cranial nerve VIII, which arises from the
brainstem.

Which of the following is a derivative of the second pharyngeal arch?


A. Eustachian tube
B. External auditory meatus
C. Palatine tonsil
D. Stylohyoid muscle
E.

Tensor tympani

The correct answer is D. The stylohyoid is derived from the second pharyngeal arch, which
also gives rise to the muscles of facial expression, the stapedius, the posterior belly of the
digastric muscle, Reichert's cartilage, and the facial nerve.
The Eustachian tubes (choice A) are derived from the first pharyngeal pouch, which also gives
rise to the middle ear cavity and the inner epithelial lining of the tympanic membrane.
The external auditory meatus (choice B is derived from the first pharyngeal cleft, which also
gives rise to the outer epithelial lining of the tympanic membrane.
The palatine tonsil (choice C) is derived from the epithelial lining of the second pharyngeal
pouch.
The tensor tympani (choice E) is derived from the first pharyngeal arch, which also gives rise
to the muscles of mastication, the anterior belly of the digastric muscle, the mylohyoideus,
the tensor veli palantini, the maxillary and mandibular bones, and the maxillary and
mandibular divisions of the trigeminal nerve.
Note that knowing the embryology of these structures helps you remember the innervation
of the muscles of the face.
First pharyngeal arch
Second pharyngeal arch muscles of facial expression facial nerve

Вам также может понравиться